Hock_Performance Mgmt

152
8/16/2019 Hock_Performance Mgmt http://slidepdf.com/reader/full/hockperformance-mgmt 1/152 Question 1 - ICMA 10.P1.142 - Performance Measures  A company is concerned that its divisional managers are not making decisions that are in the best interests of the overall corporation. In order to prevent this, the company should use a performance evaluation system that focuses on  A. residual income. B. operating income. C. controllable costs. D. flexible budget variances. A. When division managers are evaluated on the basis of residual income, they are more likely to embark upon projects that will be in the best interests of the overall corporation. Residual income is not a rate, but instead it is an amount of income. As long as the expected net income before taxes from the project is greater than the target monetary return in (calculated as a percentage of invested capital), the project will be in the best interests of the oveerall corporation and it will also have positive residual income. Using residual income overcomes a weakness inherent in using return on investment for manager evaluation. If return on investment is used, a project that would have a higher return than the company's cost of capital and thus would be good for the company may be rejected by the division manager because its return on investment is lower than the division's current return on investment and thus it would bring the division's return on investment down. B. A performance evaluation system that focuses on operating income would not necessarily encourage divisional managers to make decisions that are in the best interests of the company. C. A performance evaluation system that focuses on controllable costs would not necessarily encourage divisional managers to make decisions that are in the best interests of the company. D. A performance evaluation system that focuses on flexible budget variances would not necessarily encourage divisional managers to make decisions that are in the best interests of the company. Question 2 - ICMA 10.P1.105 - Manufacturing Input Variances -- Materials and Labor Lee Manufacturing uses a standard cost system with overhead applied based on direct labor hours. The manufacturing budget for the production of 5,000 units for the month of May included the following information. Direct labor (10,000 hours at $15 per hour) $150,000 Variable overhead 30,000 Fixed overhead 80,000 During May, 6,000 units were produced and the direct labor efficiency variance was $1,500 unfavorable. Based on this information, the actual number of direct labor hours used in May was  A. 9,500 hours. B. 12,100 hours. C. 11,900 hours. D. 10,100 hours.  A. This is not the correct answer. See correct answer for an explanation. We have been unable to determine how to calculate this incorrect answer choice. If you have calculated it, please let us know how you did it so we can create a full explanation of why this answer choice is incorrect. Please send us an Part 1 : 10/05/14 22:20:20 (c) HOCK international, page 1

Transcript of Hock_Performance Mgmt

Page 1: Hock_Performance Mgmt

8/16/2019 Hock_Performance Mgmt

http://slidepdf.com/reader/full/hockperformance-mgmt 1/152

Question 1 - ICMA 10.P1.142 - Performance Measures

 A company is concerned that its divisional managers are not making decisions that are in the best interests of theoverall corporation. In order to prevent this, the company should use a performance evaluation system that focuses on

 A. residual income.B. operating income.

C. controllable costs.D. flexible budget variances.

A.

When division managers are evaluated on the basis of residual income, they are more likely to embark uponprojects that will be in the best interests of the overall corporation. Residual income is not a rate, but insteadit is an amount of income. As long as the expected net income before taxes from the project is greater thanthe target monetary return in (calculated as a percentage of invested capital), the project will be in the bestinterests of the oveerall corporation and it will also have positive residual income.

Using residual income overcomes a weakness inherent in using return on investment for manager evaluation.If return on investment is used, a project that would have a higher return than the company's cost of capitaland thus would be good for the company may be rejected by the division manager because its return oninvestment is lower than the division's current return on investment and thus it would bring the division'sreturn on investment down.

B. A performance evaluation system that focuses on operating income would not necessarily encourage divisionalmanagers to make decisions that are in the best interests of the company.

C. A performance evaluation system that focuses on controllable costs would not necessarily encourage divisionalmanagers to make decisions that are in the best interests of the company.

D. A performance evaluation system that focuses on flexible budget variances would not necessarily encouragedivisional managers to make decisions that are in the best interests of the company.

Question 2 - ICMA 10.P1.105 - Manufacturing Input Variances -- Materials and Labor 

Lee Manufacturing uses a standard cost system with overhead applied based on direct labor hours. Themanufacturing budget for the production of 5,000 units for the month of May included the following information.Direct labor (10,000 hours at $15 per hour) $150,000Variable overhead 30,000Fixed overhead 80,000

During May, 6,000 units were produced and the direct labor efficiency variance was $1,500 unfavorable. Based on thisinformation, the actual number of direct labor hours used in May was

 A. 9,500 hours.B. 12,100 hours.C. 11,900 hours.D. 10,100 hours.

 A.

This is not the correct answer. See correct answer for an explanation.

We have been unable to determine how to calculate this incorrect answer choice. If you have calculated it, please letus know how you did it so we can create a full explanation of why this answer choice is incorrect. Please send us an

Part 1 : 10/05/14 22:20:20

(c) HOCK international, page 1

Page 2: Hock_Performance Mgmt

8/16/2019 Hock_Performance Mgmt

http://slidepdf.com/reader/full/hockperformance-mgmt 2/152

email at [email protected]. Include the full Question ID number and the actual incorrect answer choice --not its letter, because that can change with every study session created. The Question ID number appears in the upperright corner of the ExamSuccess screen. Thank you in advance for helping us to make the HOCK study materialsbetter.

B.

The formula for the quantity or efficiency variance is (AQ – SQ) × SP. The standard amount of direct labor perunit is 2 hours (10,000 hours planned divided by planned production of 5,000 units). The company actuallyproduced 6,000 units. Therefore, the standard quantity of direct labor for the actual production was 6,000 × 2,or 12,000 hours. The standard price was $15 per hour. We know what the variance was, but we do not knowwhat the actual quantity of direct labor used was. We can use the variance formula and solve for AQ.

(AQ – SQ) × SP = Quantity/Direct Labor Efficiency variance(AQ – 12,000) × 15 = 1,500

Solving for AQ:15AQ – 180,000 = 1,50015AQ = 181,500AQ = 12,100

C. This answer results from using the efficiency variance formula with a negative variance amount to calculate theactual quantity of direct labor hours used. The variance is unfavorable, and for a cost, an unfavorable variance is apositive amount; so the variance amount used with the formula should be positive.

D. This is the $1,500 unfavorable variance added to the budgeted $150,000 and the sum divided by $15 per hour. Thevariance is based on the amount of direct labor allowed for the actual output, not on the amount of direct labor allowedfor the budgeted output.

Question 3 - CMA 693 3.11 - Performance Measures

Edith Carolina, president of the Deed Corporation, requires a minimum return on investment of 8% for any project to beundertaken by her company. The company is decentralized, and leaves investment decisions up to the discretion of thedivision managers as long as the 8% return is expected to be realized. Michael Sanders, manager of the CosmeticsDivision, has had a return on investment of 14% for his division for the past 3 years and expects the division to havethe same return in the coming year. Sanders has the opportunity to invest in a new line of cosmetics which is expectedto have a return on investment of 12%.

If the Deed Corporation evaluates managerial performance using return on investment, what will be the preference fortaking on the proposed cosmetics line by Edith Carolina and Michael Sanders?

 A. Carolina will accept / Sanders will rejectB. Carolina will reject / Sanders will rejectC. Carolina will reject / Sanders will acceptD. Carolina will accept / Sanders will accept

A. Since return on investment uses a percentage of return as the measurement, Sanders would not accept thisproposal. That is because the return of this investment is less than the return that Sanders' division hasearned in the past. Therefore, accepting this investment will decrease Sanders' return on investment. On theother hand, Carolina would accept the project because the return of this project is greater than the 8%required by the company.

B. Carolina would accept the project because the return of this project is greater than the 8% required by the company.

C. Carolina would accept the project, because the return of this project is greater than the 8% required by the

Part 1 : 10/05/14 22:20:20

(c) HOCK international, page 2

Page 3: Hock_Performance Mgmt

8/16/2019 Hock_Performance Mgmt

http://slidepdf.com/reader/full/hockperformance-mgmt 3/152

company. Sanders will not accept the project, because accepting this investment would decrease Sanders' return oninvestment.

D. Sanders will not accept the project, because accepting this investment would decrease Sanders' return oninvestment.

Question 4 - CMA 694 3.18 - Performance Measures

The imputed interest rate used in the residual income approach to performance evaluation can best be described asthe

 A. Historical weighted-average cost of capital for the company.B. Target return on investment set by the company's management.C. Average return on investments for the company over the last several years.D. Average lending rate for the year being evaluated.

 A.

The formula for Residual Income is:

  Net income before taxes on project or investment opportunity – Target return in dollars: a % of employed assets or invested capital= Residual Income

The imputed rate of interest that is used is not the historical weighted-average cost of capital for the company.

B.

The formula for Residual Income is:

  Net income before taxes on project or investment opportunity – Target return in dollars: a % of employed assets or invested capital= Residual Income

The imputed rate of interest that is used is the target return on investment that has been set by management.

C.

The formula for Residual Income is:

  Net income before taxes on project or investment opportunity – Target return in dollars: a % of employed assets or invested capital= Residual Income

The imputed rate of interest that is used is not the average return on investments for the company over the last several

years.

D.

The formula for Residual Income is:

  Net income before taxes on project or investment opportunity – Target return in dollars: a % of employed assets or invested capital= Residual Income

Part 1 : 10/05/14 22:20:20

(c) HOCK international, page 3

Page 4: Hock_Performance Mgmt

8/16/2019 Hock_Performance Mgmt

http://slidepdf.com/reader/full/hockperformance-mgmt 4/152

The imputed rate of interest that is used is not the average lending rate for the year being evaluated.

Question 5 - ICMA 10.P1.106 - Manufacturing Input Variances -- Materials and Labor 

 At the beginning of the year, Douglas Company prepared the following monthly budget for direct materials.Units produced and sold 10,000 15,000Direct material $15,000 $22,500

 At the end of the month, the company's records showed that 12,000 units were produced and sold and $20,000 wasspent for direct materials. The variance for direct materials is

 A. $5,000 favorable.B. $2,000 unfavorable.C. $2,000 favorable.D. $5,000 unfavorable.

 A.

The question asks for the "variance" for direct materials. It does not specify price or quantity variance. Not enoughinformation is given to enable us to split the variance into price and quantity variances, anyway. So to answer thisquestion, we just calculate the total flexible budget materials variance. The total flexible budget materials variance isthe difference between the actual cost for the actual production and the flexible budget cost for the actual production.

This answer is not correct because the actual cost is greater than the flexible budget amount, so the variance must beunfavorable. Furthermore, this is the difference between the actual amount spent for direct materials and the budgetedamount at a production volume of 10,000. However, 12,000 units were produced. The variance is the flexible budgetvariance, which is the difference between the actual amount spent and the flexible budget amount for the actualproduction.

B.

We first need to find the budgeted cost per unit produced for the direct materials, and we can use either of theproduction levels given to calculate it because the per unit cost is the same. $15,000 / 10,000 units = $1.50 perunit. $22,500 / 15,000 units = $1.50 per unit.

The question asks for the "variance" for direct materials. It does not specify price or quantity variance. Notenough information is given to enable us to split the variance into price and quantity variances, anyway. Sowe can just calculate the total flexible budget materials variance. The total flexible budget materials varianceis the difference between the actual cost for the actual production, which was $20,000, and the flexible budgetcost for the actual production, which was $1.50 per unit multiplied by the actual production of 12,000 units, or$18,000.

The variance is $20,000 actual − $18,000 budgeted = $2,000. Since the result is positive and it is a cost, this isan unfavorable variance.

C.

The question asks for the "variance" for direct materials. It does not specify price or quantity variance. Not enoughinformation is given to enable us to split the variance into price and quantity variances, anyway. So to answer thisquestion, we just calculate the total flexible budget materials variance. The total flexible budget materials variance isthe difference between the actual cost for the actual production and the flexible budget cost for the actual production.

This answer is not correct because the actual cost is greater than the flexible budget amount, so the variance must beunfavorable.

Part 1 : 10/05/14 22:20:20

(c) HOCK international, page 4

Page 5: Hock_Performance Mgmt

8/16/2019 Hock_Performance Mgmt

http://slidepdf.com/reader/full/hockperformance-mgmt 5/152

D. This is the difference between the actual amount spent for direct materials and the budgeted amount at a productionvolume of 10,000. However, 12,000 units were produced. The variance is the flexible budget variance, which is thedifference between the actual amount spent and the flexible budget amount for the actual production.

Question 6 - CIA 593 IV.16 - Responsibility Centers and Reporting Segments

Which of the following is the most significant disadvantage of a cost-based transfer price?

 A. Requires externally developed information.B. Imposes market effects on company operations.C. May not promote long-term efficiencies.D. Requires internally developed information.

 A. Externally generated information is required for market-based transfer pricing.

B. Market conditions being imposed within the company are a result of a market-based transfer price.

C. When the transfer price is based on the costs of production, the producing department has no incentive to

control costs. Therefore, because there is no need to control costs, inefficiencies may creep into the processover time.

D. Other transfer price methods require internally generated information and the information that is needed for transferpricing decisions is information that is needed for other purposes as well and is probably already produced by thecompany.

Question 7 - ICMA 10.P1.114 - Manufacturing Input Variances -- Overhead

 A company has a fixed overhead volume variance that is $10,000 unfavorable. The most likely cause for this variance

is that

 A. more was produced than planned.B. the production supervisory salaries were greater than planned.C. the production supervisory salaries were less than planned.D. less was produced than planned.

 A. The fixed overhead production-volume variance is a measure of capacity utilization. More produced than plannedwill result in a favorable variance, not an unfavorable variance.

B. The fixed overhead production-volume variance is not a comparison of actual costs to planned costs. It is instead ameasure of capacity utilization.

C. The fixed overhead production-volume variance is not a comparison of actual costs to planned costs. It is instead a

measure of capacity utilization.

D. The fixed overhead production-volume variance is a measure of capacity utilization. Less produced thanplanned will result in an unfavorable variance.

Question 8 - CMA 1294 3.25 - Manufacturing Input Variances -- Materials and Labor 

Part 1 : 10/05/14 22:20:20

(c) HOCK international, page 5

Page 6: Hock_Performance Mgmt

8/16/2019 Hock_Performance Mgmt

http://slidepdf.com/reader/full/hockperformance-mgmt 6/152

 An unfavorable direct labor efficiency variance could be caused by a(n)

 A. Unfavorable materials usage variance.B. Unfavorable fixed overhead volume variance.C. Favorable variable overhead spending variance.D. Unfavorable variable overhead spending variance.

A.

The labor efficiency variance is calculated as follows: (Actual Hours − Standard Hours for Actual Output) ×Standard Rate. An unfavorable direct labor efficiency variance means that more hours were actually spentthan the standard allowed for the actual output. This could happen due to an inefficiency on the part ofemployees, downtime, or poor quality of raw materials that required excessive rework.

The materials efficiency or usage variance is calculated as follows: (Actual Quantity − Standard Quantity forActual Output) × Standard Price. An unfavorable materials usage variance indicates that more materials wereused than allowed for the actual level of output.

These two variances may be interrelated, as working on more materials than they were supposed to causedthe workers to spend more time than they should have according to the standard for actual output.

B. The fixed overhead volume variance is the difference between the budgeted amount of fixed overhead and theamount of fixed overhead applied (standard rate × standard input for the actual level of output). There is no connectionbetween the fixed overhead volume variance and the direct labor efficiency variance.

C. The variable overhead spending variance is calculated as follows: (Actual Application Rate − Standard ApplicationRate) × Actual Quantity. There is no connection between the variable overhead spending variance and the direct laborefficiency variance.

D. The variable overhead spending variance is calculated as follows: (Standard Application Rate − Actual ApplicationRate) × Actual Quantity. There is no connection between the variable overhead spending variance and the direct laborefficiency variance.

Question 9 - CMA 1292 3.19 - Manufacturing Input Variances -- Overhead

Nanjones Company manufactures a line of products distributed nationally through wholesalers. Presented below areplanned manufacturing data for the year and actual data for November of the current year. The company appliesoverhead based on planned machine hours using a predetermined annual rate.

Planning Data

  Annual November Fixed manufacturing overhead $1,200,000 $100,000Variable manufacturing overhead 2,400,000 220,000Direct labor hours 48,000 4,000Machine hours 240,000 22,000

Data for November  Direct labor hours (actual) 4,200Direct labor hours (plan based on output) 4,000Machine hours (actual) 21,600Machine hours (plan based on output) 21,000Fixed manufacturing overhead $101,200Variable manufacturing overhead $214,000

The fixed overhead volume variance for November was

Part 1 : 10/05/14 22:20:20

(c) HOCK international, page 6

Page 7: Hock_Performance Mgmt

8/16/2019 Hock_Performance Mgmt

http://slidepdf.com/reader/full/hockperformance-mgmt 7/152

 A. $1,200 unfavorable.B. $10,000 favorable.C. $5,000 favorable.D. $5,000 unfavorable.

 A.

This is the fixed overhead spending variance, not the fixed overhead production-volume variance.

The fixed overhead spending variance is the difference between actual fixed overhead incurred for the month (given as$101,200) and budgeted fixed overhead for the month (given as $100,000). Since it is positive, it is unfavorable.

The fixed overhead production-volume variance is the difference between the budgeted amount of fixed overhead andthe amount of fixed overhead applied (standard rate × standard input for the actual level of output). We already saidthat the budgeted fixed overhead is given as $100,000. The amount applied must be calculated from the informationgiven. The amount of fixed overhead applied is the application rate per machine hour multiplied by the number ofmachine hours allowed for the actual output. The application rate per machine hour is the annual budgeted fixedoverhead divided by the annual number of machine hours planned to be used.

B. The fixed overhead production-volume variance is the difference between the budgeted amount of fixed overheadand the amount of fixed overhead applied (standard rate × standard input for the actual level of output). If in the

calculation of fixed manufacturing overhead applied the static budget amount of machine hours (22,000) were used,we would come up with an amount of $110,000 FOH applied. And the fixed overhead volume variance would be$10,000 ($110,000 − $100,000) favorable; which is incorrect. In the calculation of fixed overhead applied, we shoulduse the number of standard machine hours for the actual level of output (21,000 hours). See the correct answer for acomplete explanation and information on interpreting this variance.

C.

The fixed overhead production-volume variance is the difference between the budgeted amount of fixedoverhead and the amount of fixed overhead applied (standard rate × standard input for the actual level ofoutput). It is Budgeted Fixed Overhead minus Applied Fixed Overhead. This is the only time for an expensevariance calculation that a budgeted cost amount comes before an actual cost amount, and yet a negativeamount (actual is higher than budget) is a favorable variance.

Nanjones applies overhead based on planned machine hours using a predetermined annual rate. The amountof the planned fixed manufacturing overhead was $1,200,000 and the amount of planned machine hours were240,000. Thus, the standard application rate for fixed manufacturing overhead was $5 per machine hour($1,200,000 ÷ 240,000). The number of machine hours allowed for the actual level of output in November was21,000. Now we can calculate the amount of applied fixed manufacturing overheads as $105,000 ($5 × 21,000).The budgeted amount of fixed manufacturing overhead is the static budget amount planned for November of$100,000. Therefore, the fixed overhead volume variance is: $100,000 − $105,000 = ($5,000) favorable.

The fixed overhead production-volume variance is caused by the actual production level being different fromthe production level used to calculate the budgeted fixed overhead rate. It measures usage of facilities, and soit is not a comparison of actual incurred cost with budgeted cost the way other variances are. Whenproduction facilities are used more than was planned (actual is greater than budget), the variance will befavorable because the company has gotten more use from its capacity than it thought it would. When

production facilities are used less than was planned (actual is less than budget), the variance is unfavorablebecause it means the company had unused capacity. The amount of the variance is a rough measure of thecost of the unused capacity.

When the company has unused capacity, the cost for that unused capacity should not be passed on tocustomers in higher prices. Instead, the company should find other uses for the capacity. Management mayconsider developing a new product to make use of the unused capacity, renting out part of the factory toanother company, taking contract work that other companies are outsourcing, or even selling the unusedfacilities. This variance helps management to see the cost of the unused capacity.

Part 1 : 10/05/14 22:20:20

(c) HOCK international, page 7

Page 8: Hock_Performance Mgmt

8/16/2019 Hock_Performance Mgmt

http://slidepdf.com/reader/full/hockperformance-mgmt 8/152

Fortunately for Nanjones, it is not in that situation. Its production-volume variance is favorable.

D.

This answer is the amount of applied fixed overhead minus the budgeted fixed overhead. The fixed overheadproduction-volume variance is the budgeted amount of fixed overhead minus the amount of fixed overhead applied(standard rate × standard input for the actual level of output). This is the only time for an expense variance calculation

that a budgeted cost amount comes before an actual cost amount, and yet a negative amount is a favorable variance. A negative amount, meaning the amount of fixed overhead applied was greater than the budgeted amount, is favorablebecause it means the facilities were used more than was planned, which is good.

Question 10 - CMA 1291 3.4 - Manufacturing Input Variances -- Materials and Labor 

 Arrow Industries employs a standard cost system in which direct materials inventory is carried at standard cost. Arrowhas established the following standards for the prime costs of one unit of product.

 StandardQuantity

StandardPrice

StandardCost

Direct materials 8 pounds$1.80 per pound $14.40Direct labor 0.25 hours $8.00 per hour 2.00  $16.40

During November, Arrow purchased 160,000 pounds of direct materials at a total cost of $304,000. The total factorywages for November were $42,000, 90% of which were for direct labor. Arrow manufactured 19,000 units of productduring November using 142,500 pounds of direct materials and 5,000 direct labor hours.

The direct labor usage (efficiency) variance for November is

 A. $2,200 favorable.B. $2,000 unfavorable.C. $1,800 unfavorable.D. $2,000 favorable.

 A. The labor efficiency variance is unfavorable as the actual hours used were greater than the hours allowed for theactual production. See the correct answer for a complete explanation.

B. The labor efficiency variance is a quantity variance, calculated as follows: (Actual Hours − Standard Hoursfor Actual Output) × Standard Rate. A total of 5,000 direct labor hours were actually used in production. Weknow that 19,000 units of product were manufactured during the period. The standard direct labor hoursallowed for production of one unit is 0.25 hours. Therefore, the standard direct labor hours allowed for theproduction of the period is 19,000 × 0.25 = 4,750 hours. The standard rate is $8. Therefore, the direct laborefficiency variance is (5,000 − 4,750) × $8 = $2,000 unfavorable. Because the actual hours used were greaterthan the standard hours allowed for the actual output, the variance is positive and unfavorable.

C. The labor efficiency variance is (Actual Hours − Standard Hours for the Actual Output) × Standard Rate. See thecorrect answer for a complete explanation.

D. The labor efficiency variance is unfavorable as the actual hours used were greater than the hours allowed for theactual production. See the correct answer for a complete explanation.

Question 11 - ICMA 10.P1.132 - Performance Measures

David Burke is manager of claims processing for Continental Health Care System. His performance is evaluated using

Part 1 : 10/05/14 22:20:20

(c) HOCK international, page 8

Page 9: Hock_Performance Mgmt

8/16/2019 Hock_Performance Mgmt

http://slidepdf.com/reader/full/hockperformance-mgmt 9/152

various measures agreed upon in advance with Diane Lewis, general manager. Lewis asked Burke to recommendseveral measures to evaluate the performance of his unit next year. Which one of the following performancemeasures would likely have the least positive effect on Burke’s motivation and performance?

 A. Average processing time per claim.B. Processing cost per claim.C. Total dollar amount of claims processed per month.

D. Percentage of claims processed accurately the first time.

 A. Average processing time per claim is something a manager can control through training. Measuring Burke'sperformance in this area would have the positive effect of encouraging him to keep processing time as low as possible.

B. Processing cost per claim is something a manager can control. Measuring Burke's performance in this area wouldhave the positive effect of encouraging him to keep costs as low as possible.

C.

Total dollar amount of claims processed per month is not an accurate way of measuring speed of processing,because some claims will be for high amounts and others will be for lower amounts. That is not something themanager of claims processing can control, so his performance should not be measured on it.

Furthermore, evaluating his performance on that could lead to his giving priority to processing the largerclaims. That would lead to larger claims being approved before smaller claims, and larger sums of cash wouldbe paid out earlier to settle those larger claims. More cash paid out earlier would increase cash outflow,resulting in lower cash balances and higher cost of capital. Giving priority in processing would also result inpoor customer service to the filers of the smaller claims.

D. Percentage of claims processed accurately the first time is something a manager can control through training.Measuring Burke's performance in this area would have the positive effect of encouraging him to improve the accuracyof his staff.

Question 12 - CMA 1296 3.24 - Responsibility Centers and Reporting Segments

The inventory control supervisor at Wilson Manufacturing Corporation reported that a large quantity of a partpurchased for a special order that was never completed remains in stock. The order was not completed because thecustomer defaulted on the order. The part is not used in any of Wilson's regular products. After consulting withWilson's engineers, the vice president of production approved the substitution of the purchased part for a regular partin a new product. Wilson's engineers indicated that the purchased part could be substituted providing it was modified.The units manufactured using the substituted part required additional direct labor hours resulting in an unfavorabledirect labor efficiency variance in the Production Department. The unfavorable direct labor efficiency variance resultingfrom the substitution of the purchased part in inventory would best be assigned to the

 A. Production manager.B. Sales manager.C. Inventory supervisor.

D. Vice president of production.

 A. The production manager was not involved in the decision making about the substitution of the regular materials forthe special order materials.

B. The sales manager was not involved in production and decision making about the substitution of the regularmaterials for the special order materials.

C. The inventory supervisor was not involved in production and decision making about the substitution of the regularmaterials for the special order materials.

Part 1 : 10/05/14 22:20:20

(c) HOCK international, page 9

Page 10: Hock_Performance Mgmt

8/16/2019 Hock_Performance Mgmt

http://slidepdf.com/reader/full/hockperformance-mgmt 10/152

D. The vice president of production made the decision about the substitution of the regular materials for thespecial order materials. Thus, the unfavorable direct labor efficiency variance resulting from the substitutionof the purchased part in inventory would best be assigned to the vice president of production.

Question 13 - CMA 694 3.22 - Manufacturing Input Variances -- Materials and Labor 

Under a standard cost system, labor price variances are usually not attributable to

 A. The use of a single average standard rate.B. Union contracts approved before the budgeting cycle.C. Labor rate predictions.D. The assignment of different skill levels of workers than planned.

 A. If the standard labor rate was set using a single average standard rate, it may cause a labor rate variance. If the ratedoesn't reflect the proportion of hours worked of each wage rate group of workers, a variance will result.

B. The labor price or rate variance is calculated as: (Actual Rate − Standard Rate) × Actual Hours. The only

figure union contracts can influence is the actual labor rate. The standard rate is set by the budget at thebeginning of the year. As union contracts are approved before the budgeting cycle begins, the informationabout potential changes in wages and salaries is already included in the budget and standards. Therefore, aunion contract approved before the budgeting cycle cannot be the cause of a labor rate variance.

C. As with all future predictions, labor rate predictions can have inaccuracies. If the standard labor rate was set usingthese predicted numbers, it may cause a labor rate variance.

D. The assignment of different skill levels of workers than was planned in most cases will cause a labor pricevariances. The labor rate variance is calculated as: (Actual Rate − Standard Rate) × Actual Hours. When there is adifference between the rates of assigned (with lower or higher working skills) and planned workers, a labor ratevariance will most likely occur.

Question 14 - ICMA 10.P1.129 - Responsibility Centers and Reporting Segments

Kern Manufacturing has several divisions and evaluates performance using segment income. Since sales includetransfers to other divisions, Kern has established a price for internal sales as cost plus 10%. Red Division hasrequested 10,000 units of Green Division’s product. Green Division is selling its product externally at a 60% markupover cost. The corporate policy will encourage the Green Division to

 A. reject the sale to the Red Division because it does not provide the same markup as external sales.B. accept the sale to the Red Division if it is operating at full capacity and the sale will contribute to fixed costs.C. transfer the product to the Red Division if it does not require the Green Division to give up any external sales.D. transfer the product to the Red Division because all costs are being covered and the division will earn a 10% profit.

 A. The problem does not say whether the Green Division is operating at capacity or whether it has excess capacityadequate to produce the internal order. This answer choice would be correct only if Green Division is operating atcapacity and thus would have an opportunity cost to accept the internal order. Since we don't know the status of GreenDivision's capacity, we cannot state that the corporate policy would encourage this action from Green.

B. If Green Division is operating at full capacity, it will have an opportunity cost for lost sales as another cost that wouldneed to be covered by the internal order. The opportunity cost would be the contribution margin that the lost saleswould have provided. Green Division would not accept the sale to Red Division because doing so would cause GreenDivision's profits to decrease.

Part 1 : 10/05/14 22:20:20

(c) HOCK international, page 10

Page 11: Hock_Performance Mgmt

8/16/2019 Hock_Performance Mgmt

http://slidepdf.com/reader/full/hockperformance-mgmt 11/152

C. If the Green Division does not have to give up any external sales in order to produce the order for RedDivision, it will sell the product to the Red Division at the company's internal sales price of cost plus 10%,even though the markup on the sale will not be as great as that of a sale to an external customer would be.Since the Green Division does not have an external customer to use the excess capacity, the internal sale willadd to Green Division's operating income. Green Division's operating income will be higher under thecircumstances if it accepts the internal sale at the lower profit margin than if it turns it down.

D.

The problem does not say whether the Green Division is operating at capacity or whether it has excess capacityadequate to produce the internal order. This answer choice would be correct only if Green Division has excesscapacity. If Green Division does not have excess capacity, we need to include the opportunity cost of lost sales asanother cost that needs to be covered by the internal order. Since we don't know the status of Green Division'scapacity, we cannot state that the corporate policy would encourage this action from Green.

Question 15 - CMA 691 3.28 - Responsibility Centers and Reporting Segments

The basic purpose of a responsibility accounting system is

 A. Authority.B. Budgeting.C. Motivation.D. Variance analysis.

 A. Authority is an element of responsibility accounting. However, it is not the main purpose.

B. Budgeting is an element of responsibility accounting. However, it is not the main purpose.

C. Motivation is the basic purpose of a responsibility accounting. According to responsibility accounting,managers are responsible for those factors that they can control. Their performance is evaluated on how wellthey manage the areas over which they exercise influence, whether they are costs, revenues or both.

D. Variance analysis is an element of responsibility accounting. However, it is not the main purpose.

Question 16 - CIA 594 III.71 - Responsibility Centers and Reporting Segments

Which of the following is not true of responsibility accounting?

 A. The focus of cost center managers will normally be more narrow than that of profit center managers.B. Managers should only be held accountable for factors over which they have significant influence.C. When a responsibility accounting system exists, operations of the business are organized into separate areascontrolled by individual managers.

D. Every factor that affects a firm's financial performance ultimately is controllable by someone, even if that someone isthe person at the top of the firm.

 A. A cost center has only costs under its control while a profit center has both costs and revenues under its control.Therefore, it is true that the focus of a cost center manager will be more narrow that the focus of a profit centermanager.

B. In responsibility accounting, managers should be held accountable only for the things that they are able to control.Therefore, this statement is true.

Part 1 : 10/05/14 22:20:20

(c) HOCK international, page 11

Page 12: Hock_Performance Mgmt

8/16/2019 Hock_Performance Mgmt

http://slidepdf.com/reader/full/hockperformance-mgmt 12/152

C. In order for a responsibility system to work, the company needs to be organized into areas or divisions or someother breakdown based upon what a position is able to influence and control. Therefore, this statement is true.

D. Unfortunately, not everything that impacts a firm's performance is controllable by someone in the firm. Agood example of this type of exogenous variable is the economy. As the economy itself improves or weakens,the company will also be affected. But no person in the company is in a position to influence the economy.Therefore, this statement is not true.

Question 17 - CMA 697 3.26 - Performance Measures

Fabro Inc. produced 1,500 units of Product RX-6 last week. The inputs to the production process for Product RX-6were as follows:

450 pounds of Material A at a cost of $1.50 per pound

300 pounds of Material Z at a cost of $2.75 per pound

300 labor hours at a cost of $15.00 per hour 

What is the best productivity measure for the first-line supervisor in Fabro Inc.'s production plant?

 A. $15.00 per labor hour.B. $2.00 per pound.C. 0.33 units per dollar input.D. 5.00 units per labor hour.

 A. This is the labor rate. However, the first-line supervisor does not control the labor rates and his or her performancecannot be measured on this basis.

B. This is the average price for the pound of material used in production Product RX-6. However, the first-linesupervisor does not control the price and his or her performance cannot be measured on this basis.

C. This is the units per labor dollar. However, the first-line supervisor does not control the labor rates and his or herperformance cannot be measured on this basis.

D. The first-line supervisor is usually responsible for the productivity in terms of units produced during theperiod. He or she can not control the price of inputs (materials, labor). Thus, the best productivity measure forthe first-line supervisor is the units produced per labor hour, which is 5.00 units per labor hour (1,500 ÷ 300).

Question 18 - CMA 694 3.19 - Introduction to Variance Analysis and Standard Costs

Which one of the following is least likely to be involved in establishing standard costs for evaluation purposes?

 A. Budgetary accountants.

B. Industrial engineers.C. Top management.D. Quality control personnel.

 A. Budgetary accountants are involved in establishing standard costs for evaluation purposes.

B. Industrial engineers are involved in establishing standard costs for evaluation purposes.

C. Top management is primarily involved in formulating strategy plans and budgets. Establishing standardcosts for evaluation purposes is a task of management of a lower level. These standards are used to estimate

Part 1 : 10/05/14 22:20:20

(c) HOCK international, page 12

Page 13: Hock_Performance Mgmt

8/16/2019 Hock_Performance Mgmt

http://slidepdf.com/reader/full/hockperformance-mgmt 13/152

what costs should be under normal conditions of operations. Industrial engineers, budgetary accountants,quality control personnel and employees who will be evaluated using these criteria are involved in theprocess.

D. Quality control personnel are involved in establishing standard costs for evaluation purposes.

Question 19 - CMA 695 3.24 - Manufacturing Input Variances -- Materials and Labor 

Blaster Inc., a manufacturer of portable radios, purchases the components from subcontractors to use to assembleinto a complete radio. Each radio requires three units each of Part XBEZ52, which has a standard cost of $1.45 perunit. During May, Blaster experienced the following with respect to Part XBEZ52.  UnitsPurchases ($18,000) 12,000Consumed in manufacturing 10,000Radios manufactured 3,000

During May, Blaster Inc. incurred a materials efficiency variance of 

 A. $1,450 unfavorable.B. $4,350 favorable.C. $4,350 unfavorable.D. $1,450 favorable.

A. The efficiency variance is calculated as (AQ − SQ) × SP. The actual quantity is 10,000. The standard quantityis 9,000 (3 units of part XBEZ52 to produce one radio multiplied by 3,000 radios that were manufactured). Thestandard price is $1.45 per part XBEZ52. Therefore, the materials efficiency variance is (10,000 − 9,000) × $1.45= $1,450 unfavorable.

B. The materials efficiency variance is unfavorable because the amount of XBEZ52 actually consumed inmanufacturing was greater than the standard quantity. This answer is also incorrect because the quantity of unitspurchased was used in the formula instead of the quantity consumed in manufacturing.

C.

The materials efficiency variance is calculated as (AQ − SQ) × SP. This answer results from using the quantity of unitspurchased in the formula instead of the quantity consumed in manufacturing.

D. The materials efficiency variance is unfavorable because the amount of XBEZ52 actually consumed inmanufacturing was greater than the standard quantity.

Question 20 - CMA 1293 3.25 - Variance Analysis Concepts

 A manufacturing firm planned to manufacture and sell 100,000 units of product during the year at a variable cost perunit of $4.00 and a fixed cost per unit of $2.00. The firm fell short of its goal and only manufactured 80,000 units at atotal incurred cost of $515,000. The firm's manufacturing cost variance was:

 A. $5,000 unfavorable.B. $5,000 favorable.C. $35,000 unfavorable.D. $85,000 favorable.

 A. Incurred total manufacturing costs were less than total budgeted manufacturing costs in the flexible budget, so the

Part 1 : 10/05/14 22:20:20

(c) HOCK international, page 13

Page 14: Hock_Performance Mgmt

8/16/2019 Hock_Performance Mgmt

http://slidepdf.com/reader/full/hockperformance-mgmt 14/152

variance is favorable.

B.

The total manufacturing cost variance is the difference between all actual incurred manufacturing costs(direct materials, direct labor, variable overhead and fixed overhead) and the direct materials, direct labor,variable overhead and fixed overhead manufacturing budgeted costs in the flexible budget. To answer this

question, we need to first calculate what the total budgeted manufacturing cost is in the flexible budget,including all variable manufacturing costs and all fixed manufacturing costs. Then, we need to compare thetotal actual costs incurred ($515,000) with that total budgeted amount.

The budgeted variable cost per unit was $4, and that included direct materials, direct labor, and variableoverhead. Therefore, the flexible budget amount for total variable costs (including direct materials, directlabor and variable overhead) would have been $4 multiplied by the 80,000 units actually produced, or$320,000.

Budgeted fixed overhead is exactly the same in the flexible budget as it is in the static budget, because totalfixed overhead costs do not change with changes in the production level the way variable costs do. Therefore,the budgeted fixed overhead in the flexible budget will be the budgeted $2 per unit multiplied by the 100,000units planned. When we do that, we get $200,000. By doing that, we are "backing into" the total fixed overhead

amount that management expected when it made up the budget. Taking that total and dividing it by theexpected production level of 100,000 does not make it a variable cost. Fixed costs do not change withchanges in the production level, so the fact that fewer units were manufactured than planned will not causefixed costs in the flexible budget to change.

So we have total budgeted manufacturing costs in the flexible budget of $320,000 + $200,000, which is$520,000. Actual incurred manufacturing costs totaled $515,000. $515,000 − $520,000 = $(5,000) favorablevariance.

C. This answer results from treating the fixed manufacturing costs as variable costs in calculating the total budgetedcosts for comparison with the actual costs. Fixed costs do not change with the level of activity, as long as the activityremains within the relevant range. Therefore, the budgeted fixed costs to use in calculating the total cost varianceshould be no different from what they were when the plan was originally developed. When the plan was originallydeveloped, the total budgeted fixed costs were divided by the planned production level to determine a fixed

manufacturing cost per unit. To find what the original fixed cost budgeted amount was, we need to reverse the processby multiplying the $2.00 per unit cost by the planned production level of 100,000. That will be the total budgeted fixedcost we need to use in calculating total budgeted costs for calculating the total cost variance.

D. This amount is based on the assumption of 100,000 units of output. See the correct answer for a completeexplanation.

Question 21 - CMA 1287 4.30 - Manufacturing Input Variances -- Materials and Labor 

Todco planned to produce 3,000 units of its single product, Teragram, during November. The standard specificationsfor one unit of Teragram include six pounds of materials at $.30 per pound. Actual production in November was 3,100units of Teragram. The accountant computed a favorable materials purchase price variance of $380 and anunfavorable materials quantity variance of $120. Based on these variances, one could conclude that

 A. More materials were purchased than were used.B. More materials were used than were purchased.C. The actual cost of materials was less than the standard cost.D. The actual usage of materials was less than the standard allowed.

 A. A favorable materials purchase price variance and an unfavorable materials quantity variance do not relate to thequantity of materials purchased and used in production.

Part 1 : 10/05/14 22:20:20

(c) HOCK international, page 14

Page 15: Hock_Performance Mgmt

8/16/2019 Hock_Performance Mgmt

http://slidepdf.com/reader/full/hockperformance-mgmt 15/152

B. A favorable materials purchase price variance and an unfavorable materials quantity variance do not relate to thequantity of materials purchased and used in production.

C. A favorable materials purchase price variance means the actual purchase price was less than wasbudgeted (the standard).

D. An unfavorable materials quantity variance means that more materials were used in production than budgeted(standard), not less.

Question 22 - CMA 1290 3.5 - Manufacturing Input Variances -- Materials and Labor 

Franklin Glass Works' production budget for the year ended November 30 was based on 200,000 units. Each unitrequires two standard hours of labor for completion. Total overhead was budgeted at $900,000 for the year, and thefixed overhead rate was estimated to be $3.00 per unit. Both fixed and variable overhead are assigned to the producton the basis of direct labor hours. The actual data for the year ended November 30 are presented as follows. Actual production in units 198,000 Actual direct labor hours 440,000

 Actual variable overhead $352,000 Actual fixed overhead $575,000

The standard hours allowed for actual production for the year ended November 30 total:

 A. 247,500.B. 400,000.C. 396,000.D. 495,000.

 A.

This is not the correct answer. See the correct answer for a complete explanation.

We have been unable to determine how to calculate this incorrect answer choice. If you have calculated it, please letus know how you did it so we can create a full explanation of why this answer choice is incorrect. Please send us anemail at [email protected]. Include the full Question ID number and the actual incorrect answer choice --not its letter, because that can change with every study session created. The Question ID number appears in the upperright corner of the ExamSuccess screen. Thank you in advance for helping us to make your HOCK study materialsbetter.

B. This is the budgeted number of units multiplied by two standard hours of labor allowed per unit. This is the standardnumber of hours allowed for the budgeted production. The question asks for the standard number of hours allowed forthe actual production.

C. Each unit requires two standard hours of labor for completion. The actual production in units is 198,000.Thus, the standard hours allowed for actual production for the year ended November 30 was 396,000 hours

(198,000 units actually produced × 2 hours allowed per unit).

D.

This is not the correct answer. See the correct answer for a complete explanation.We have been unable to determinehow to calculate this incorrect answer choice. If you have calculated it, please let us know how you did it so we cancreate a full explanation of why this answer choice is incorrect. Please send us an email [email protected]. Include the full Question ID number and the actual incorrect answer choice -- not itsletter, because that can change with every study session created. The Question ID number appears in the upper rightcorner of the ExamSuccess screen. Thank you in advance for helping us to make your HOCK study materials better.

Part 1 : 10/05/14 22:20:20

(c) HOCK international, page 15

Page 16: Hock_Performance Mgmt

8/16/2019 Hock_Performance Mgmt

http://slidepdf.com/reader/full/hockperformance-mgmt 16/152

Question 23 - ICMA 10.P1.009 - Introduction to Variance Analysis and Standard Costs

When compared with ideal standards, practical standards:

 A. Result in a less desirable basis for the development of budgets.B. Produce lower per-unit product costs.C. Incorporate very generous allowances for spoilage and worker inefficiencies.D. Serve as a better motivating target for manufacturing personnel.

 A. Practical standards are a more desirable basis as rarely does any company work under ideal situations. To budgetat those levels would be to set the company up for multiple budget variances and unmotivated employees.

B. A practical standard includes allowances for downtime and idle time. Therefore it would actually increase theper-unit costs somewhat.

C. Practical standards do allow for downtime or idle time, but those allowances do not include incompetence orirresponsible use of resources.

D. Practical standards are excellent measures for employees to keep themselves in check. Knowing howmuch to use or how much to pay gives employees realistic guidelines that can be applied to everyday workingconditions. They are reasonable, not impossible.

Question 24 - ICMA 10.P1.104 - Manufacturing Input Variances -- Materials and Labor 

 A company isolates its raw material price variance in order to provide the earliest possible information to the managerresponsible for the variance. The budgeted amount of material usage for the year was computed as follows:

150,000 units of finished goods × 3 pounds/unit × $2.00/pound = $900,000.

 Actual results for the year were the following:Finished goods produced 160,000 unitsRaw materials purchased500,000 poundsRaw materials used 490,000 poundsCost per pound $2.02

The raw material price variance for the year was

 A. $9,800 unfavorable.B. $10,000 unfavorable.C. $9,600 unfavorable.D. $20,000 unfavorable.

 A.

The question says "A company isolates its raw material price variance in order to provide the earliest possibleinformation to the manager responsible for the variance." This is the definition of the purchase price variance. Thequestion is worded this way to find out whether you know what the purchase price variance is and what it is used for.

This variance was calculated using the raw materials used instead of the raw materials purchased.

B.

Part 1 : 10/05/14 22:20:20

(c) HOCK international, page 16

Page 17: Hock_Performance Mgmt

8/16/2019 Hock_Performance Mgmt

http://slidepdf.com/reader/full/hockperformance-mgmt 17/152

The question says "A company isolates its raw material price variance in order to provide the earliest possibleinformation to the manager responsible for the variance." This is the definition of the purchase price variance.The question is worded this way to find out whether you know what the purchase price variance is and what itis used for.

The formula for the price variance is (AP – SP) × AQ. Because this is asking for the purchase price variance,we use the actual quantity purchased, not the actual quantity used for AQ.

The actual price is given in the question as $2.02 per pound of raw material.

The standard price is given in the question as $2.00 per pound of raw material.

The actual quantity purchased is given as 500,000 pounds.

The purchase price variance = ($2.02 – $2.00) × 500,000 = $10,000 unfavorable.

C.

The formula for the price variance is (AP – SP) × AQ.

This variance was calculated using the raw materials standard quantity instead of the actual quantity. The questionsays "A company isolates its raw material price variance in order to provide the earliest possible information to the

manager responsible for the variance." This is the definition of the purchase price variance. The question is wordedthis way to find out whether you know what the purchase price variance is and what it is used for.

Therefore, the "actual quantity" used in the formula should be the raw materials purchased.

D.

This is the direct material quantity variance. The question asks for the direct material purchase price variance.

Question 25 - CMA 1295 3.25 - Manufacturing Input Variances -- Materials and Labor 

Which one of the following variances is most controllable by the production control supervisor?

 A. Fixed overhead budget variance.B. Material price variance.C. Variable overhead spending variance.D. Material usage variance.

 A. The fixed overhead budget variance cannot be controlled by the production control supervisor.

B. The purchasing department is the most responsible for the material price variance, not the production controlsupervisor.

C. The variable overhead spending variance is related to the difference between the actual variable overhead cost perunit (this is calculated as the actual overhead costs divided by the actual usage of the allocation base) and the

standard application rate. It is the difference between the actual amount of variable overhead incurred and the standardamount of variable overhead allowed for the actual quantity of the variable overhead allocation base used for the actualoutput produced. The production control supervisor does not control this.

D. The material usage variance is the difference between the actual material usage and the standard usage forthis level of output, multiplied by the standard material price. This variance occurs during the productionprocess and is therefore most controllable by the production control supervisor. There are a number ofreasons that could cause this variance: poor production employees' performance, product design, waste,theft, and poor material quality, etc.

Part 1 : 10/05/14 22:20:20

(c) HOCK international, page 17

Page 18: Hock_Performance Mgmt

8/16/2019 Hock_Performance Mgmt

http://slidepdf.com/reader/full/hockperformance-mgmt 18/152

Question 26 - CMA 692 3.16 - Manufacturing Input Variances -- Overhead

 An organization that specializes in reviewing and editing technical magazine articles sets the following standards forevaluating the performance of the professional staff:

 Annual budgeted fixed costs for normal capacity level of 10,000 articles reviewed and edited: $600,000Standard professional hours per 10 articles: 200 hours

Flexible budget of standard labor costs to process 10,000 articles: $10,000,000

The following data apply to the 9,500 articles that were actually reviewed and edited during the current year:

Total hours used by professional staff: 192,000 hours

Flexible costs: $9,120,000

Total cost: $9,738,000

The fixed cost spending variance for the year is:

 A. $48,000 unfavorable.

B. $18,000 favorable.C. $18,000 unfavorable.D. $30,000 favorable.

 A. The fixed overhead spending variance is the actual fixed overhead incurred minus the budgeted fixed overheadcosts. This incorrect result is the consequence of the wrong assumption that the budgeted amount of fixed overheadshould be adjusted to the level of production the way variable costs are in the flexible budget. In the calculation, wehave to use $600,000 of budgeted fixed costs, not $570,000 adjusted for the level of production of 9,500 articles. Again, fixed costs do not vary with the level of output.

B. The variance is unfavorable. This answer results from subtracting actual fixed overhead incurred from budgetedfixed overhead instead of the reverse. See the correct answer for a complete explanation.

C. The fixed overhead spending variance is the actual fixed overhead incurred minus the budgeted fixed

overhead costs. Actual fixed overhead costs are $618,000 ($9,738,000 total cost incurred − $9,120,000 flexible(variable) costs). The budgeted fixed overhead costs are $600,000. The fixed overhead spending variance is$18,000 unfavorable ($618,000 actual fixed costs − $600,000 budgeted fixed costs). The actual costs exceedthe budgeted costs, so the variance is unfavorable.

D. The variance is unfavorable. See the correct answer for a complete explanation.

Question 27 - CMA 684 4.9 - Performance Measures

Return on investment (ROI) is a term often used to express income earned on capital invested in a business unit. Acompany's ROI is increased if:

 A. Sales remain the same and expenses are reduced by the same dollar amount that total assets increase.B. Sales increase by the same dollar amount as expenses and total assets.C. Sales decrease by the same dollar amount that expenses increase.D. Net profit margin on sales increases by the same percentage as total assets.

A.

ROI is calculated as net income divided by total assets. One way to solve this problem is to simply set upsome actual numbers for a basic ROI. For example, sales = $500,000, expenses = $400,000, net income =

Part 1 : 10/05/14 22:20:20

(c) HOCK international, page 18

Page 19: Hock_Performance Mgmt

8/16/2019 Hock_Performance Mgmt

http://slidepdf.com/reader/full/hockperformance-mgmt 19/152

$100,000 and total assets = $400,000. ROI = $100,000 ÷ $400,000, or 0.25.

If we reduce expenses by $50,000 and increase total assets by $50,000, net income will increase by $50,000 to$150,000 because sales remained the same while expenses were reduced. Total assets will increase by$50,000 to $450,000. ROI will change to $150,000 ÷ $450,000, which is 0.33, an increase.

B.

ROI is calculated as net income divided by total assets. One way to solve this problem is to set up some actualnumbers for a basic ROI. For example, sales = $500,000, expenses = $400,000, net income = $100,000 and totalassets = $400,000. ROI = $100,000 ÷ $400,000, or 0.25.

If we increase sales, expenses and total assets by the same amounts, for example by $50,000, the new amounts willbe: sales = $550,000, expenses = $450,000, net income = $100,000 (unchanged because both sales and expensesincreased by the same amount), and total assets = $450,000. ROI will become $100,000 ÷ $450,000, which is 0.22.This is a decrease because the denominator has increased while the numerator has stayed the same. So ROI wouldnot be increased by this.

C.

ROI is calculated as net income divided by total assets. One way to solve this problem is to set up some actual

numbers for a basic ROI. For example, sales = $500,000, expenses = $400,000, net income = $100,000 and totalassets = $400,000. ROI = $100,000 ÷ $400,000, or 0.25.

If we decrease sales and increase expenses by $25,000 each, sales will decrease to $475,000, expenses will increaseto $425,000, and net income will fall to $50,000. ROI will become $50,000 ÷ $400,000 or 0.125, which is lower.

D.

ROI is calculated as net income divided by total assets. One way to solve this problem is to set up some actualnumbers for a basic ROI. For example, sales = $500,000, expenses = $400,000, net income = $100,000 and totalassets = $400,000. ROI = $100,000 ÷ $400,000, or 0.25.

If we increase both the profit margin on sales and the assets of the company by the same percentage, say 10%, ROIwill remain unchanged. Net income will increase by 10% to $110,000 and total assets will increase by 10% to$440,000. ROI will be $110,000 ÷ $440,000, which is unchanged at 0.25.

Question 28 - CIA 582 IV.22 - Manufacturing Input Variances -- Materials and Labor 

Which of the following is least likely to cause an unfavorable materials quantity (usage) variance?

 A. Labor that possesses skills equal to those required by the standards.B. Scheduling of substantial overtime.C. Materials that do not meet specifications.D. Machinery that has not been maintained properly.

A. An unfavorable materials quantity (usage) variance means that more materials were consumed byproduction than was scheduled by the standard. This can happen due to the number of reasons: poor workerperformance, spoilage, shrinkage, theft, design of the product, poor quality of materials, machine downtime,etc. Labor with skills equal to those required by the standards is most likely not a cause of a materialsefficiency variance, because worker performance should be adequate.

B. An unfavorable materials quantity (usage) variance means that more materials were consumed by production thanwas scheduled by the standard. This can happen due to the number of reasons: poor worker performance, spoilage,shrinkage, theft, design of the product, poor quality of materials, machine downtime, etc. Substantial overtime may

Part 1 : 10/05/14 22:20:20

(c) HOCK international, page 19

Page 20: Hock_Performance Mgmt

8/16/2019 Hock_Performance Mgmt

http://slidepdf.com/reader/full/hockperformance-mgmt 20/152

affect the performance of workers and be the cause of a materials efficiency variance.

C. An unfavorable materials quantity (usage) variance means that more materials were consumed by production thanwas scheduled by the standard. This can happen due to the number of reasons: poor worker performance, spoilage,shrinkage, theft, design of the product, poor quality of materials, machine downtime, etc. Thus, materials that do notmeet specifications may be the cause of a materials efficiency variance.

D. An unfavorable materials quantity (usage) variance means that more materials were consumed by production thanwas scheduled by the standard. This can happen due to the number of reasons: poor worker performance, spoilage,shrinkage, theft, design of the product, poor quality of materials, machine downtime, etc. Thus, machinery that has notbeen maintained properly may be the cause of a materials efficiency variance.

Question 29 - ICMA 10.P1.145 - Performance Measures

Which one of the following statements about a balanced scorecard is incorrect?

 A. It seeks to address the problems associated with traditional financial measures used to assess performance.B. It relies on the perception of the users with regard to service provided.

C. It is directly derived from the scientific management theories.D. The notion of value chain analysis plays a major role in the drawing up of a balanced scorecard.

 A. The balanced scorecard is a strategic management tool which developed as a response to problems caused byevaluating managers only on the quarterly or annual financial performance of their business units.

B.

The balanced scorecard encourages managers to focus on elements of long-term success, which are non-financialand operational indicators, instead of only on short-term financial performance. It does this by rewarding managers forimprovements in those elements of long-term success. One of the elements of long-term success is the customer'sperspective with regard to service provided.

Customer satisfaction is a vital part of the customer perspective, because if customers are not satisfied, they will taketheir business elsewhere. Customer satisfaction goals relate to how the company wants its customers to view it, i.e.,the perceptions of its customers.

C.

The balanced scorecard is not derived from scientific management theories. Scientific management was atheory developed in the late 1800s. It focused primarily on the efficiency of individual workers and improvingtheir productivity. Its principles were responsible for the development of "efficiency experts" and the use oftime and motion analysis.

The balanced scorecard is a strategic management tool which developed as a response to problems causedby evaluating managers only on the quarterly or annual financial performance of their business units.

The balanced scorecard encourages managers to focus on elements of long-term success, which arenon-financial and operational indicators, instead of only on short-term financial performance. It does this byrewarding managers for improvements in those elements of long-term success. Improvements in thesenon-financial measures provide the prospect of increased future economic value for shareholders. Whenmanagers are evaluated and rewarded based on these non-financial indicators, it leads to long-term financialperformance improvements. This again comes back to the idea of goal congruence and making sure thateveryone is working toward the same goals.

D.

Part 1 : 10/05/14 22:20:20

(c) HOCK international, page 20

Page 21: Hock_Performance Mgmt

8/16/2019 Hock_Performance Mgmt

http://slidepdf.com/reader/full/hockperformance-mgmt 21/152

Value chain analysis is used to analyze the company's chain of activities for transforming inputs into the outputs thatcustomers value. Value chain analysis can identify what activities increase value to the customers.

The customer perspective is an important part of the non-financial indicators used in balanced scorecard evaluation. Itis one of the four perspectives into which the balanced scorecard measurements used by companies typically fall. (Theothers are financial performance, innovation in internal business processses, and an organizational culture thatsupports employee innovation, growth and development.)

Developing the company’s customer perspective for a balanced scorecard involves measuring its success in themarket segment(s) it wants to serve, such as share of the market, customer satisfaction, and quality goals. Value chainanalysis provides important input to development of customer perspective metrics.

Question 30 - ICMA 10.P1.100 - Variance Analysis Concepts

Marten Company has a cost-benefit policy to investigate any variance that is greater than $1,000 or 10% of budget,whichever is larger. Actual results for the previous month indicate the following.  Budget ActualRaw materials$100,000$89,000

Direct labor 50,000 54,000

The company should investigate

 A. the material variance only.B. the labor variance only.C. both the material variance and the labor variance.D. neither the material variance nor the labor variance.

A.

The company's policy is to investigate any variance that is greater than $1,000 or 10% of budget, whichever islarger. In other words, if 10% of the budget amount is greater than $1,000, they are to investigate any variance

greater than 10%. Or, if $1,000 is greater than 10% of the budget amount, they are to investigate any variancegreater than $1,000.

The raw material budget amount is $100,000, so 10% of that is $10,000. $10,000 is greater than $1,000.Therefore, they are to investigate any variance that is greater than $10,000. Actual raw material was $89,000,which is $11,000 less than the budgeted amount. $11,000 is greater than $10,000, so they are to investigatethat variance.

The direct labor budget amount is $50,000, so 10% of that is $5,000. $5,000 is greater than $1,000. Therefore,they are to investigate any variance that is greater than $5,000. Actual direct labor was $54,000, which is$4,000 greater than the budgeted amount. $4,000 is less than $5,000. Therefore, they do not need toinvestigate that variance.

So they will investigate the raw material variance only.

B.

The company's policy is to investigate any variance that is greater than $1,000 or 10% of budget, whichever is larger.In other words, if 10% of the budget amount is greater than $1,000, they are to investigate any variance greater than10%. Or, if $1,000 is greater than 10% of the budget amount, they are to investigate any variance greater than $1,000.

The raw material budget amount is $100,000, so 10% of that is $10,000. $10,000 is greater than $1,000. Therefore,they are to investigate any variance that is greater than $10,000. The direct labor budget amount is $50,000, so 10% ofthat is $5,000. $5,000 is greater than $1,000. Therefore, they are to investigate any variance that is greater than

Part 1 : 10/05/14 22:20:20

(c) HOCK international, page 21

Page 22: Hock_Performance Mgmt

8/16/2019 Hock_Performance Mgmt

http://slidepdf.com/reader/full/hockperformance-mgmt 22/152

$5,000.

Investigating the labor variance only would not conform to this policy, because the labor variance is only $4,000.

C.

The company's policy is to investigate any variance that is greater than $1,000 or 10% of budget, whichever is larger.

In other words, if 10% of the budget amount is greater than $1,000, they are to investigate any variance greater than10%. Or, if $1,000 is greater than 10% of the budget amount, they are to investigate any variance greater than $1,000.

The raw material budget amount is $100,000, so 10% of that is $10,000. $10,000 is greater than $1,000. Therefore,they are to investigate any variance that is greater than $10,000. The direct labor budget amount is $50,000, so 10% ofthat is $5,000. $5,000 is greater than $1,000. Therefore, they are to investigate any variance that is greater than$5,000.

Investigating both the material variance and the labor variance would not conform to this policy.

D.

The company's policy is to investigate any variance that is greater than $1,000 or 10% of budget, whichever is larger.In other words, if 10% of the budget amount is greater than $1,000, they are to investigate any variance greater than

10%. Or, if $1,000 is greater than 10% of the budget amount, they are to investigate any variance greater than $1,000.

The raw material budget amount is $100,000, so 10% of that is $10,000. $10,000 is greater than $1,000. Therefore,they are to investigate any variance that is greater than $10,000. The direct labor budget amount is $50,000, so 10% ofthat is $5,000. $5,000 is greater than $1,000. Therefore, they are to investigate any variance that is greater than$5,000.

Investigating neither variance would not conform to this policy.

Question 31 - CMA 1295 3.3 - Variance Analysis Concepts

The variance that arises solely because the quantity actually sold differs from the quantity budgeted to be sold is:

 A. Sales volume variance.B. Master budget increment.C. Sales mix variance.D. Static budget variance.

A. The sales volume variance measures the impact of the difference between actual sales volume andbudgeted sales volume. It is the difference between the flexible budget and the static budget.

B. The master budget increment is an increase of budgeted amount of company's master budget.

C. The sales mix variance arises when the actual mix of products sold differs from the budgeted mix, even though the

total quantity of products sold may be the same as budgeted.D. The static budget variance is the difference between budgeted costs and revenues and the actual results.

Question 32 - CMA 1289 4.4 - Manufacturing Input Variances -- Materials and Labor 

 A favorable material price variance coupled with an unfavorable material usage variance would most likely result from:

Part 1 : 10/05/14 22:20:20

(c) HOCK international, page 22

Page 23: Hock_Performance Mgmt

8/16/2019 Hock_Performance Mgmt

http://slidepdf.com/reader/full/hockperformance-mgmt 23/152

 A. The purchase and use of higher than standard quality material.B. Labor efficiency problems.C. The purchase and use of lower than standard quality material.D. Labor mix problems.

 A. A favorable material price variance means that the price of materials consumed by production was lower thanbudgeted. However, an unfavorable material usage variance means that the quantity of materials used in production

was greater than the standard quantity.

B. An unfavorable material usage variance could be caused by labor efficiency problems. However, a favorablematerial price variance does not relate to such problems.

C. A favorable material price variance means that the price of materials consumed by production was lowerthan budgeted. An unfavorable material usage variance means that the quantity of materials used byproduction was greater than the standard quantity. Both variances could be caused by the purchase and useof lower than standard quality material.

D. An unfavorable material usage variance could be caused by labor mix problems. However, a favorable materialprice variance does not relate to such problems.

Question 33 - CMA 1295 H5 - Variance Analysis Concepts

The difference between the actual amounts and the flexible budget amounts for the actual output achieved is the

 A. Flexible budget variance.B. Sales volume variance.C. Production volume variance.D. Standard cost variance.

A. The flexible budget variance is the difference between the actual results and the flexible budget amountbased on the actual level of activity achieved in the budget period.

B. The sales volume variance is the difference between the flexible budget amount and the static budget amount.

C. The production volume variance is the difference between the budgeted fixed overhead and the fixed overheadapplied, based on the standard rate × the standard input of the fixed overhead allocation base allowed for the actuallevel of output.

D. The term "standard cost variance" is not a specific variance calculation. It may be used to refer to variance analysisin general, for example "standard cost variance analysis."

Question 34 - CMA 692 3.17 - Manufacturing Input Variances -- Materials and Labor 

 An organization that specializes in reviewing and editing technical magazine articles sets the following standards forevaluating the performance of the professional staff:

 Annual budgeted fixed costs for normal capacity level of 10,000 articles reviewed and edited: $600,000

Standard professional hours per 10 articles: 200 hours

Flexible budget of standard labor costs to process 10,000 articles: $10,000,000

The following data apply to the 9,500 articles that were actually reviewed and edited during the current year:

Total hours used by professional staff: 192,000 hours

Part 1 : 10/05/14 22:20:20

(c) HOCK international, page 23

Page 24: Hock_Performance Mgmt

8/16/2019 Hock_Performance Mgmt

http://slidepdf.com/reader/full/hockperformance-mgmt 24/152

Flexible costs: $9,120,000

Total cost: 9,738,000

The labor efficiency variance for the year is

 A. $238,000 unfavorable.B. $380,000 favorable.C. $100,000 unfavorable.D. $500,000 favorable.

 A. This is the difference between the flexible budget standard labor costs (variable costs) and total actual costs(including fixed and variable costs), which does not mean anything.

B. This is the actual labor cost of $9,120,000 minus the flexible budget labor cost of $9,500,000 (9,500 articles actuallyreviewed and edited at $1,000 standard cost per article). So this is the total flexible budget variance for labor. Thequestion asks for only the labor efficiency variance, which is one part of the total flexible budget variance.

C. The labor efficiency variance is calculated as follows: (Actual Hours − Standard Hours for Actual Output) ×Standard Rate. Actual hours are given as 192,000. The standard hours for the actual output are 190,000 (20hours per one article × 9,500 articles that were actually reviewed and edited). The standard rate is $1,000 per

article ($10,000,000 ÷ 10,000) and $50 per labor hour ($1,000 ÷ 20). Putting all of this into the formula, the laborefficiency variance is (192,000 − 190,000) × $50 = $100,000 unfavorable.

D.

The labor efficiency variance is calculated as follows: (Actual Hours − Standard Hours for Actual Output) × StandardRate. There are two mistakes in this variance calculation.

One, the standard hours for the actual output was used in place of the actual hours. And two, the number of hours inthe fixed budget was used in place of the standard hours for the actual output. See the correct answer for a completeexplanation.

Question 35 - CIA 579 IV.2 - Variance Analysis Concepts

Which of the following factors should not be considered when deciding whether to investigate a variance?

 A. Likelihood that an investigation will eliminate future occurrences of the variance.B. Whether the variance is favorable or unfavorable.C. Magnitude of the variance.D. Trend of the variances over time.

 A. The investigation of any variance, especially an unfavorable one, should take place in order to eliminate futureoccurrences of the variance. Part of the process should be to plan and take corrective actions. If an investigation wouldnot result in any changes that could eliminate future occurrences of the variance, then the benefit to be gained from theinvestigation would not be worth the cost of the investigation.

B. The behavior of the variance (negative or positive) should not influence the decision of whether toinvestigate or not to investigate the variance. The variance is the measure of how much the actual results varyfrom the budgeted (expected) results. All significant variances should be investigated. It is also critical thatbenefits from variance investigation exceed its costs.

C. The magnitude (amount) of the variance should be considered in determining whether or not to investigate avariance. Management should give more attention to significant variances, measuring how material the variance is. It iscritical that benefits from variance investigation exceed its costs.

Part 1 : 10/05/14 22:20:20

(c) HOCK international, page 24

Page 25: Hock_Performance Mgmt

8/16/2019 Hock_Performance Mgmt

http://slidepdf.com/reader/full/hockperformance-mgmt 25/152

D. The trend of the variances over time should be considered in determining whether or not to investigate a variance. Itis important to investigate variances when there is a strong tendency of unfavorable variances to increase.

Question 36 - ICMA 10.P1.098 - Variance Analysis Concepts

The following information is from the accounting records of St. Charles Enterprises.

 Static

Budget ActualSales volume (units) 82,000 75,000Selling price/unit $15.00 $15.00Variable cost/unit 9.00 9.25Fixed cost $280,000$285,000

 A staff assistant performed a comparison of budget and actual data and calculated an unfavorable operating incomevariance of $65,750. The assistant concluded that performance did not meet expectations because there was anunfavorable variance in operating income. Which one of the following is the best evaluation of this preliminaryconclusion?

 A. Both the conclusion and the variance calculation are incorrect.B. Both the conclusion and the variance calculation are correct.C. The conclusion is incorrect, but the variance calculation is informative.D. The conclusion is correct, but the variance calculation could be more informative.

 A. This is not an accurate evaluation of the preliminary conclusion.

B. This is true, but it is not the best evaluation of the preliminary conclusion.

C. This is not an accurate evaluation of the preliminary conclusion.

D.

This is the best evaluation of the preliminary conclusion. The operating income variance is $65,750unfavorable. However, this conclusion does nothing to explain why the operating income variance wasunfavorable. Was it because sales were lower than expected? Was it because variable cost was higher thanexpected? Was it because fixed cost was higher than expected? Was it all of these? How much of theoperating income variance was attributable to each, and what was the root cause of each? What can be doneabout it?

A variance report such as the following, including flexible budget amounts and flexible budget variancesalong with static budget amounts and variances, should be used to pinpoint the causes of the variance inoperating income and then to investigate further what caused each individual variance.

  Actual Static BudgetStatic Budget

Variance Flexible BudgetFlexible Budget

Variance

Revenue $1,125,000 $1,230,000 $(105,000) $1,125,000 $ -0-Variable Cost 693,750 738,000 (44,250) 675,000 18,750

Contribution Margin $ 431,250 $ 492,000 $ (60,750) $ 450,000 $ (18,750)

Fixed Cost 285,000 280,000 5,000 280,000 5,000

Operating Income $ 146,250 $ 212,000 $ 65,750 $ 170,000 $ (23,750)

Question 37 - CMA 687 4.17 - Manufacturing Input Variances -- Overhead

Part 1 : 10/05/14 22:20:20

(c) HOCK international, page 25

Page 26: Hock_Performance Mgmt

8/16/2019 Hock_Performance Mgmt

http://slidepdf.com/reader/full/hockperformance-mgmt 26/152

Selo Imports uses flexible budgeting for the control of costs. The company's annual master budget includes $324,000for fixed production supervisory salaries at a volume of 180,000 units. Supervisory salaries are expected to be incurreduniformly throughout the year. During the month of September 15,750 units were produced, and production supervisorysalaries incurred were $28,000. A performance report for September would reflect a budget variance of 

 A. $350 favorable.

B. $1,000 unfavorable.C. $1,000 favorable.D. $350 unfavorable.

 A.

This answer results from calculating the fixed overhead spending (budget) variance as if it were the variable overheadspending variance: (Actual Cost per unit of $1.77777771 − Budgeted Cost per unit of $1.802) × Actual Quantity of15,750 = $(350) favorable.

The fixed overhead spending (budget) variance is equal to the difference between the actual and budgeted amounts offixed overhead.

1 Actual cost of per unit of $1.7777777 is calculated as $28,000 ÷ 15,750.

2Budgeted cost per unit of $1.80 is calculated as $324,000 ÷ 180,000 or $27,000 ÷ 15,000.

B. The fixed overhead spending (or budget) variance is equal to the difference between the actual andbudgeted amounts of fixed overhead. The actual fixed overhead incurred was $28,000. The budgeted (staticbudget and flexible budget) amount of fixed overhead for the month was $27,000 ($324,000 ÷ 12). Thus, thefixed overhead spending (budget) variance is $1,000 unfavorable ($28,000 − $27,000).

C. The variance is unfavorable because the actual amount of fixed overhead is greater than the budgeted amount.

D.

This answer results from calculating the fixed overhead spending (budget) variance as if it were the variable overhead

spending variance but reversing actual and budgeted amounts per unit: (Budgeted Cost per unit of $1.80

1

− ActualCost per unit of $1.77777772) × Actual Quantity of 15,750 = $350 unfavorable.

The fixed overhead spending (budget) variance is equal to the difference between the actual and budgeted amounts offixed overhead.

1Budgeted cost per unit of $1.80 is calculated as $324,000 ÷ 180,000 or $27,000 ÷ 15,000.

2 Actual cost per unit of $1.7777777 is calculated as $28,000 ÷ 15,750.

Question 38 - ICMA 10.P1.117 - Manufacturing Input Variances -- Materials and Labor 

Johnson Inc. has established per unit standards for material and labor for its production department based on 900units normal production capacity as shown below.3 lbs. of direct materials @ $4 per lb.$121 direct labor hour @ $15 per hour 15Standard cost per unit $27

During the year 1,000 units were produced. The accounting department has charged the production departmentsupervisor with the following unfavorable variances.Materials Quantity Variance Materials Price Variance

Part 1 : 10/05/14 22:20:20

(c) HOCK international, page 26

Page 27: Hock_Performance Mgmt

8/16/2019 Hock_Performance Mgmt

http://slidepdf.com/reader/full/hockperformance-mgmt 27/152

 Actual usage 3,300 lbs. Actual cost $12,600Standard usage 3,000 lbs. Standard cost 12,000

Unfavorable 300 lbs. Unfavorable $ 600

Bob Sterling, the production supervisor, has received a memorandum from his boss stating that he did not meet theestablished standards for material prices and quantity and corrective action should be taken. Sterling is very unhappyabout the situation and is preparing to reply to the memorandum explaining the reasons for his dissatisfaction. All of

the following are valid reasons for Sterling’s dissatisfaction except that the

 A. variance calculations fail to properly reflect that actual production exceeded normal production capacity.B. material price variance is the responsibility of the purchasing department.C. cause of the unfavorable material usage variance was the acquisition of substandard material.D. standards have not been adjusted to the engineering changes.

A. This is not a valid reason for Sterling's dissatisfaction. The standard usage of 3,000 lbs. is the standardquantity of 3 lbs. of direct materials per unit multiplied by the 1,000 units actually produced. And the standardcost of $12,000 is the standard price of $4 per lb. multiplied by the standard usage of 3 lbs. per unit, and theproduct multiplied by the 1,000 units actually produced. Thus, the standard usage and standard cost used inthe calculation of the variances do reflect the fact that actual production exceeded normal production capacity.

B. This is a true statement. The purchasing department purchases the raw materials, not the production department.

C. If the raw material was, in fact, substandard, that could easily cause an unfavorable material usage variance. Thequality of the raw materials purchased is primarily the responsibility of the purchasing department. It is not theresponsibility of the production department.

D. If engineering changes have been made and the standards have not been changed, that could cause a materialusage variance. The production department is not responsible for setting standards and maintaining them.

Question 39 - CIA 595 III.24 - Manufacturing Input Variances -- Materials and Labor 

Which of the following management practices involves concentrating on areas that deserve attention and placing lessattention on areas operating as expected?

 A. Responsibility accounting.B. Management by exception.C. Management by objectives.D. Benchmarking.

 A. Responsibility accounting is an accounting system that measures accounting results of each responsibility centerseparately; it also measures consolidated results.

B. Management by exception means that management focuses on areas where there are problems, asidentified by the fact that there is a variance from the standard. In order for a company to use management byexception, standards must be set and there must be a system whereby variances are identified and reported

to the appropriate level of the company.

C. The primary area of concentration in MBO is goal congruence; it is behavioral, communication-oriented, and aresponsibility approach system.

D. Benchmarking is the process of a company using the standards set by other companies as a target or model for itsown operations. This is also called best practices. It is the process of continuously trying to emulate (imitate) the bestcompanies in the world.

Part 1 : 10/05/14 22:20:20

(c) HOCK international, page 27

Page 28: Hock_Performance Mgmt

8/16/2019 Hock_Performance Mgmt

http://slidepdf.com/reader/full/hockperformance-mgmt 28/152

Question 40 - CMA 1296 3.23 - Responsibility Centers and Reporting Segments

The purpose of identifying manufacturing variances and assigning their responsibility to a person/department should beto:

 A. Pinpoint fault for operating problems in the organization.B. Determine the proper cost of the products produced so that selling prices can be adjusted accordingly.C. Use the knowledge about the variances to promote learning and continuous improvement in the manufacturingoperations.D. Trace the variances to finished goods so that the inventory can be properly valued at year-end.

 A. Reports of variances from the budget should be used to direct attention to problems, and the emphasis should beon information, not on blame.

B. The selling price is mostly determined by the manufacturing costs and other aspects like market price level, servicecosts allocation, etc. The variances are usually not significant aspects in the process of selling price determination.

C. The purpose of identifying manufacturing variances and assigning their responsibility to aperson/department is to use the knowledge about the variances to promote learning and continuous

improvement. One of the main purposes for responsibility centers and responsibility accounting is to enableevaluation of subunits' performance and contribute to measuring the performance of the subunits' managers.This provides motivation for managers of the subunits. By knowing what they are responsible for andcontrolling those items, managers should be more motivated than if they were evaluated on somethingoutside of their control.

D. Depending on the significance of a variance, it is either written off to the finished goods account or allocated to theinventory accounts and finished goods on pro-rata basis. Variances are not used to properly value inventory at yearend.

Question 41 - ICMA 10.P1.112 - Manufacturing Input Variances -- Overhead

Harper Company's performance report indicated the following information for the past month. Actual total overhead $1,600,000Budgeted fixed overhead 1,500,000 Applied fixed overhead at $3 per labor hour 1,200,000 Applied variable overhead at $0.50 per labor hour 200,000  Actual labor hours 430,000

Harper's total overhead spending variance for the month was

 A. $200,000 unfavorable.B. $100,000 favorable.

C. $185,000 unfavorable.D. $115,000 favorable

 A. This is actual total overhead minus applied total overhead (fixed and variable). It correctly indicates an unfavorablevariance, but the calculation is missing the budgeted element. See correct answer for a full explanation.

B. This is the actual total overhead minus budgeted fixed overhead. It is incorrect for two reasons. First, if thecalculation were correct, the variance would be an unfavorable one, not a favorable one, because the actual washigher than the budgeted amount and this is a cost variance. And second, variable overhead is included in the totalactual overhead amount but is not included in the amount that is subtracted from total actual overhead.

Part 1 : 10/05/14 22:20:20

(c) HOCK international, page 28

Page 29: Hock_Performance Mgmt

8/16/2019 Hock_Performance Mgmt

http://slidepdf.com/reader/full/hockperformance-mgmt 29/152

C. This is actual total overhead minus applied fixed overhead minus (Standard Variable OH Rate × Actual ApplicationBase). The fixed overhead amount subtracted should be the budgeted fixed overhead, not the applied fixed overhead.

D. The total overhead spending variance is the total of the fixed overhead spending variance + the variableoverhead spending variance.

The fixed overhead spending variance is:

Actual Fixed Overhead Incurred Minus Budgeted Fixed Overhead.

The variable  overhead spending variance is:

Actual Variable Overhead Incurred Minus Budgeted Variable Overhead Based on Inputs Actually Used(standard rate × actual usage of application base).

Therefore, the total overhead spending variance (fixed and variable) is calculated as follows:

  Actual Total Overhead Incurred

Minus: Budgeted Fixed Overhead

Minus: Budgeted Variable Overhead (Rate × Actual Application Base)

——————————————————————————————

Equals: Total Overhead Spending Variance

Actual Total Overhead = $1,600,000

Budgeted Fixed Overhead = $1,500,000

Budgeted Variable OH (Rate × Actual Application Base) = $0.50 × 430,000 = $215,000

Therefore, the total overhead spending variance = $1,600,000 − $1,500,000 − $215,000 = $(115,000). Since

overhead is a cost, a negative variance is a favorable variance.

Question 42 - CIA 1192 IV.20 - Manufacturing Input Variances -- Materials and Labor 

 A manufacturer has the following direct materials standard for one of its products.

Direct materials: 3 pounds @ $1.60/pound = $4.80

The company records all inventory at standard cost. Data for the current period regarding the manufacturer's budgetedand actual production for the product as well as direct materials purchases and issues to production for manufacture ofthe product are presented as follows.

Budgeted production for the period: 8,000 units Actual production for the period: 7,500 units

Direct materials purchases:

Pounds purchased: 25,000 pounds

Total cost: $38,750

Direct materials issued in production: 23,000 pounds

Part 1 : 10/05/14 22:20:20

(c) HOCK international, page 29

Page 30: Hock_Performance Mgmt

8/16/2019 Hock_Performance Mgmt

http://slidepdf.com/reader/full/hockperformance-mgmt 30/152

The direct materials purchase price variance for the current period is

 A. $1,250 favorable.B. $1,125 favorable.C. $1,200 favorable.D. $1,150 favorable.

A. The purchase price variance is calculated as follows: (AP − SP) × AQ, where AQ is the actual quantitypurchased. The actual price for the pound of material is $1.55 ($38,750 ÷ 25,000). The standard price is $1.60.The quantity purchased is 25,000. The direct materials purchase price variance is ($1.55 − $1.60) × 25,000 =$(1,250) favorable. The variance is favorable because the actual price the materials were purchased for is lessthan the standard price.

B. This result is calculated using the quantity of materials allowed for the production, 22,500 pounds (3 lb. Per unit offinished product × 7,500 units produced). However, to calculate the purchase price variance, we need to use thepurchased quantity of material (25,000 pounds). See the correct answer for a complete explanation.

C. This result is calculated using the standard quantity of materials, 24,000 pounds (3 lb. per unit of finished productand 8,000 units budgeted for the production). However, to calculate the purchase price variance, we need to use thepurchased quantity of material (25,000 pounds). See the correct answer for a complete explanation.

D. This is the material price variance, not the purchase price variance. To calculate the purchase price variance weneed to use the purchased quantity of material (25,000), not the quantity consumed by production (23,000).

Question 43 - CMA 1293 3.22 - Manufacturing Input Variances -- Materials and Labor 

ChemKing uses a standard costing system in the manufacture of its single product. The 35,000 units of raw material ininventory were purchased for $105,000, and two units of raw material are required to produce one unit of final product.In November, the company produced 12,000 units of product. The standard cost for material allowed for the output was$60,000, and there was an unfavorable quantity variance of $2,500.

ChemKing's standard price for one unit of material is

 A. $2.50.B. $2.00.C. $5.00.D. $3.00.

A. The total standard cost allowed for the actual output is $60,000. Two units of raw material are allowed foreach unit produced, and the company produced 12,000 units. Therefore, the standard quantity allowed for theactual output was 12,000 × 2, or 24,000 units. The standard price for one unit of material was thus $60,000 ÷24,000 units of direct materials, or $2.50 per unit of direct materials.

B.

This is not the correct answer. See the correct answer for a complete explanation.

We have been unable to determine how to calculate this incorrect answer choice. If you have calculated it, please letus know how you did it so we can create a full explanation of why this answer choice is incorrect. Please send us anemail at [email protected]. Include the full Question ID number and the actual incorrect answer choice --not its letter, because that can change with every study session created. The Question ID number appears in the upperright corner of the ExamSuccess screen. Thank you in advance for helping us to make your HOCK study materialsbetter.

C. This is the standard direct materials cost per unit of product. However, the question asks for the standard cost per

Part 1 : 10/05/14 22:20:20

(c) HOCK international, page 30

Page 31: Hock_Performance Mgmt

8/16/2019 Hock_Performance Mgmt

http://slidepdf.com/reader/full/hockperformance-mgmt 31/152

unit of direct material.

D. This is the actual price per unit of raw materials purchased ($105,000 paid ÷ 35,000 units of raw materialspurchased = $3), not the standard price per unit.

Question 44 - CMA 1291 3.8 - Responsibility Centers and Reporting Segments

 A segment of an organization is referred to as a profit center if it has

 A. Authority to make decisions affecting the major determinants of profit including the power to choose its markets andsources of supply.B. Authority to provide specialized support to other units within the organization.C. Authority to make decisions affecting the major determinants of profit including the power to choose its markets andsources of supply and significant control over the amount of invested capital.D. Authority to make decisions over the most significant costs of operations including the power to choose the sourcesof supply.

A. By definition, a profit center is a segment of the organization that has authority to make decisions affecting

the major determinants of profit, i.e., revenues and expenses. This includes the power to choose its marketsand sources of supply.

B. This is refers to support center and it is not classified as a center under responsibility accounting.

C. This is the definition of an investment center.

D. This is the definition of a cost center.

Question 45 - CIA 597 3.18 - Manufacturing Input Variances -- Materials and Labor 

 A company reported a significant material quantity variance for the month of January. All of the following are possibleexplanations for this variance except 

 A. Producing more units than planned for in the master budget.B. Processing a large number of rush orders.C. An inadequately trained and supervised labor force.D. Cutbacks in preventive maintenance.

A. Producing more units than planned in the master budget will not affect the quantity of the materials usedfor each unit.

B. Rush orders disrupt the manufacturing process by interfering with normal work routines, practices, and procedures.These disruptions will adversely affect each of the manufacturing processes, including the efficient use of material,

labor, and overhead.

C. An inadequately trained and supervised labor force will have more material waste and spoilage than an adequatelytrained and supervised labor force.

D. Poorly functioning machines will have more material waste and spoilage.

Part 1 : 10/05/14 22:20:20

(c) HOCK international, page 31

Page 32: Hock_Performance Mgmt

8/16/2019 Hock_Performance Mgmt

http://slidepdf.com/reader/full/hockperformance-mgmt 32/152

Question 46 - CIA 1189 IV.17 - Manufacturing Input Variances -- Materials and Labor 

One of the items produced by a manufacturer of lawn and garden tools is a chain saw. The direct labor standard forassembling and testing a chain saw is 2.5 hours at $8 per hour. Budgeted production for October was 1,200 units. Actual production during the month was 1,000 units, and direct labor cost was $27,840 for 3,200 hours. What was thedirect labor price (rate) variance for October?

 A. $3,840 favorable.B. $2,240 unfavorable.C. $5,600 unfavorable.D. $2,240 favorable.

 A. This is the difference of actual labor cost of $27,840 and the standard labor cost for the scheduled level ofproduction of $24,000 ($8 × 2.5 hours × 1,200 units). Furthermore, the actual labor rate was greater than the standardlabor rate, so the variance is unfavorable.

B. The labor rate variance is calculated as follows: (Actual Rate − Standard Rate) × Actual Hours. The actuallabor rate was $8.70 ($27,840 ÷ 3,200). The standard labor rate was $8. The actual hours is 3,200. The labor ratevariance is ($8.70 − $8.00) × 3,200 unfavorable. The actual labor rate was greater than the standard labor rate,so the variance is unfavorable.

C. This is the direct labor efficiency variance. The question asks for the direct labor rate variance.

D. The actual labor rate was greater than the standard labor rate, so the variance is unfavorable.

Question 47 - ICMA 10.P1.093 - Manufacturing Input Variances -- Materials and Labor 

The benefits of management by exception reporting include all of the following except a reduction in

 A. reliance on advance planning.B. unfocused management actions.

C. reports production costs.D. information overload.

A. A reduction in reliance on advance planning is not a result of the use of management by exception.Advance planning is important regardless of whether management by exception is being used or not beingused.

B. When management by exception is used, the only things that are reported to management are variances whichindicate problems. This enables management to focus its efforts on things that need attention, so a benefit ofmanagement by exception is that there is a reduction of unfocused management actions.

C. Fewer reports are generated when management by exception is used, because the only things that are reported tomanagement are variances which indicate problems. Therefore, a benefit of management by exception is that reportsproduction costs are reduced.

D. Management receives less information when management by exception is used, because the only things that arereported to management are variances which indicate problems. Thus, a benefit of management by exception is thatinformation overload is reduced.

Question 48 - CMA 1296 3.16 - Responsibility Centers and Reporting Segments

Part 1 : 10/05/14 22:20:20

(c) HOCK international, page 32

Page 33: Hock_Performance Mgmt

8/16/2019 Hock_Performance Mgmt

http://slidepdf.com/reader/full/hockperformance-mgmt 33/152

Rockford Manufacturing Corporation uses a responsibility accounting system in its operations. Which one of thefollowing items is least likely to appear in a performance report for a manager of one of Rockford's assembly lines?

 A. Materials.B. Direct labor.C. Depreciation on the manufacturing facility.D. Repairs and maintenance.

 A. Materials would be controllable by the manager of an assembly line and would therefore be included on themanager's report.

B. Direct labor would be controllable by the manager of an assembly line and would therefore be included on themanager's report.

C. Depreciation on the manufacturing facility is a cost that is dependent upon past decisions made at a higherlevel of the organization, as well as on the accounting policies adopted by management. Therefore, it isoutside of the control of an assembly line manager and would not be included on the manager's performancereport.

D. Repairs and maintenance would be controllable by the manager of an assembly line and would therefore beincluded on the manager's report.

Question 49 - CMA 693 3.29 - Performance Measures

Which one of the following firms is likely to experience dysfunctional motivation on the part of its managers due to itsallocation methods?

 A. Tashkent Auto's MIS is operated out of headquarters and serves its various divisions. Tashkent's allocation of theMIS-related costs to its divisions is limited to costs the divisions will incur if they were to outsource their MIS needs.B. Manhattan Electronics uses the sales revenue of its various divisions to allocate costs connected with the upkeep ofits headquarters building. It also uses ROI to evaluate the divisional performances.C. Rainier Industrial does not allow its service departments to pass on their cost overruns to the productiondepartments.D. To allocate depreciation of forklifts used by workers at its central warehouse, Shahlimar Electronics usespredetermined amounts calculated on the basis of the long-term average use of the services provided.

 A. This question asks which of these allocation methods would lead to negative (dysfunctional) behavior on the part ofmanagers. By allocating costs equal to a market value of the services provided, the management of the divisions willuse the services in the necessary manner.

B. This question asks which of these allocation methods would lead to negative (dysfunctional) behavior onthe part of managers. If a division receives more allocated costs for increased levels of sales, this will serve asa minor demotivator for the manager of the divisions. Also, using ROI to measure divisional performance isprobably not the best method since the denominator in ROI will include things that are outside the control ofthe divisional managers. Furthermore, use of ROI to evaluate divisional performance could lead to managers'

rejecting profitable new investments because their ROI is lower than the divisions' historical ROI and couldbring down future ROI.

C. This question asks which of these allocation methods would lead to negative (dysfunctional) behavior on the part ofmanagers. By not allowing cost overruns in the service departments to be passed to other departments, this willmotivate the service departments to control their costs.

D. This question asks which of these allocation methods would lead to negative (dysfunctional) behavior on the part ofmanagers. The allocation of costs based on usage is a reasonable method for the allocation of these costs.

Part 1 : 10/05/14 22:20:20

(c) HOCK international, page 33

Page 34: Hock_Performance Mgmt

8/16/2019 Hock_Performance Mgmt

http://slidepdf.com/reader/full/hockperformance-mgmt 34/152

Question 50 - CMA Sample Q3.11 - Manufacturing Input Variances -- Materials and Labor 

Garland Company uses a standard cost system. The standard for each finished unit of product allows for 3 pounds ofplastic at $0.72 per pound. During December, Garland bought 4,500 pounds of plastic at $0.75 per pound and used4,100 pounds in the production of 1,300 finished units of product. What is the materials purchase price variance for the

month of December?

 A. $150 unfavorable.B. $117 unfavorable.C. $135 unfavorable.D. $123 unfavorable.

 A. The price variance is calculated as follows: (Actual Price − Standard Price) × Actual Quantity. The purchase pricevariance is calculated using all of the units purchased, not just the units that are put into production. See the correctanswer for a complete explanation.

B. The price variance is calculated as follows: (Actual Price − Standard Price) × Actual Quantity. The purchase pricevariance is calculated using all of the units purchased, not just the units that are put into production. See the correctanswer for a complete explanation.

C. The price variance is calculated as follows: (Actual Price − Standard Price) × Actual Quantity. The purchaseprice variance is calculated using all of the units purchased, not just the units that are put into production.The purchase price variance is $135 unfavorable [($0.75 − $0.72) × 4,500]. The actual price exceeds thestandard, thus, the variance is unfavorable.

D. The price variance is calculated as follows: (Actual Price − Standard Price) × Actual Quantity. The purchase pricevariance is calculated using all of the units purchased (4,500), not just the units that are put into production (4,100).See the correct answer for a complete explanation.

Question 51 - CMA 1292 3.21 - Performance Measures

 A firm earning a profit can increase its return on investment by

 A. Increasing sales revenues and operating expenses by the same percentage.B. Decreasing sales revenues and operating expenses by the same percentage.C. Increasing investment and operating expenses by the same dollar amount.D. Increasing sales revenue and operating expenses by the same dollar amount.

A.

Since the firm is profitable, revenue is greater than expenses, so increasing both revenue and expenses bythe same percentage will cause the increase in revenue to be larger than the increase in expenses. As a result,net income will increase and, since investment would not change, ROI will increase.

B. Decreasing sales will decrease income and decreasing expenses will increase income. Since the firm is profitable,income is greater than expenses, so decreasing both income and expenses by the same percentage will cause thedecrease in income to be larger than the decrease in expenses. As a result, net income will decline and, sinceinvestment would not change, ROI will decrease.

C. One way to solve this problem is to simply set up a basic ROI of, for example, $100,000 of profit / $400,000 totalassets giving a ROI of 0.25 and then go through the choices changing these figures as outlined in each option. In thischoice we should increase the investment (the denominator) and expenses (which will decrease income) by the sameamount. Let us assume that this amount is $50,000. That would make the ROI calculation $50,000 ÷ $450,000, which

Part 1 : 10/05/14 22:20:20

(c) HOCK international, page 34

Page 35: Hock_Performance Mgmt

8/16/2019 Hock_Performance Mgmt

http://slidepdf.com/reader/full/hockperformance-mgmt 35/152

is 0.111, or a decrease in the ROI.

D. One way to solve this problem is to simply set up a basic ROI of, for example, $100,000 of profit ÷ $400,000 totalassets giving a ROI of 0.25, and then go through the choices changing these figures as outlined in each option. In thischoice we would increase sales (which will increase income) and expenses (which will decrease income) by the sameamount. This will cause income to remain unchanged and so will not change the ROI of the company.

Question 52 - ICMA 10.P1.131 - Performance Measures

 All of the following are considered appropriate goals for measuring a division manager's efficiency for a budgetingperiod except

 A. a targeted share of the market.B. a reduction in the organizational structure (fewer employees doing a given amount of work).C. earnings per share projections.D. budgeted operating income.

 A. A targeted share of the market for a division is considered an appropriate goal for measuring a division manager's

efficiency for a budgeting period because the division's actual share of the market can be compared with the targetedshare of the market to measure the division manager's performance.

B. A targeted reduction in the organizational structure for a division is considered an appropriate goal for measuring adivision manager's efficiency for a budgeting period because the division's actual reduction accomplished can becompared with the targeted reduction to measure the manager's performance.

C. Earnings per share projections and actual earnings per share are calculated for the company as a whole.There is no such thing as earnings per share for an individual division. Therefore, earnings per shareprojections are not considered an appropriate goal for measuring a division manager's efficiency for abudgeting period.

D. Budgeted operating income for a division is considered an appropriate goal for measuring a division manager'sefficiency for a budgeting period because actual operating income can be compared with budgeted operating incometo measure the division manager's performance.

Question 53 - CIA 1191 IV.18 - Responsibility Centers and Reporting Segments

 A company plans to implement a bonus plan based on segment performance. In addition, the company plans toconvert to a responsibility accounting system for segment reporting. The following costs, which have been included inthe segment performance reports that have been prepared under the current system, are being reviewed to determineif they should be included in the responsibility accounting segment reports:

1. Corporate administrative costs allocated on the basis of net segment sales.

2. Personnel costs assigned on the basis of the number of employees in each segment.

3. Fixed computer facility costs divided equally among each segment.

4. Variable computer operational costs charged to each segment based on actual hours used times apredetermined standard rate; any variable cost efficiency or inefficiency remains in the computer department.

Of these four cost items, the only item that could logically be included in the segment performance reports prepared ona responsibility accounting basis would be the

 A. Variable computer operational costs.B. Fixed computer facility costs.C. Personnel costs.

Part 1 : 10/05/14 22:20:20

(c) HOCK international, page 35

Page 36: Hock_Performance Mgmt

8/16/2019 Hock_Performance Mgmt

http://slidepdf.com/reader/full/hockperformance-mgmt 36/152

D. Corporate administrative costs.

A.

When financial results are used to evaluate managers on their performance for rewards such as bonuses, theonly costs charged to each manager for purposes of the evaluation should be costs each manager cancontrol. Other costs may be allocated to each manager's responsibility center in the accounting system, but

they should be excluded from the amounts used in the managers' evaluations.

The best allocation basis and one that will permit the allocated costs to be validly used in manager evaluationis one in which the managers are able to control the incurrence of costs by their department. An allocationbased on the actual usage of the item being allocated is an appropriate method of allocation for responsibilityaccounting purposes because that is something the manager can control. In this case, variable computeroperational costs charged to each segment based on actual hours used is a charge the manager can controlby controlling the usage. Managers of individual segments cannot control the overall incurrence of thecomputer operational costs, but they can control their usage of the computer services and thus they cancontrol how much of the cost is allocated to their segments.

Therefore, these variable computer costs can be included in the responsibility accounting report. The otheranswer choices all represent costs that segment managers are not able to control.

B.

Costs included on segment performance reports should be costs that the segment manager can control. Managers ofindividual segments cannot control the incurrence of fixed computer costs incurred by the computer department, asthose are overhead costs for the entire company and the decisions about the costs are made elsewhere. Furthermore,this allocation is not made on the basis of usage of the computer department by each segment. Therefore, thisallocation of costs would not be appropriate to include on segment performance reports.

C. The "personnel costs" referred to here are human resource department administrative costs that are being allocatedto each segment on the basis of how many employees each segment has. This is an allocation of overhead. Costsincluded on segment performance reports should be costs that the segment manager can control. Managers ofindividual segments cannot control the incurrence of human resource department administrative costs, as those areoverhead costs for the entire company and the decisions about the costs are made elsewhere. Therefore, this

allocation of costs would not be appropriate to include on segment performance reports.

D.

Costs included on segment performance reports should be costs that the segment manager can control. Managers ofindividual segments cannot control the incurrence of corporate administrative costs, as those are overhead costs forthe entire company and the decisions about the costs are made elsewhere. Furthermore, the allocation of the costs onthe basis of segment revenues could make the managers feel they are being charged more, or "punished," for doingwell. A manager might even cut back on sales efforts, which would have a negative effect on the company, in an effortto cut the amount of common costs being allocated to his or her division.

It might be impossible to allocate corporate administrative costs on any basis that the individual managers can control.Thus for analysis purposes, corporate administrative costs could be allocated in this way within the internal accountingsystem. However, these allocated costs would not be appropriate to include on a segment performance report used for

manager evaluation.

Question 54 - CIA 1191 IV.15 - Manufacturing Input Variances -- Materials and Labor 

The total budgeted direct labor cost of a company for the month was set at $75,000 when 5,000 units were planned tobe produced. The following standard cost, stated in terms of direct labor hours (DLH), was used to develop the budgetfor direct labor cost:

Part 1 : 10/05/14 22:20:20

(c) HOCK international, page 36

Page 37: Hock_Performance Mgmt

8/16/2019 Hock_Performance Mgmt

http://slidepdf.com/reader/full/hockperformance-mgmt 37/152

1.25 DLH x $12.00/DLH = $15.00/unit produced

The actual operating results for the month were as follows: Actual units produced 5,200 Actual direct labor hours worked 6,600 Actual direct labor cost $77,220

The direct labor efficiency variance for the month would be

 A. $1,200 unfavorable.B. $3,000 unfavorable.C. $2,220 unfavorable.D. $4,200 unfavorable.

A. The direct labor efficiency variance is calculated as follows: (Actual Hours − Standard Hours for ActualOutput) × Standard Rate. The standard hours for the actual output were 6,500 (1.25 DLH × 5,200 actual unitsproduced). The standard rate is $12. Thus, the direct labor efficiency variance is (6,600 − 6,500) × $12/DLH =$1,200 unfavorable. Since the actual hours were greater than the standard hours for the actual output, thevariance is unfavorable.

B. The direct labor efficiency variance is calculated as follows: (Actual Hours − Standard Hours for Actual Output) ×Standard Rate. This incorrect answer results from using the standard hours for the actual output (1.25 DLH × 5,200 =6,500 DLH) instead of the actual hours and the standard hours for the planned output (1.25 DLH × 5,000 = 6,250 DLH)instead of the standard hours for the actual output.

C. This is the difference between actual costs and budgeted costs (master budget). This is not the direct laborefficiency variance.

D. The direct labor efficiency variance is calculated as follows: (Actual Hours − Standard Hours for Actual Output) ×Standard Rate. This incorrect answer of $4,200 unfavorable results from using the standard hours for the initiallyplanned output of 5,000 units or master budget figure (1.25 DLH × 5,000 = 6,250 DLH) instead of the standard hoursallowed for the actual output.

Question 55 - ICMA 10.P1.092 - Variance Analysis Concepts

 An advantage of using a flexible budget compared to a static budget is that in a flexible budget

 A. budgeted costs for a given output level can be compared with actual costs for the same level of output.B. shortfalls in planned production are clearly presented.C. standards can easily be changed to adjust to changing circumstances.D. fixed cost variances are more clearly presented.

A. This statement is somewhat backwards, but it is the best answer choice from among those given. Flexiblebudget amounts for variable revenues and costs are adjusted to the actual level of activity that has occurredbefore actual revenues and costs are compared with them. So the "given" output level is derived from the

actual activity level, not the other way around.

B. If production is less than was planned, variances in a static budget variance report will be greater than variances in aflexible budget variance report. The decrease in costs due to the decrease in production will be included in the staticbudget variance report, whereas decreases in costs due to decreases in production are not included in a flexiblebudget report.

C. To be as meaningful as possible, standards should be updated whenever circumstances change, regardless ofwhether a static budget or a flexible budget is being used. The use of a flexible budget does not make it easier toupdate standards.

Part 1 : 10/05/14 22:20:20

(c) HOCK international, page 37

Page 38: Hock_Performance Mgmt

8/16/2019 Hock_Performance Mgmt

http://slidepdf.com/reader/full/hockperformance-mgmt 38/152

D. Fixed cost variances are not more clearly presented on a flexible budget variance report than on a static budgetvariance report.

Question 56 - CMA 1291 3.9 - Responsibility Centers and Reporting Segments

 A segment of an organization is referred to as an investment center if it has

 A. Authority to make decisions affecting the major determinants of profit including the power to choose its markets andsources of supply and significant control over the amount of invested capital.B. Authority to make decisions affecting the major determinants of profit including the power to choose its markets andsources of supply.C. Authority to provide specialized support to other units within the organization.D. Authority to make decisions over the most significant costs of operations including the power to choose the sourcesof supply.

A. By definition, an investment center is a part of the business that has the authority to make decisionsaffecting the major determinants of profit including the power to choose its markets and sources of supply

and significant control over the amount of invested capital.

B. This is the definition of a profit center. An investment center also has a responsibility for invested capital.

C. This is the definition of a service center, which is a type of cost center.

D. This is the definition of a cost center.

Question 57 - ICMA 10.P1.135 - Performance Measures

Which one of the following should be used for evaluating the performance of the Repair and Maintenance Department

that repairs production equipment in a firm devoted to making keyboards for computers?

 A. The variance between the firm’s budgeted and actual net income.B. The fixed overhead volume variances.C. The response time and degree of satisfaction among the production departments.D. The total factory overhead variances.

 A. The "customers" of the Repair and Maintenance Department are internal. They are the users of the productionequipment that the Repair and Maintenance Department is responsible for keeping in good working order. Although theRepair and Maintenance Department may be indirectly responsible for an impact on net income if they do not do theirob and do not keep the production machinery running, they are not responsible for activities that directly impact thevariance between the firm's budgeted and actual net income. Performance measures that measure the department'sperformance more directly should be used.

B. The "customers" of the Repair and Maintenance Department are internal. They are the users of the productionequipment that the Repair and Maintenance Department is responsible for keeping in good working order. Although theRepair and Maintenance Department may be indirectly responsible for an impact on factory overhead costs if they donot do their job and do not keep the production machinery running, they are not responsible for activities that directlyimpact the fixed overhead volume variances or any other factory variances. Performance measures that measure thedepartment's performance more directly should be used.

C. The "customers" of the Repair and Maintenance Department are internal. They are the users of theproduction equipment that the Repair and Maintenance Department is responsible for keeping in goodworking order. The response time and degree of satisfaction among the Repair and Maintenance Department's

Part 1 : 10/05/14 22:20:20

(c) HOCK international, page 38

Page 39: Hock_Performance Mgmt

8/16/2019 Hock_Performance Mgmt

http://slidepdf.com/reader/full/hockperformance-mgmt 39/152

customers is an appropriate measurement for evaluating the performance of the Repair and MaintenanceDepartment.

D. The "customers" of the Repair and Maintenance Department are internal. They are the users of the productionequipment that the Repair and Maintenance Department is responsible for keeping in good working order. Although theRepair and Maintenance Department may be indirectly responsible for an impact on factory overhead costs if they donot do their job and do not keep the production machinery running, they are not responsible for activities that directly

impact the total factory overhead variances or any other factory variances. Performance measures that measure thedepartment's performance more directly should be used.

Question 58 - CMA 691 3.24 - Performance Measures

Most firms use return on investment (ROI) to evaluate the performance of investment center managers. If topmanagement wishes division managers to use all assets without regard to financing, the denominator in the ROIcalculation will be

 A. Working capital plus other assets.B. Shareholders' equity.

C. Total assets employed.D. Total assets available.

 A. By using working capital in this measure the company is excluding assets that are financed by short-termobligations. This is because working capital is current assets minus current liabilities.

B. Using shareholders' equity takes into account only those assets financed with equity. It does not include assets thatwere financed by debt, or other liabilities.

C. If the company uses total assets employed, there will be no motivation for the managers to make better use of theassets that are currently not employed.

D. If the company does not care what the financing of the assets used is, the company should use total assetsavailable as the denominator in ROI. This will mean that managers are expected to use all of the assets togenerate return for the company.

Question 59 - ICMA 10.P1.133 - Performance Measures

Paul Cooper, shipping manager for DFG Distributors, is responsible for managing the staff and all relatedtransportation equipment to fill orders for bakery products from local retailers and deliver the products to thoseretailers. Which one of the following groups of three performance measures most likely would result in the highestlevel of goal congruence?

 A. Labor cost per order; transportation cost per order; number of orders completed per day.

B. Customer satisfaction; elapsed time to complete an order; percentage of orders filled accurately.C. Orders completed per employee per day; employee injuries per hour worked; number of vehicle accidents per year.D. The percentage of orders filled on time; the percentage of orders filled accurately; average cost to fill and deliver anorder.

 A. The shipping manager is responsible for filling customer orders efficiently and accurately and delivering themquickly, at the lowest possible cost without negatively impacting customer service. Transportation cost per order is notsomething the shipping manager can control, as decisions about delivery vehicles are made at another level and fuelcosts are not controllable. Number of orders completed per day is not meaningful because it does nothing to measurehow quickly and accurately orders are filled after they are received.

Part 1 : 10/05/14 22:20:20

(c) HOCK international, page 39

Page 40: Hock_Performance Mgmt

8/16/2019 Hock_Performance Mgmt

http://slidepdf.com/reader/full/hockperformance-mgmt 40/152

B. The shipping manager is responsible for filling customer orders efficiently and accurately and delivering themquickly, at the lowest possible cost without negatively impacting customer service. Customer satisfaction could meancustomer satisfaction with the product as well as with the speed and accuracy of the delivery; and the shippingmanager is not responsible for the product so he should not be evaluated on that. Elapsed time to complete an orderdoes relate to the speed with which an order is filled. However, if there are no standards to compare the elapsed timeagainst, the measurement is not meaningful.

C. The shipping manager is responsible for filling customer orders efficiently and accurately and delivering themquickly, at the lowest possible cost without negatively impacting customer service. Orders completed per employee perday is not meaningful because that can be affected by the number of orders received, and the number of ordersreceived is not something the shipping manager can control. Though safety on the job is important, employee injuriesper hour worked and number of vehicle accidents per year are not related to the shipping manager's primaryresponsibilities.

D.

The shipping manager is responsible for filling customer orders efficiently and accurately and delivering themquickly, at the lowest possible cost without negatively impacting customer service.

Percentage of orders filled on time, percentage of orders filled accurately and average cost to fill and deliveran order are determined by comparing actual performance with established standards for those performancemeasures. These are appropriate measurements for aligning the shipping manager's goals with those of thecompany, because as the manager strives to attain those standards, he will be fulfilling his responsibilities.

Question 60 - ICMA 10.P1.108 - Manufacturing Input Variances -- Materials and Labor 

Richter Company has an unfavorable materials efficiency (usage) variance for a particular month. Which one of thefollowing is least likely to be the cause of this variance?

 A. Poor performance of the shipping employees.B. Poor design of the production process or product.

C. Inadequate training of the direct labor employees.D. Poor quality of the raw materials.

A. The performance of the shipping employees is not connected with the production process. Therefore,neither good nor bad performance of them can affect the materials efficiency variance.

B. Poor design can cause many production problems. One of the problems it could cause is material waste, which willlead to unfavorable materials efficiency variances.

C. Inadequate training of the direct labor employees could be a cause of the unfavorable materials efficiency (usage)variance. Inadequately trained labor will make more mistakes, thereby wasting more direct materials than willadequately trained labor.

D. Poor quality of the raw materials can cause an unfavorable material efficiency variance, because more of the direct

material will be wasted.

Question 61 - CMA 693 3.14 - Responsibility Centers and Reporting Segments

The least complex segment or area of responsibility for which costs are allocated is a(n):

 A. Investment center.

Part 1 : 10/05/14 22:20:20

(c) HOCK international, page 40

Page 41: Hock_Performance Mgmt

8/16/2019 Hock_Performance Mgmt

http://slidepdf.com/reader/full/hockperformance-mgmt 41/152

B. Cost center.C. Contribution center.D. Profit center.

 A. An investment center is responsible for revenues, costs and investment. Therefore, it is not the least complexsegment or area of responsibility for which costs are allocated.

B. A cost center is the least complex of the different types of centers. It is responsible only for the incurrenceof costs.

C. Contribution center is not a term used in responsibility center analysis and reporting.

D. A profit center is responsible for both revenues and expenses. Therefore, it is not the least complex segment orarea of responsibility for which costs are allocated.

Question 62 - CMA 692 3.20 - Manufacturing Input Variances -- Materials and Labor 

Jackson Industries employs a standard cost system in which direct materials inventory is carried at standard cost.

Jackson has established the following standards for the prime costs of one unit of product.

 StandardQuantity

StandardPrice

StandardCost

Direct materials 5 pounds$3.60/pound $18.00Direct labor 1.25 hours 12.00/hour 15.00  $33.00

During May, Jackson purchased 125,000 pounds of direct materials at a total cost of $475,000. The total factory wagesfor May were $364,000, 90% of which were for direct labor. Jackson manufactured 22,000 units of product during Mayusing 108,000 pounds of direct materials and 28,000 direct labor hours.

The direct labor price (rate) variance for May is

 A. $7,200 unfavorable.B. $6,000 unfavorable.C. $8,400 unfavorable.D. $8,400 favorable.

 A. Since the actual labor rate of $11.70 is lower than the standard labor rate of $12, the variance is favorable. See thecorrect answer for a complete explanation.

B. This result is the direct labor efficiency variance, but this question asks for the direct labor rate variance. See thecorrect answer for a complete explanation.

C. Since the actual labor rate of $11.70 is lower than the standard labor rate of $12, the variance is favorable. See thecorrect answer for a complete explanation.

D. The labor price/rate variance is calculated as: (Actual Rate − Standard Rate) × Actual Hours. The actual totaldirect labor cost is $327,600 ($364,000 × 90%), and the actual labor rate is $11.70 ($327,600 ÷ 28,000 hoursused). The standard labor rate is $12. The actual hours used is 28,000. Therefore, the labor rate variance is($11.70 − $12.00) × 28,000 = ($8,400) favorable. Since the actual labor rate of $11.70 is lower than the standardlabor rate of $12, the variance is favorable.

Question 63 - ICMA 10.P1.143 - Performance Measures

Part 1 : 10/05/14 22:20:20

(c) HOCK international, page 41

Page 42: Hock_Performance Mgmt

8/16/2019 Hock_Performance Mgmt

http://slidepdf.com/reader/full/hockperformance-mgmt 42/152

To insure that a divisional vice president places appropriate focus on both the short-term and the long-term objectivesof the division, the best approach would be to evaluate the vice president’s performance by using

 A. residual income since it will eliminate the rejection of capital investments that have a return less than ROI butgreater than the cost of capital.B. financial and nonfinancial measures, including the evaluation of quality, customer satisfaction, and market

performance.C. division segment margin or profit margin.D. return on investment (ROI) which permits easy and quick comparisons to other similar divisions.

 A. Residual income focuses on the achievement of short-term objectives.

B. Financial measures such as return on investment, residual income, and profit margins focus on theachievement of short-term objectives. Nonfinancial measures such as evaluation of quality, customersatisfaction and market performance focus on the achievement of long-term objectives.

C. Segment margin and profit margin focus on the achievement of short-term objectives.

D. Return on investment focuses on the achievement of short-term objectives.

Question 64 - ICMA 10.P1.120 - Responsibility Centers and Reporting Segments

The production manager of the Super T-shirt Company is responsible for the activity of her department and the costsassociated with production. Super T adheres to a responsibility centered budget process, and the manager’sperformance is measured by how well she performs to budget. Recently, the dark horse team won the local collegebasketball tournament. As a result, the sales department, which operates as a profit center, received an order for10,000 t-shirts, but only if they could be delivered in three days. The production manager said she could meet theschedule, but only by incurring overtime pay that would cause her to be over budget for hourly wages paid. Whatwould be the best course of action for the sales department and the production manager to undertake in this case?

 A. Charge the overtime to the sales department's budget.

B. Accept the order and ignore the effect on the production budget when conducting the performance review.C. Refuse the overtime and produce only what the production department is capable of while staying within the budget.D. Accept the order and overrun the production manager's budget.

A.

The amount the customer is charged for the order should include an additional charge for the overtimerequired to meet the three day delivery requirement. On the expense side, the additional overtime cost shouldbe charged to the sales department, if the sales department operates as a profit center and has expenses. Thevariances caused by the additional income and the additional expense will offset one another in the salesdepartment's variance reporting.

If the sales department does not operate as a profit center, then both the additional income from charging the

customer for the overtime and the overtime expense should be allocated to a department that does operate asa profit center, so the variances caused by the additional income and the additional expense will offset oneanother.

B. This is not a satisfactory course of action. The amount the customer is charged for the order should include anadditional charge for the overtime required to meet the three day delivery requirement. If the additional revenue for theovertime is not credited to the production department, the additional cost for the overtime should not be chargedagainst the production department, either. A promise to ignore the effect of the additional cost (without the offsettingrevenue) is something that might be forgotten by the time the production manager's performance is reviewed.Furthermore, performance reviews are not the only use made of responsibility center reports. They are also used for

Part 1 : 10/05/14 22:20:20

(c) HOCK international, page 42

Page 43: Hock_Performance Mgmt

8/16/2019 Hock_Performance Mgmt

http://slidepdf.com/reader/full/hockperformance-mgmt 43/152

internal decision-making. Charging the additional cost to the production budget without an offsetting revenue itemwould distort the performance reports of both the sales department and the production department.

C. If the order is a profitable one, the order should be accepted because that is what is best for the company as awhole. The amount the customer is charged for the order should include an additional charge for the overtime requiredto meet the three day delivery requirement.

D. The amount the customer is charged for the order should include an additional charge for the overtime required tomeet the three day delivery requirement. If the additional revenue for the overtime is not credited to the productiondepartment, the additional cost for the overtime should not be charged against the production department.

Question 65 - CIA 1185 IV.12 - Sales and Market Variances

 Actual and budgeted information about the sales of a product are presented for June as follows.  Actual BudgetUnits 8,000 10,000Sales Revenue$92,000$105,000

The sales price variance for June was

 A. $10,000 unfavorable.B. $10,500 unfavorable.C. $10,000 favorable.D. $8,000 favorable.

 A. The actual sales price ($11.50) was higher than budgeted sales price ($10.50), which means that variance isfavorable. See the correct answer for a complete explanation.

B. The actual sales price ($11.50) was higher than budget ($10.50), which means that variance is favorable. See thecorrect answer for a complete explanation.

C. This answer is the result of using the budgeted quantity of units to be sold (10,000) in the variance formula. Theactual quantity of units sold (8,000) should be used. See the correct answer for a complete explanation.

D.

The sales price variance for revenue measures the impact of the difference in the sales price per unit betweenactual and budgeted amounts and is calculated as follows: (Actual Price per unit − Standard Price per unit) ×Actual Quantity, or (AP − SP) × AQ.

The actual unit sales price is $11.50 ($92,000 ÷ 8,000). The budgeted unit sales price is $10.50 ($105,000 ÷10,000). The sales price variance is ($11.50 − $10.50) × 8,000, which equals $8,000. Since the actual sales pricewas higher than the budgeted sales price, the variance is positive. For an income item, a positive variance isfavorable.

Question 66 - CMA 1291 3.14 - Sales and Market Variances

Folsom Fashions sells a line of women's dresses. Folsom's performance report for November follows.Actual Budget

Dresses sold 5,000 6,000Sales $ 235,000 $ 300,000Variable costs (145,000) (180,000)

Part 1 : 10/05/14 22:20:20

(c) HOCK international, page 43

Page 44: Hock_Performance Mgmt

8/16/2019 Hock_Performance Mgmt

http://slidepdf.com/reader/full/hockperformance-mgmt 44/152

Contribution margin $ 90,000 $ 120,000Fixed costs (84,000) (80,000)Operating income $ 6,000 $ 40,000

The company uses a flexible budget to analyze its performance and to measure the effect on operating income of thevarious factors affecting the difference between budgeted and actual operating income.

The effect of the sales quantity variance on the contribution margin for November is

 A. $20,000 unfavorable.B. $15,000 unfavorable.C. $18,000 unfavorable.D. $30,000 unfavorable.

A. This question is asking for the sales volume/quantity variance on the contribution margin that is calculatedas follows: (Actual Sales Volume − Budgeted Sales Volume) × Standard Contribution per Unit. The totalbudgeted contribution margin was $120,000, which gives us a $20 contribution margin per unit ($120,000 ÷6,000). Now we can calculate the sales volume variance: (5,000 − 6,000) × $20 = ($20,000) unfavorable. Theactual sales volume was lower than budgeted, and that caused the negative impact of $20,000 on thecontribution margin.

B. This is the sales price variance. See the correct answer for a complete explanation.

C. This answer results from using the actual unit contribution margin instead of the budgeted unit contribution margin.See the correct answer for a complete explanation.

D. This answer is the total sales variance, which includes variances caused by differences in both the sales price andin the quantity sold. The total sales variance on the contribution margin is the difference between actual and budgetedamount of contribution margin. The sales volume/quantity variance is calculated as follows: (Actual Sales Volume −Budgeted Sales Volume) × Budgeted Contribution per Unit. See the correct answer for a complete explanation.

Question 67 - CIA 595 III.96 - Responsibility Centers and Reporting Segments

Which of the following techniques would be best for evaluating the management performance of a department that isoperated as a cost center?

 A. Variance analysis.B. Payback method.C. Return on assets ratio.D. Return on investment ratio.

A. A cost center is a type of center in responsibility accounting classifications that is responsible only for theincurrence of costs. A cost center does not have any revenue, and therefore does not have any profit. Thus,performance evaluation based on variance analysis of costs is the best basis for performance evaluation of acost center manager.

B. The payback method is used to analyze alternative investment opportunities, not management performance.

C. A cost center is a type of center in responsibility accounting classifications that is responsible only for the incurrenceof costs. A cost center does not have any revenue, and therefore does not have any profit. Thus, a return on assetsratio cannot be calculated for a cost center.

D. Cost center is a type of center in responsibility accounting classifications that is responsible only for the incurrenceof costs. A cost center does not have any revenue, and therefore does not have any profit. Thus, a return oninvestment ratio cannot be calculated for a cost center.

Part 1 : 10/05/14 22:20:20

(c) HOCK international, page 44

Page 45: Hock_Performance Mgmt

8/16/2019 Hock_Performance Mgmt

http://slidepdf.com/reader/full/hockperformance-mgmt 45/152

Page 46: Hock_Performance Mgmt

8/16/2019 Hock_Performance Mgmt

http://slidepdf.com/reader/full/hockperformance-mgmt 46/152

labor variance. Therefore, the total labor variance minus the labor efficiency variance must equal the labor rate (price)variance.

Question 70 - CMA 1292 3.17 - Manufacturing Input Variances -- Overhead

Nanjones Company manufactures a line of products distributed nationally through wholesalers. Presented below areplanned manufacturing data for the year and actual data for November of the current year. The company appliesoverhead based on planned machine hours using a predetermined annual rate.

Planning Data  Annual November Fixed manufacturing overhead $1,200,000 $100,000Variable manufacturing overhead 2,400,000 220,000Direct labor hours 48,000 4,000Machine hours 240,000 22,000

Data for November  Direct labor hours (actual) 4,200Direct labor hours (plan based on output) 4,000

Machine hours (actual) 21,600Machine hours (plan based on output) 21,000Fixed manufacturing overhead $101,200Variable manufacturing overhead $214,000

The amount of over or underapplied variable manufacturing overhead for November was

 A. $2,000 overapplied.B. $4,000 underapplied.C. $6,000 underapplied.D. $6,000 overapplied.

 A.

This answer results from applying the variable overhead using normal costing, which is incorrect. The problem saysthat overhead is applied based on planned machine hours, which means standard costing is being used.

Under standard costing, overhead to be applied is calculated by multiplying the predetermined rate by the amount of (inthis case) machine hours that should have been used  for the amount actually produced.

Under normal costing, overhead to be applied is calculated by multiplying the predetermined rate by the amount of theallocation base that was actually used for the amount actually produced.

B.

Nanjones applies overhead based on planned machine hours using a predetermined annual rate. The amountof planned variable manufacturing overhead was $2,400,000 and amount of planned machine hours were

240,000. Thus, the application rate for variable manufacturing overhead was $10 per hour ($2,400,000 /240,000).

Under standard costing -- which is being used here because the problem tells us that overhead is appliedbased on planned machine hours -- overhead to be applied is calculated by multiplying the predetermined rateby the amount of (in this case) machine hours that should have been used  for the amount actually produced.The problem tells us that the planned machine hours based on output was 21,000, and therefore, the amountof variable overhead applied was $10 × 21,000, or $210,000. The actual variable overhead incurred was$214,000. Therefore, variable manufacturing overhead was underapplied by $4,000.

Part 1 : 10/05/14 22:20:20

(c) HOCK international, page 46

Page 47: Hock_Performance Mgmt

8/16/2019 Hock_Performance Mgmt

http://slidepdf.com/reader/full/hockperformance-mgmt 47/152

C.

This answer results from two errors:

(1) Calculating the amount of variable overhead applied by multiplying the predetermined rate by the amount ofmachine hours planned to be used for the planned output. However, under standard costing -- which is being usedhere because the problem tells us that overhead is applied based on planned machine hours -- overhead to be applied

is calculated by multiplying the predetermined rate by the amount of (in this case) machine hours that should havebeen used  for the amount actually produced.

(2) Reversing or misinterpreting the calculation of the variance amount. The variance is the actual variable overheadincurred minus the variable overhead applied. Because overhead is a cost, a positive result is an unfavorable variance,and the variable overhead was underapplied (less was applied than was incurred). A negative result is a favorablevariance, and the variable overhead was overapplied (more was applied than was incurred).

D.

This answer results from calculating the amount of variable overhead applied by multiplying the predetermined rate bythe amount of machine hours planned to be used for the planned output. However, under standard costing -- whichis being used here because the problem tells us that overhead is applied based on planned machine hours -- overheadto be applied is calculated by multiplying the predetermined rate by the amount of (in this case) machine hours that

should have been used  for the amount actually produced.

Question 71 - CMA 692 3.19 - Manufacturing Input Variances -- Materials and Labor 

Jackson Industries employs a standard cost system in which direct materials inventory is carried at standard cost.Jackson has established the following standards for the prime costs of one unit of product.

 StandardQuantity

StandardPrice

StandardCost

Direct materials 5 pounds$3.60/pound $18.00Direct labor 1.25 hours 12.00/hour 15.00

  $33.00

During May, Jackson purchased 125,000 pounds of direct materials at a total cost of $475,000. The total factory wagesfor May were $364,000, 90% of which were for direct labor. Jackson manufactured 22,000 units of product during Mayusing 108,000 pounds of direct materials and 28,000 direct labor hours.

The direct materials usage (quantity) variance for May is

 A. $5,850 unfavorable.B. $7,200 unfavorable.C. $7,600 favorable.D. $7,200 favorable.

 A. Since the actual quantity used is less than the standard quantity budgeted for the current level of production, the

variance is favorable. See the correct answer for a complete explanation.

B. Since the actual quantity used is less than the standard quantity budgeted for the current level of production, thevariance is favorable. See the correct answer for a complete explanation.

C. The material usage variance is calculated as: (Actual Quantity − Standard Quantity for Actual Output) × StandardPrice. This answer results from using the actual price paid for the materials during the period, not the standard price.

D. The material usage variance is calculated as: (Actual Quantity − Standard Quantity for Actual Output) ×Standard Price. The actual quantity is 108,000 pounds. The standard quantity allowed for the actual output is

Part 1 : 10/05/14 22:20:20

(c) HOCK international, page 47

Page 48: Hock_Performance Mgmt

8/16/2019 Hock_Performance Mgmt

http://slidepdf.com/reader/full/hockperformance-mgmt 48/152

110,000 pounds (5 pounds of direct materials per unit of product times 22,000 units produced). The standardprice is $3.60 per pound. The material usage variance is (108,000 − 110,000) × $3,60 = $(7,200) favorable. Thevariance is favorable because the actual quantity used is less than the standard quantity for the actual output,and this is a cost variance.

Question 72 - CMA 1295 3.5 - Responsibility Centers and Reporting Segments

Responsibility accounting defines an operating center that is responsible for revenue and costs as a(n):

 A. Operating unit.B. Profit center.C. Revenue center.D. Division.

 A. An operating unit could be any of the different types of responsibility centers.

B. A profit center is responsible for both revenues and costs.

C. A revenue center is responsible only for revenues and not for costs.

D. A division could be any of the different types of responsibility centers.

Question 73 - ICMA 10.P2.236 - Responsibility Centers and Reporting Segments

The Robo Division, a decentralized division of GMT Industries, has been approached to submit a bid for a potentialproject for the RSP Company. Robo Division has been informed by RSP that they will not consider bids over$8,000,000. Robo Division purchases its materials from the Cross Division of GMT Industries. There would be noadditional fixed costs for either the Robo or Cross Divisions. Information regarding this project is as follows.

  Cross DivisionRobo DivisionVariable Costs $1,500,000 $4,800,0001

Transfer Price 3,700,000 —

If Robo Division submits a bid for $8,000,000, the amount of contribution margin recognized by the Robo Division andGMT Industries, respectively, is

1Robo Division's variable costs of $4,800,000 represent variable conversion costs only (direct labor and variableoverhead), since the division purchases its raw materials from the Cross Division of the same company.

 A. Robo $3,200,000 and GMT $(500,000).B. Robo $(500,000) and GMT $(2,000,000).C. Robo $3,200,000 and GMT $1,700.000.D. Robo $(500,000) and GMT $1,700,000.

 A.

The amount of contribution margin recognized by the Robo Division is calculated here as $8,000,000 minus RoboDivision's variable conversion costs of $4,800,000. This is incorrect because it omits the amount paid by Robo Divisionto Cross Division for raw materials.

The amount of contribution margin recognized by the consolidated company, GMT Industries, is calculated here as$8,000,000 minus Robo Division's variable conversion costs of $4,800,000 and the the amount paid by Robo to Crossof $3,700,000 for raw materials. This is incorrect for two reasons: (1) the consolidated GMT Industries will also have

Part 1 : 10/05/14 22:20:20

(c) HOCK international, page 48

Page 49: Hock_Performance Mgmt

8/16/2019 Hock_Performance Mgmt

http://slidepdf.com/reader/full/hockperformance-mgmt 49/152

Cross Division's variable costs of $1,500,000, so they should also be subtracted from the revenue to be earned. (2)The amount paid by Robo to Cross for raw materials ($3,700,000) should not be included because it has no effect onthe consolidated company's contribution margin or net income. The revenue to the Cross Division is the same as thecost to the Robo Division. Both amounts are eliminated in the consolidation. Intercompany sales are eliminated inconsolidation because including them would inflate both revenue and expenses for the consolidated company.

The intercompany sales are included in each division's segment report they are eliminated in the consolidation.

B.

The amount for Robo's contribution margin is correct. However, the amount of contribution margin recognized by GMTIndustries calculated here results from subtracting Cross Division's variable costs ($1,500,000), Robo Division'svariable conversion costs ($4,800,000), and the transfer priced costs for the raw materials sold by Cross to Robo($3,700,000) from the revenue of $8,000,000. The transfer priced costs for materials purchased by Robo from Crossshould not be included in either the consolidated company revenue or in the consolidated company costs because theyhave no effect on the consolidated company. The revenue to the Cross Division is the same as the cost to the RoboDivision, and both amounts are eliminated in the consolidation. Intercompany sales are eliminated in consolidationbecause including them would inflate both revenue and expenses for the consolidated company.

The intercompany sales are included in each division's segment report, but they are eliminated in the consolidation.

C.

The amount of contribution margin recognized by the Robo Division is calculated here as $8,000,000 minus RoboDivision's variable conversion costs of $4,800,000. This is incorrect because it omits the amount paid by Robo to Crossfor the raw materials.

The intercompany sales are included in each division's segment report they are eliminated in the consolidation. Thus,the amount for GMT's contribution margin is correct.

D.

The contribution margin recognized by Robo Division will be sales of $8,000,000 minus its own variableconversion costs of $4,800,000 minus $3,700,000 for the materials supplied by Cross, which equals $(500,000).

The contribution margin recognized by the consolidated company, GMT, will be sales of $8,000,000 minusRobo's variable conversion costs of $4,800,000 minus Cross’s variable costs of $1,500,000 for production ofthe raw materials, which is $1,700,000.

The amount paid by Robo to Cross for the raw materials has no effect on the consolidated company'scontribution margin or net income. The revenue to the Cross Division is the same as the cost to the RoboDivision. Intercompany sales are eliminated in consolidation, so both the revenue to Cross and the cost toRobo are eliminated in the consolidation. Intercompany sales are eliminated in consolidation becauseincluding them would inflate both revenue and expenses for the consolidated company.

The intercompany sales are included in each division's segment report and therefore they are included inRobo's contribution margin; but they are eliminated in the consolidation.

Question 74 - CMA 1295 3.4 - Manufacturing Input Variances -- Overhead

Variable overhead is applied on the basis of standard direct labor hours. If, for a given period, the direct labor efficiencyvariance is unfavorable, the variable overhead efficiency variance will be:

 A. Zero.B. Unfavorable.

Part 1 : 10/05/14 22:20:20

(c) HOCK international, page 49

Page 50: Hock_Performance Mgmt

8/16/2019 Hock_Performance Mgmt

http://slidepdf.com/reader/full/hockperformance-mgmt 50/152

C. Favorable.D. The same amount as the labor efficiency variance.

 A.

If the direct labor efficiency variance is unfavorable, the variable overhead efficiency variance will not be zero.

The direct labor efficiency variance is (Actual Hours − Standard Hours for Actual Output) × Standard Labor Rate.

The variable overhead efficiency variance is (Actual Activity Level of VOH allocation base used for Actual Output −Standard Activity Level of VOH allocation base allowed for Actual Output) × Standard Application Rate.

Since direct labor hours is the allocation base, the "actual" and "standard" will be the same in both formulas. The onlydifference between the direct labor efficiency variance and the variable overhead efficiency variance will be the hourlyrate that the difference between "actual" and "standard" is multiplied by.

B.

The direct labor efficiency variance is (Actual Hours − Standard Hours for Actual Output) × Standard LaborRate.

The variable overhead efficiency variance is (Actual Activity Level of VOH allocation base used for ActualOutput − Standard Activity Level of VOH allocation base allowed for Actual Output) × Standard ApplicationRate.

Since direct labor hours is the allocation base, the "actual" and "standard" will be the same in both formulas.The only difference between the direct labor efficiency variance and the variable overhead efficiency variancewill be the hourly rate that the difference between "actual" and "standard" is multiplied by.

Therefore, if the direct labor efficiency variance is unfavorable, the variable overhead efficiency variance willalso be unfavorable. (And if the direct labor efficiency variance is favorable, the variable overhead efficiencyvariance will also be favorable.)

C.

If the direct labor efficiency variance is unfavorable, the variable overhead efficiency variance will not be favorable.

The direct labor efficiency variance is (Actual Hours − Standard Hours for Actual Output) × Standard Labor Rate.

The variable overhead efficiency variance is (Actual Activity Level of VOH allocation base used for Actual Output −Standard Activity Level of VOH allocation base allowed for Actual Output) × Standard Application Rate.

Since direct labor hours is the allocation base, the "actual" and "standard" will be the same in both formulas. The onlydifference between the direct labor efficiency variance and the variable overhead efficiency variance will be the hourlyrate that the difference between "actual" and "standard" is multiplied by.

D.

If the direct labor efficiency variance is unfavorable, the variable overhead efficiency variance will not be the sameamount as the labor efficiency variance.

The direct labor efficiency variance is (Actual Hours − Standard Hours for Actual Output) × Standard Labor Rate.

The variable overhead efficiency variance is (Actual Activity Level of VOH allocation base used for Actual Output −Standard Activity Level of VOH allocation base allowed for Actual Output) × Standard Application Rate.

Since direct labor hours is the allocation base, the "actual" and "standard" will be the same in both formulas. Thedifference between the direct labor efficiency variance and the variable overhead efficiency variance will be the hourly

Part 1 : 10/05/14 22:20:20

(c) HOCK international, page 50

Page 51: Hock_Performance Mgmt

8/16/2019 Hock_Performance Mgmt

http://slidepdf.com/reader/full/hockperformance-mgmt 51/152

rate that the difference between "actual" and "standard" is multiplied by.

Question 75 - CMA 1291 3.17 - Variance Analysis Concepts

Folsom Fashions sells a line of women's dresses. Folsom's performance report for November follows.Actual Budget

Dresses sold 5,000 6,000Sales $ 235,000 $ 300,000Variable costs (145,000) (180,000)Contribution margin $ 90,000 $ 120,000Fixed costs (84,000) (80,000)Operating income $ 6,000 $ 40,000

The company uses a flexible budget to analyze its performance and to measure the effect on operating income of thevarious factors affecting the difference between budgeted and actual operating income.

The fixed cost variance for November is

 A. $5,000 unfavorable.B. $4,000 favorable.C. $4,000 unfavorable.D. $5,000 favorable.

 A. This would be the variable cost flexible budget variance, except that the variable cost flexible variance is favorable,not unfavorable. The question asks for the fixed cost variance, so the variable cost variance is not the correct answer.See the correct answer for a complete explanation.

B. The budgeted amount of fixed cost was less than actual amount of fixed cost incurred. This means that the varianceis unfavorable. See the correct answer for a complete explanation.

C. In this question we are asked to calculate the fixed cost variance. It is simply the difference between the

budgeted fixed costs and the actual fixed costs. For Folsom, the fixed cost variance is $4,000 unfavorable($80,000 − $84,000). The actual fixed cost was greater than the budgeted fixed cost, which means that thevariance is unfavorable.

D. This is the variable cost flexible budget variance, not the fixed cost variance. See the correct answer for a completeexplanation.

Question 76 - CIA 1193 IV.19 - Responsibility Centers and Reporting Segments

The Eastern division sells goods internally to the Western division of the same company. The quoted external price inindustry publications from a supplier near Eastern is $200 per ton plus transportation. It costs $20 per ton to transportthe goods to Western. Eastern's actual market cost per ton to buy the direct materials to make the transferred productis $100. Actual per ton direct labor is $50. Other actual costs of storage and handling are $40. The company presidentselects a $220 transfer price. This is an example of

 A. Negotiated transfer pricing.B. Market-based transfer pricing.C. Cost plus 20% transfer pricing.D. Cost-based transfer pricing.

 A. This is not an example of negotiated transfer pricing. A negotiated transfer price would be one somewhere between

Part 1 : 10/05/14 22:20:20

(c) HOCK international, page 51

Page 52: Hock_Performance Mgmt

8/16/2019 Hock_Performance Mgmt

http://slidepdf.com/reader/full/hockperformance-mgmt 52/152

$190 and $220 that is agreed on by the parties involved.

B. Since the transfer price that has been set is the market price, this is market based transfer pricing.

C. This is not an example of cost plus 20% transfer pricing. A cost plus 20% transfer price would be based on the costof $190.

D. This is not an example of cost-based transfer pricing. A cost-based transfer price would be $190.

Question 77 - CMA 1292 3.22 - Responsibility Centers and Reporting Segments

When using a contribution margin format for internal reporting purposes, the major distinction between segmentmanager performance and segment performance is

 A. Direct variable costs of producing the product.B. Unallocated fixed cost.C. Direct fixed cost controllable by others.D. Direct fixed cost controllable by the segment manager.

 A. The difference between segment manager performance and segment performance is noncontrollable fixed coststhat are traceable. Direct variable costs of production are included in both measures.

B. The difference between segment manager performance and segment performance is noncontrollable fixed coststhat are traceable. Unallocated fixed costs are not used in either measure.

C. The difference between segment manager performance and segment performance is noncontrollable fixedcosts that are traceable. These are fixed costs that are controlled by others.

D. The difference between segment manager performance and segment performance is noncontrollable fixed coststhat are traceable. Direct fixed costs controlled by the segment manager are included in both measures.

Question 78 - CMA 1291 3.1 - Manufacturing Input Variances -- Materials and Labor 

 Arrow Industries employs a standard cost system in which direct materials inventory is carried at standard cost. Arrowhas established the following standards for the prime costs of one unit of product.

 StandardQuantity

StandardPrice

StandardCost

Direct materials 8 pounds$1.80 per pound $14.40Direct labor .25 hours $8.00 per hour 2.00  $16.40

During November, Arrow purchased 160,000 pounds of direct materials at a total cost of $304,000. The total factory

wages for November were $42,000, 90% of which were for direct labor. Arrow manufactured 19,000 units of productduring November using 142,500 pounds of direct materials and 5,000 direct labor hours.

The direct materials purchase price variance for November is:

 A. $14,250 favorable.B. $16,000 favorable.C. $14,250 unfavorable.D. $16,000 unfavorable.

Part 1 : 10/05/14 22:20:20

(c) HOCK international, page 52

Page 53: Hock_Performance Mgmt

8/16/2019 Hock_Performance Mgmt

http://slidepdf.com/reader/full/hockperformance-mgmt 53/152

 A.

The price variance is calculated as follows: (AP − SP) × AQ. This answer is incorrect for two reasons. (1) In thecalculation of the materials purchase price variance, we use the amount of materials purchased, not consumed byproduction. This answer uses the amount consumed. (2) The actual price is greater than the standard price, so thevariance is unfavorable. See the correct answer for a complete explanation.

B. The variance is unfavorable because the actual price is higher than the standard price. See the correct answer for acomplete explanation.

C. In the calculation of the materials purchase price variance we use the amount of materials purchased (160,000),not the amount consumed by production (142,500). See the correct answer for a complete explanation.

D.

The price variance is calculated as follows: (AP − SP) × AQ. The actual price is $1.90 per pound ($304,000 ÷160,000). The standard price is $1.80 per pound. Since the question asks for the direct materials purchaseprice variance, we use the amount of materials actually purchased (160,000 pounds) instead of the amountactually consumed by production (142,500 pounds) as the actual quantity. Thus, the purchase price varianceis ($1.90 − $1.80) × 160,000 = $16,000 unfavorable. The variance is unfavorable because the actual price wasgreater than the standard price.

Question 79 - CMA 695 3.23 - Manufacturing Input Variances -- Materials and Labor 

Blaster Inc., a manufacturer of portable radios, purchases the components from subcontractors to use to assembleinto a complete radio. Each radio requires three units each of Part XBEZ52, which has a standard cost of $1.45 perunit. During May, Blaster experienced the following with respect to Part XBEZ52.  UnitsPurchases ($18,000) 12,000Consumed in manufacturing 10,000Radios manufactured 3,000

During May, Blaster Inc. incurred a purchase price variance of 

 A. $450 favorable.B. $600 unfavorable.C. $450 unfavorable.D. $500 favorable.

 A. This answer results from (1) using the standard quantity allowed for the actual output (9,000 units) instead of theactual quantity purchased (12,000); and (2) misinterpreting the results of that calculation as a favorable variance.

B. The price variance is calculated as follows: (AP − SP) × AQ. The actual price is $1.50 per unit ($18,000 ÷12,000). The standard price is $1.45 per unit, and the actual quantity purchased was 12,000 units. Note that weuse the actual quantity purchased in the formula because we need to determine the purchase price variance.The purchase price variance is $600 unfavorable [($1.50 − $1.45) × 12,000]. The variance is positive becausethe actual price was greater than the standard price, and because this is a cost variance, the variance isunfavorable.

C. The price variance is calculated as follows: (AP − SP) × AQ. This answer results from using the standard quantityallowed for the actual output (9,000 units) instead of the actual quantity purchased (12,000).

D. This answer is not correct for two reasons: (1) it results from using the number of units consumed in manufacturing(10,000) as the Actual Quantity in the variance formula, instead of the number of units purchased (12,000). Becausethis question asks for the purchase price variance, we must use the number of units actually purchased as the Actual

Part 1 : 10/05/14 22:20:20

(c) HOCK international, page 53

Page 54: Hock_Performance Mgmt

8/16/2019 Hock_Performance Mgmt

http://slidepdf.com/reader/full/hockperformance-mgmt 54/152

Quantity. And (2) it misinterprets the answer as favorable when the calculated result of the numbers used is anunfavorable variance.

Question 80 - ICMA 10.P1.138 - Performance Measures

KHD Industries is a multidivisional firm that evaluates its managers based on the return on investment (ROI) earned bytheir divisions. The evaluation and compensation plans use a targeted ROI of 15% (equal to the cost of capital) andmanagers receive a bonus of 5% of basic compensation for every one-percentage point that the division's ROIexceeds 15%. David Evans, manager of the Consumer Products Division, has made aforecast of the division's operations and finances for next year that indicates the ROI would be 24%. In addition, newshort-term programs were identified by the Consumer Products Division and evaluated by the finance staff as follows.Program

Projected ROI

 A 13%B 19%C 22%

D31%

 Assuming no restrictions on expenditures, what is the optimal mix of new programs that would add value to KHDIndustries?

 A. B, C, and D only.B. A, B, C, and D.C. C and D only.D. D only.

A.

Programs B, C and D have projected ROIs that are higher than the firm's 15% cost of capital. Therefore,assuming no restrictions on expenditures, all three of those programs would add value to KHD Industries.

Note that the problem asks for which programs would add value to the company, not which programs wouldmaximize the manager's bonus. If, as the manager projects, the company does earn an ROI equal to 24% onoperations other than these new programs next year, then the addition of B and C would cause the division'sROI to be lower than 24%, because their individual projected ROIs are below 24%. That would, in turn, causethe manager's bonus to be lower than it would be with a 24% ROI. However, since the projected ROIs for Band C are higher than the company's cost of capital, those programs should be undertaken since they will addvalue to the company.

B. Not all of the programs will add value to KHD Industries. Only the programs with projected ROIs that are higher thanthe firm's cost of capital will add value.

C. These are not the only programs that would add value to KHD Industries. All programs with projected ROIs that arehigher than the firm's cost of capital will add value.

D.

D is the only program with a projected ROI that is higher than the manager's projection of the division's ROI excludingthe new programs. Thus, it is the only program that would increase overall ROI for the division above 24% and thusincrease the manager's bonus.

Due to the way the manager's bonus is determined, the manager may be tempted to undertake program D only.However, the question does not ask which program(s) would increase the manager's bonus. It asks which programswould add value to the company as a whole. There are other programs that would add value to the company as a

Part 1 : 10/05/14 22:20:20

(c) HOCK international, page 54

Page 55: Hock_Performance Mgmt

8/16/2019 Hock_Performance Mgmt

http://slidepdf.com/reader/full/hockperformance-mgmt 55/152

whole although they would not increase the manager's bonus.

Question 81 - ICMA 10.P1.103 - Manufacturing Input Variances -- Materials and Labor 

Christopher Akers is the chief executive officer of SBL Inc., a masonry contractor. The financial statements have justarrived showing a $3,000 loss on the new stadium job that was budgeted to show a $6,000 profit. Actual and budgetinformation relating to the materials for the job are as follows.  ActualBudgetBricks - number of bundles 3,000 2,850Bricks - cost per bundle $7.90 $8.00

Which one of the following is a correct statement regarding the stadium job for SBL?

 A. The price variance was favorable by $285.B. the flexible budget variance was unfavorable by $900.C. the price variance was favorable by $300.D. The efficiency variance was unfavorable by $1,185.

 A. The price variance formula used to calculate this was incorrect, because it uses the Standard Quantity instead of the Actual Quantity in the formula.

B. The budget amount in this problem is not a flexible budget amount. It is the budgeted amount for one job, and thequantity specified in the job contract is the quantity that will be supplied. Thus, there is no flexible budget for this job,and if there is no flexible budget, there can be no flexible budget variance.

C. This is a true statement. The Price Variance is (AP — SP) × AQ. This is ($7.90 — $8.00) × 3,000 = ($300)favorable.

D. The efficiency variance formula used to calculate this was incorrect, because it uses the Actual Price instead of theStandard Price in the formula.

Question 82 - CMA 1289 4.2 - Manufacturing Input Variances -- Materials and Labor 

 An unfavorable direct labor efficiency variance could be caused by a(n)

 A. Unfavorable variable overhead spending variance.B. Favorable fixed overhead volume variance.C. Favorable variable overhead spending variance.D. Unfavorable material usage variance.

 A. The variable overhead spending variance is the difference between the actual amount of variable overhead incurredand the standard amount of variable overhead allowed for the actual quantity of the VOH allocation base used for the

actual output produced. There is no connection between this difference and the direct labor usage (efficiency) variance.

B. The fixed overhead volume variance is the difference between the budgeted amount of fixed overhead and theamount of fixed overhead applied (standard rate × standard input for the actual level of output). There is no connectionbetween this difference and the direct labor usage (efficiency) variance.

C. The variable overhead spending variance is the difference between the actual amount of variable overhead incurredand the standard amount of variable overhead allowed for the actual quantity of the VOH allocation base used for theactual output produced. There is no connection between this difference and the direct labor usage (efficiency) variance.

Part 1 : 10/05/14 22:20:20

(c) HOCK international, page 55

Page 56: Hock_Performance Mgmt

8/16/2019 Hock_Performance Mgmt

http://slidepdf.com/reader/full/hockperformance-mgmt 56/152

D.

An unfavorable direct labor efficiency variance means that more time was spent in production than budgeted.A number of reasons could cause this: poor performance of production employees, poor product design,waste, theft, poor material quality, etc.

An unfavorable material usage variance means that more material was spent to produce units of finished

product. Poor material quality could cause an unfavorable material usage variance. The poor quality materialcould also require more time to be spent by production workers to perform their tasks. That is why anunfavorable direct labor efficiency variance could be caused by unfavorable material usage variance.

Question 83 - CMA 697 3.30 - Performance Measures

James Webb is the general manager of the Industrial Product Division, and his performance is measured using theresidual income method. Webb is reviewing the following forecasted information for his division for next year:

Category Amount

(thousands)

Working capital $1,800Revenue 30,000Plant and equipment 17,200

If the imputed interest charge is 15% and Webb wants to achieve a residual income target of $2,000,000, what willcosts have to be in order to achieve the target?

 A. $25,690,000B. $9,000,000C. $10,800,000D. $25,150,000

 A. See the correct answer for a complete explanation.

B. See the correct answer for a complete explanation.

C. See the correct answer for a complete explanation.

D. Residual income is equal to net income before taxes minus target return in dollars (a % of assets orinvested capital). Invested capital is equal to the sum of working capital and plant and equipment or$19,000,000 ($1,800,000 + $17,200,000). Hence, the imputed interest charge is equal to $2,850,000 ($19,000,000× 15%). Net income needs to be equal to the sum of the imputed interest charge and the amount of residualincome, or $4,850,000 ($2,850,000 + $2,000,000). Thus, costs are equal to the revenue minus net income or$25,150,000 (30,000,000 − $4,850,000).

Question 84 - CIA 1186 IV.16 - Responsibility Centers and Reporting Segments

The data available for the current year are given below.

 Whole

companyDivision 1Division 2Variable manufacturing costs $400,000 $220,000 $180,000Unallocated costs (e.g., president's salary) 100,000Fixed costs controllable by division managers(e.g., advertising, engineering supervision costs) 90,000 50,000 40,000

Part 1 : 10/05/14 22:20:20

(c) HOCK international, page 56

Page 57: Hock_Performance Mgmt

8/16/2019 Hock_Performance Mgmt

http://slidepdf.com/reader/full/hockperformance-mgmt 57/152

Net revenue 1,000,000 600,000 400,000Variable selling and administrative costs 130,000 70,000 60,000Fixed costs controllable by others(e.g., depreciation, insurance) 120,000 70,000 50,000

Based upon the information presented above, the contribution margin for the company was:

 A. $400,000B. $530,000C. $470,000D. $600,000

 A. This is the variable manufacturing costs. The contribution margin is sales minus variable costs.

B. This is the total variable costs. The contribution margin is sales minus variable costs.

C. Contribution margin is sales revenue minus variable costs. Net revenue was $1,000,000, variablemanufacturing costs were $400,000, and variable selling and administrative costs were $130,000. $1,000,000 −$400,000 − $130,000 = $470,000.

D. This is revenues minus variable manufacturing costs. The contribution margin is sales minus all variable costs, notust variable manufacturing costs.

Question 85 - CMA 684 4.26 - Manufacturing Input Variances -- Materials and Labor 

Each unit of Product XK-46 requires three direct labor hours. Employee benefit costs are treated as direct labor costs.Data on direct labor are

Number of direct employees: 25

Weekly productive hours per employee: 35

Estimated weekly wages per employee: $245

Employee benefits (related to weekly wages): 25%

The standard direct labor cost per unit of Product XK-46 is

 A. $29.40.B. $21.00.C. $36.75.D. $26.25.

 A. This answer results from calculating employee benefit costs as 40% of wages instead of 25%.

B. Employee benefit costs are treated as direct labor costs. We therefore need to include this amount in calculation ofthe standard direct labor cost per unit.

C. There are 35 weekly productive hours per employee, not 25 as was used in this calculation.

D. The estimated weekly costs (wages plus employee benefits) per employee are $306.25 [($245 + ($245 ×25%)]. The hourly labor cost is $8.75 ($306.25 ÷ 35 hours per week per employee). Three direct labor hours arerequired to produce one unit of product XK-46. Thus, the unit labor cost is $26.25 ($8.75 × 3).

Question 86 - ICMA 10.P1.134 - Performance Measures

Part 1 : 10/05/14 22:20:20

(c) HOCK international, page 57

Page 58: Hock_Performance Mgmt

8/16/2019 Hock_Performance Mgmt

http://slidepdf.com/reader/full/hockperformance-mgmt 58/152

P.C. Programs Inc. produces software for individual users and small businesses. Rita Morgan manages the customerhot line department for the firm and is responsible for answering customer questions related to software productsproduced by all divisions of the firm. For purposes of promoting goal congruence, which one of the following would bethe least appropriate measure of her performance?

 A. Average time to provide an answer or solution to a customer.

B. Number of calls to the hot line for each new release of software.C. Number of customer complaints due to incorrect responses given to customers.D. Average time a customer is on hold.

 A. The hot line manager's responsibility is to see that customers are assisted with problems by receiving the correctanswer quickly. Average time to provide an answer or solution to a customer is something the manager of the hot linedepartment can control by properly staffing and training the department employees. Therefore, this would be anappropriate measure of the hot line manager's performance.

B. The hot line manager's responsibility is to see that customers are assisted with problems by receiving thecorrect answer quickly. Number of calls to the hot line for a new release of software is not something themanager of the hot line department can control. It is determined by how "buggy" a new release is, and that isthe responsibility of the software developers. Therefore, this would not be an appropriate measure of the hotline manager's performance.

C. The hot line manager's responsibility is to see that customers are assisted with problems by receiving the correctanswer quickly. The number of incorrect responses given to customers is something the manager of the hot linedepartment can control by properly staffing and training the department employees. Therefore, this would be anappropriate measure of the hot line manager's performance.

D. The hot line manager's responsibility is to see that customers are assisted with problems by receiving the correctanswer quickly. Average time a customer is on hold is something the manager of the hot line department can controlby properly staffing and training the department employees. Therefore, this would be an appropriate measure of the hotline manager's performance.

Question 87 - CMA 1293 3.21 - Responsibility Centers and Reporting Segments

 A successful responsibility accounting reporting system is dependent upon

 A. The correct allocation of controllable variable costs.B. A reasonable separation of costs into their fixed and variable components since fixed costs are not controllable andmust be eliminated from the responsibility report.C. Identification of the management level at which all costs are controllable.D. The proper delegation of responsibility and authority.

 A. There are some fixed costs that wil l also be controllable and therefore should also be included on the responsibilityreport.

B. Some fixed costs are controllable by the different managers so they should be included in the responsibility report.See the correct answer for a complete explanation.

C. Not only is the control of a cost relevant in responsibility accounting, but in some cases the responsibility for the costbeing incurred may also be relevant to the allocation of costs for the responsibility report. For example, if a managermakes a decision that causes additional costs to be incurred in a different department, those additional costs should be'allocated' to the manager who caused them to be incurred, even if that manager is unable to control the coststhemselves. Therefore, costs are not controlled at any specific management level but by individual managers' actions.

D. The identification of who is responsible for the costs being incurred and the control over the coststhemselves is critical to the effectiveness of a responsibility accounting system. This can be done through the

Part 1 : 10/05/14 22:20:20

(c) HOCK international, page 58

Page 59: Hock_Performance Mgmt

8/16/2019 Hock_Performance Mgmt

http://slidepdf.com/reader/full/hockperformance-mgmt 59/152

proper delegation of responsibility and authority.

Question 88 - CIA 1189 IV.18 - Manufacturing Input Variances -- Materials and Labor 

One of the items produced by a manufacturer of lawn and garden tools is a chain saw. The direct labor standard forassembling and testing a chain saw is 2.5 hours at $8 per hour. Budgeted production for October was 1,200 units. Actual production during the month was 1,000 units, and direct labor cost was $27,840 for 3,200 hours. What is thedirect labor efficiency variance?

 A. $2,240 unfavorable.B. $6,090 favorable.C. $5,600 favorable.D. $5,600 unfavorable.

 A. This is the labor rate variance. The question asks for the labor efficiency variance.

B. The labor efficiency variance is calculated as: (Actual Hours − Standard Hours for Actual Output) × Standard Rate.This answer results from using the actual rate instead of the standard rate. In addition, the variance is unfavorable

because the actual hours used were greater than the standard hours for the actual output.

C. The hours spent on production were greater than the hours allowed for the actual level of production, which createsan unfavorable labor efficiency variance.

D. The labor efficiency variance is calculated as: (Actual Hours − Standard Hours for Actual Output) ×Standard Rate. Actual hours are 3,200. The standard hours allowed for actual output are 2,500 hr. (2.5 hoursper unit × 1000 units actually produced). The standard rate is $8. Therefore, the direct labor efficiency varianceis: (3,200 − 2,500) × $8 = $5,600 unfavorable. The hours spent on production were greater than the hoursallowed for the actual level of production, which gave us an unfavorable labor efficiency variance.

Question 89 - ICMA 10.P1.107 - Manufacturing Input Variances -- Materials and Labor 

Randall Company uses standard costing and flexible budgeting and is evaluating its direct labor. The total budgetvariance can usually be broken down into two other variances identified as the

 A. direct labor cost variance and the direct labor volume variance.B. direct labor cost variance and direct labor efficiency variance.C. direct labor rate variance and direct labor volume variance.D. direct labor rate variance and direct labor efficiency variance.

 A. There are no variances called "direct labor cost variance" and "direct labor volume variance."

B. There is no variance called "direct labor cost variance."

C. There is no variance called "direct labor volume variance."

D. The direct labor rate variance is the price variance for labor, while the direct labor efficiency variance is thequantity variance for labor.

Question 90 - CMA 692 3.23 - Responsibility Centers and Reporting Segments

Part 1 : 10/05/14 22:20:20

(c) HOCK international, page 59

Page 60: Hock_Performance Mgmt

8/16/2019 Hock_Performance Mgmt

http://slidepdf.com/reader/full/hockperformance-mgmt 60/152

The WK Company uses a performance reporting system that reflects the company's decentralization of decisionmaking. The departmental performance report shows one line of data for each subordinate who reports to the groupvice president. The data presented show the actual costs incurred during the period, the budgeted costs, and allvariances from budget for that subordinate's department. The WK Company is using a system called

 A. Responsibility accounting.B. Flexible budgeting.

C. Program budgeting.D. Cost-benefit accounting.

A. Responsibility accounting is a system in which costs are allocated to managers and/or departments basedon who is responsible for the incurrence of the costs. This is the method described in the question.

B. Flexible budgeting is the adaptation of variable items in the static budget to reflect the actual level of activityachieved during the period. This is not what is described in the question.

C. This is a budgeting system and it does not describe the situation in the question.

D. Cost-benefit accounting is not an accounting method or a reporting system.

Question 91 - ICMA 10.P1.123 - Responsibility Centers and Reporting Segments

Which one of the following is an incorrect description of transfer pricing?

 A. It measures exchanges between a company and external customers.B. If no market price exists, the transfer price may be based on cost.C. It measures the value of goods or services furnished by a profit center to other responsibility centers within acompany.D. If a market price exists, that price may be used as a transfer price.

A. A transfer price is not a price charged by the company to external customers, so this is an incorrect

description of transfer pricing. A transfer price is the price charged by one unit of the company to another unitof the same company for the services or goods produced by the first unit and "sold" to the second unit.

B. When there is an external market for the product, this is almost always the best transfer price to use for profitabilityand performance measurement, because it is objective. However, if no market price exists, a transfer price may bebased on cost, such as the cost of production plus an opportunity cost, the variable cost, the full cost, or cost plus. Anegotiated price or arbitrary pricing may also be used.

C. A transfer price is the price charged by one unit of the company to another unit of the same company for theservices or goods produced by the first unit and "sold" to the second unit.

D. The transfer price is set as the current price of the selling division’s product in an arms-length transaction. Whenthere is an external market for the product, this is almost always the best transfer price to use for profitability andperformance measurement, because it is objective. It satisfies the "arm’s length" requirement by taxing authorities.

Furthermore, it satisfies the management of the buying company that they are paying a fair price for the goods and themanagement of the selling company that they are receiving a fair price for the goods.

Question 92 - ICMA 10.P1.126 - Responsibility Centers and Reporting Segments

Morrison's Plastics Division, a profit center, sells its products to external customers as well as to other internal profitcenters. Which one of the following circumstances would justify the Plastics Division selling a product internally to

Part 1 : 10/05/14 22:20:20

(c) HOCK international, page 60

Page 61: Hock_Performance Mgmt

8/16/2019 Hock_Performance Mgmt

http://slidepdf.com/reader/full/hockperformance-mgmt 61/152

another profit center at a price that is below the market-based transfer price?

 A. The profit centers' managers are evaluated on the basis of unit operating income.B. Routine sales commissions and collection costs would be avoided.C. The buying unit has excess capacity.D. The selling unit is operating at full capacity.

 A. This is a good reason to negotiate the transfer price so that it will be acceptable to both managers. But this by itselfdoes not justify the Plastics Division selling a product internally to another profit center at a price that is below themarket-based transfer price, or at any particular transfer price, for that matter.

B. If selling the product internally enables the selling division to avoid paying sales commissions andcollection costs that it would have to pay to sell outside, then it is justified to deduct an amount up to theamount of the saved routine sales and collection costs from the market price to establish the transfer price.The selling division will not be hurt by doing that, and the buying division can benefit from a lower price. Thecompany as a whole will benefit from the saved sales commissions and collection costs.

C. If the selling unit had excess capacity, that would justify selling the product internally at a price that is below themarket-based transfer price. But the fact that the buying unit has excess capacity is not relevant.

D. When the selling unit is operating at full capacity, it would have to not produce something that could have been soldto an outside customer at the market price. The selling division will therefore have an additional opportunity cost if itproduces the product for the internal division. This would not justify selling the product internally at a price that is belowthe market-based transfer price.

Question 93 - CMA 694 3.30 - Responsibility Centers and Reporting Segments

 An appropriate transfer price between two divisions of The Stark Company can be determined from the following data:Fabricating Division  

Market price of subassembly $50Variable cost of subassembly $20

Excess capacity (in units) 1,000Assembling Division  Number of units needed 900

What is the natural bargaining range for the two divisions?

 A. Any amount less than $50.B. $50 is the only acceptable price.C. Between $20 and $50.D. Between $50 and $70.

 A. The transfer price should not be lower than the variable costs of production, even if there is excess capacity. As longas selling price covers the variable costs and even a very small amount of fixed costs in case of excess capacity, it isbeneficial to the company.

B. An acceptable price is within a range, and $50 is not the only acceptable price.

C. The basic issue of transfer prices is simply how much should one unit of a company charge another unit ofthe same company for its goods or services. The goal in setting a transfer price is that the method used willstimulate both the buying and selling department managers to do what will provide the greatest benefit to thecompany as a whole, rather than to act in their own interest. When there is an external market for the product,market price is almost always the best transfer price to use. Thus, market price is at the maximum of thenatural range. When the company has idle capacity, the variable cost approach to determining the transfer

Part 1 : 10/05/14 22:20:20

(c) HOCK international, page 61

Page 62: Hock_Performance Mgmt

8/16/2019 Hock_Performance Mgmt

http://slidepdf.com/reader/full/hockperformance-mgmt 62/152

price also works well. Since the Fabricating Division has enough capacity to fulfill the demand of theAssembling Division without any over-time, the variable cost approach is also acceptable and is at theminimum of the natural range.

D. It would subvert overall profit goals of the company to choose a transfer price above the free market level.

Question 94 - CMA 696 3.25 - Manufacturing Input Variances -- Overhead

 Ardmore Enterprises uses a standard cost system in its small appliance division. The standard cost of manufacturingone unit of Zeb is as follows:Materials - 60 pounds at $1.50 per pound $ 90Labor - 3 hours at $12 per hour 36Factory overhead - 3 hours at $8 per hour 24Total standard cost per unit $150

The budgeted variable factory overhead rate is $3 per labor hour, and the budgeted fixed factory overhead is $27,000per month. During May, Ardmore produced 1,650 units of Zeb compared with a normal capacity of 1,800 units. The

actual cost per unit was as follows:Materials (purchased and used) - 58 pounds at $1.65 per pound $ 95.70Labor - 3.1 hours at $12 per hour 37.20Factory overhead - $39,930 per 1,650 units 24.20Total actual cost per unit $157.10

The flexible budget overhead variance for May is

 A. $3,270 unfavorable.B. $1,920 favorable.C. $1,920 unfavorable.D. $3,270 favorable.

 A. The flexible budget overhead variance is favorable. The budgeted overhead is greater than the actual, which means

the variance is favorable. See the correct answer for a complete explanation.

B.

The flexible budget overhead variance equals the difference between the total actual overhead incurred andthe flexible budget total overhead (variable and fixed).

The flexible budget fixed overhead equals the master budget amount of $27,000. The budgeted variablefactory overhead rate is $3 per labor hour, the standard hours to produce one unit of product is 3 hours, and1,650 units were produced. Thus, the flexible budget variable factory overhead was $14,850.

The actual overhead costs were $39,930. The total flexible budget variable factory overhead is $41,850($14,850 + $27,000). Therefore, the flexible budget overhead variance is ($1,920) favorable ($39,930 − $41,850).

Since the actual overhead is less than the budgeted overhead, the variance is favorable.

C. The flexible budget overhead variance is favorable. The budgeted overhead is greater than actual which means thevariance is favorable. See the correct answer for a complete explanation.

D. This is the flexible budget variance based on the master budget level of output of 1,800 units. Actual factoryoverhead of $39,930 − ([$3 × 3 × 1,800] + $27,000) budgeted fixed overhead = ($3,270) favorable. However, the actuallevel of production of 1,650 units should be used to calculate flexible budget amounts for variable costs, not the masterbudget level. See the correct answer for a complete explanation.

Part 1 : 10/05/14 22:20:20

(c) HOCK international, page 62

Page 63: Hock_Performance Mgmt

8/16/2019 Hock_Performance Mgmt

http://slidepdf.com/reader/full/hockperformance-mgmt 63/152

Page 64: Hock_Performance Mgmt

8/16/2019 Hock_Performance Mgmt

http://slidepdf.com/reader/full/hockperformance-mgmt 64/152

 A. Contribution margin reporting breaks costs down into fixed and variable costs. While this may be used inconjunction with responsibility accounting, this by itself is not describing the method used by the company.

B. Responsibility accounting is a system in which cost and revenue data is reported based on who (manageror division) is able to control them or is responsible for them. This is the system described in the question.

C. Segment reporting is the reporting of results by segment: product line, geography or some other distinguishingcharacteristic. This is not descriptive of the method used by this company.

D. Absorption cost accounting relates to the allocation of fixed costs. This is not descriptive of the method used by thiscompany.

Question 97 - CIA 1193 IV.21 - Responsibility Centers and Reporting Segments

 A and B are autonomous divisions of a corporation. They have no beginning or ending inventories, and the number ofunits produced is equal to the number of units sold. Following is financial information relating to the two divisions:  D i v i s i o n

  A BSales $150,000$400,000Other revenue 10,000 15,000Direct materials 30,000 65,000Direct labor 20,000 40,000Variable factory overhead 5,000 15,000Fixed factory overhead 25,000 55,000Variable selling and administrative expense 15,000 30,000Fixed sell ing and administrative expense 35,000 60,000Central corporate expenses (allocated) 12,000 20,000

What is the contribution margin of Division B?

 A. $265,000B. $150,000C. $235,000D. $205,000

A. The contribution margin of Department B is equal to the total revenues of Department B less total variablecosts of Department B. It is calculated as follows: $400,000 + $15,000 − $65,000 − $40,000 − $15,000 − 30,000 =$265,000.

B. This answer assumes that fixed costs are also deductible. However, contribution margin is calculated as totalrevenues minus total variable costs.

C. The contribution margin of Department B is equal to the total revenues of Department B less total variable costs of

Department B.

D. The contribution margin of Department B is equal to the total revenues of Department B minus the total variablecosts of Department B.

Question 98 - ICMA 10.P1.090 - Variance Analysis Concepts

Part 1 : 10/05/14 22:20:20

(c) HOCK international, page 64

Page 65: Hock_Performance Mgmt

8/16/2019 Hock_Performance Mgmt

http://slidepdf.com/reader/full/hockperformance-mgmt 65/152

The following performance report was prepared for Dale Manufacturing for the month of April.

  ActualResults

StaticBudget Variance

 Sales units 100,000 80,000 20,000 F Sales dollars $190,000 $160,000 $30,000 F Variable costs 125,000 96,000 29,000 U

 Fixed costs 45,000 40,000 5,000 U  -------------- ------------- ------------ Operating income $20,000 $24,000 $4,000 U

Using a flexible budget, Dale's total sales-volume variance is

 A. $16,000 favorable.B. $4,000 unfavorable.C. $20,000 unfavorable.D. $6,000 favorable.

A. The sales volume variance is the flexible budget amount minus the static budget amount. A sales volumevariance can be calculated for every line on an income statement. The total sales volume variance is the

flexible budget operating income minus the static budget operating income. In this problem, the flexiblebudget amounts are not given and must be calculated. The flexible budget for sales dollars is the staticbudget amount of $160,000 divided by the static budget sales units of 80,000 and multiplied by the actualsales units of 100,000, which is $200,000. The flexible budget for variable costs is calculated the same way andis $120,000. The fixed cost in the flexible budget is the same as the fixed cost in the static budget: $40,000.The flexible budget operating income is therefore $200,000 − $120,000 − $40,000, which equals $40,000. Thesales volume variance is the flexible budget operating income of $40,000 minus the static budget operatingincome of $24,000, which is $16,000. Because the variance is positive on a net income line, it is a favorablevariance.

B. This is the total static budget variance.

C. This is the total flexible budget variance, or actual operating income minus flexible budget operating income.

D. This answer results from adjusting the fixed costs in the static budget to a flexible budget amount that reflects thedifference between the static budget sales units and the actual sales units. However, fixed costs do not change withchanges in sales or manufacturing volume, and so the fixed costs in the flexible budget should be the same as thefixed costs in the static budget.

Question 99 - CIA 1192 IV.21 - Manufacturing Input Variances -- Materials and Labor 

 A manufacturer has the following direct materials standard for one of its products.

Direct materials: 3 pounds @ $1.60/pound = $4.80

The company records all inventory at standard cost. Data for the current period regarding the manufacturer's budgetedand actual production for the product as well as direct materials purchases and issues to production for manufacture ofthe product are presented as follows.

Budgeted production for the period: 8,000 units

 Actual production for the period: 7,500 units

Direct materials purchases:

Pounds purchased: 25,000 pounds

Total cost: $38,750

Part 1 : 10/05/14 22:20:20

(c) HOCK international, page 65

Page 66: Hock_Performance Mgmt

8/16/2019 Hock_Performance Mgmt

http://slidepdf.com/reader/full/hockperformance-mgmt 66/152

Page 67: Hock_Performance Mgmt

8/16/2019 Hock_Performance Mgmt

http://slidepdf.com/reader/full/hockperformance-mgmt 67/152

efficiency variance will be:

 A. Zero.B. A function of the direct labor efficiency variance.C. Favorable, if output exceeds the budgeted level.D. Unfavorable, if output is less than the budgeted level.

A.

The formula for the variable overhead efficiency variance is: (Actual Activity Level of VOH Application Baseactually used − Standard Activity Level of application base allowed for actual output) × Standard ApplicationRate.

In standard costing, overhead is applied on the basis of some activity such as machine hours or direct laborhours. It is applied on the basis of the amount of that activity (machine hours, direct labor hours) that isallowed according to the standard for the actual output, not the actual amount of hours used.

When normal costing is being used, overhead is applied instead on the basis of units of the activity baseactually used for the actual output. When overhead is applied on that basis, overhead is applied on the basisof actual activity, not the standard activity allowed.

Therefore, if overhead is applied on the basis of units of the activity base actually used for the actual output,there will be no difference between the actual activity level (the first number in the parentheses in the formula)and the activity level used to apply the overhead (the second number in the parentheses in the formula), sothe variance will be equal to zero.

B.

The formula for the variable overhead efficiency variance is: (Actual Activity Level of VOH Application Base actuallyused − Standard Activity Level of application base allowed for actual output) × Standard Application Rate.

If overhead is applied on the basis of actual units of output, there will be no difference between the actual activity level(the first number in the parentheses in the formula) and the activity level used to apply the overhead (the secondnumber in the parentheses in the formula). The direct labor efficiency variance is (Actual Quantity of Direct Labor

Hours Used − Standard Quantity of Direct Labor Hours Allowed for the Actual Output) × Standard Labor Rate per Hour.The actual quantity and the standard quantity of direct labor hours are different amounts, so there will be no connectionbetween the direct labor efficiency variance and the variable overhead efficiency variance.

See the correct answer for a more complete explanation.

C.

The formula for the variable overhead efficiency variance is: (Actual Activity Level of VOH Application Base actuallyused − Standard Activity Level of application base allowed for actual output) × Standard Application Rate.

If overhead is applied on the basis of actual units of output, there will be no difference between the actual activity level(the first number in the parentheses in the formula) and the activity level used to apply the overhead (the secondnumber in the parentheses in the formula), so the variance will be neither favorable nor unfavorable.

D.

The formula for the variable overhead efficiency variance is: (Actual Activity Level of VOH Application Base actuallyused − Standard Activity Level of application base allowed for actual output) × Standard Application Rate.

If overhead is applied on the basis of actual units of output, there will be no difference between the actual activity level(the first number in the parentheses in the formula) and the activity level used to apply the overhead (the secondnumber in the parentheses in the formula), so the variance will be neither favorable nor unfavorable.

Part 1 : 10/05/14 22:20:20

(c) HOCK international, page 67

Page 68: Hock_Performance Mgmt

8/16/2019 Hock_Performance Mgmt

http://slidepdf.com/reader/full/hockperformance-mgmt 68/152

Question 102 - CMA 1295 3.7 - Manufacturing Input Variances -- Overhead

The variance in an absorption costing system that measures the departure from the denominator level of activity thatwas used to set the fixed overhead rate is the

 A. Flexible budget variance.B. Production volume variance.C. Efficiency variance.D. Spending variance.

 A. The flexible budget variance is the difference between the actual results and the flexible budget amounts.

B. In the budgeting process, the company must determine the level of activity to use. This level of activity isalso called the denominator level. The fixed overhead production volume variance is caused by the actualproduction level being different from the production level that was used to calculate the budgeted fixedoverhead rate.

C. There is no fixed overhead efficiency variance. It does not exist.

D. The fixed overhead spending variance is the difference between actual fixed costs and budgeted fixed costs.

Question 103 - CMA 696 3.27 - Responsibility Centers and Reporting Segments

Parkside Inc. has several divisions that operate as decentralized profit centers. Parkside's Entertainment Divisionmanufactures video arcade equipment using the products of two of Parkside's other divisions. The Plastics Divisionmanufactures plastic components, one type that is made exclusively for the Entertainment Division, while other lesscomplex components are sold to outside markets. The products of the Video Cards Division are sold in a competitivemarket; however, one video card model is also used by the Entertainment Division. The actual costs per unit used bythe Entertainment Division are presented below.

  PlasticComponents

VideoCards

Direct material $1.25 $2.40Direct labor 2.35 3.00Variable overhead 1.00 1.50Fixed overhead .40 2.25Total cost $5.00 $9.15

The Plastics Division sells its commercial products at full cost plus a 25% markup and believes the proprietary plasticcomponent made for the Entertainment Division would sell for $6.25 per unit on the open market. The market price ofthe video card used by the Entertainment Division is $10.98 per unit.

 Assume that the Entertainment Division is able to purchase a large quantity of video cards from an outside source at

$8.70 per unit. The Video Cards Division, having excess capacity, agrees to lower its transfer price to $8.70 per unit.This action would

 A. Allow evaluation of both divisions on the same basis.B. Optimize the profit goals of the Entertainment Division while subverting the profit goals of Parkside Inc.C. Optimize the overall profit goals of Parkside Inc.D. Subvert the profit goals of the Video Cards Division while optimizing the profit goals of the Entertainment Division.

 A. A transfer price of $8.70 per unit is less than the Video Cards Division's total cost of production, whereas the price isthe Entertainment Division's market price. This would not allow the evaluation of both divisions on the same basis.

Part 1 : 10/05/14 22:20:20

(c) HOCK international, page 68

Page 69: Hock_Performance Mgmt

8/16/2019 Hock_Performance Mgmt

http://slidepdf.com/reader/full/hockperformance-mgmt 69/152

B. The Entertainment Division's profit goals are not optimized because the Entertainment Division is indifferent as towhere to buy the video cards, since the external price is the same as the internal price. The profit goals of ParksideInc. would not be subverted because this action would be congruent with overall profit goals of the company. Thesuggested discounted price is above the variable cost of production for the Video Cards Division; thus, the use of idlecapacity enhances profits.

C. This action is goal congruent. The Entertainment Division is indifferent as to where to buy video cards,

since the external price is the same as the internal price. The overall profit goals of Parkside Inc. would beoptimized because the variable cost of production of the Video Cards division is less than the suggesteddiscounted price. Thus, the use of idle capacity enhances profits.

D. The profit goals of the Video Cards Division would be subverted in this case. However, the Entertainment Division'sprofit goals are not optimized because the Entertainment Division is indifferent as to where to buy the video cards,since the external price is the same as the internal price.

Question 104 - CMA 695 3.29 - Sales and Market Variances

For a company that produces more than one product, the sales volume variance can be divided into which two of thefollowing additional variances?

 A. Sales mix variance and sales price variance.B. Sales price variance and flexible budget variance.C. Sales mix variance and production volume variance.D. Sales quantity variance and sales mix variance.

 A. The sales price variance, which is equivalent to the flexible budget variance, is not a component of the sales volumevariance.

B. Neither the sales price variance nor the flexible budget variance are components of the sales volume variance. Thesales price variance is equivalent to the flexible budget variance. The sales price variance and the sales volumevariance combined equal the static budget variance. The flexible budget variance, which is equivalent to the sales pricevariance, is the difference between the actual results and the flexible budget amounts.

C. The production volume variance is a fixed manufacturing overhead variance. It is not a component of the salesvolume variance.

D. The sales volume variance measures the impact of the difference in units sold between actual andbudgeted. When there is more than one product, this total volume variance will then be broken down into twosub-variances – the sales quantity variance and the sales mix variance. The sales mix variance determines theimpact on the total sales volume variance made by the variance between the actual mix of products sold andthe planned mix of products to be sold. The sales quantity variance determines the impact on the total salesvolume variance made by the variance between the actual total quantity of products sold and the planned totalquantity of products sold.

Question 105 - CIA 594 III.73 - Manufacturing Input Variances -- Materials and Labor 

 A company manufactures one product and has a standard cost system. In April the company had the followingexperience:  Direct MaterialsDirect Labor  Actual $/unit of input (lbs. & hrs.) $28 $18Standard price/unit of input $24 $20Standard inputs allowed per unit of output 10 4

Part 1 : 10/05/14 22:20:20

(c) HOCK international, page 69

Page 70: Hock_Performance Mgmt

8/16/2019 Hock_Performance Mgmt

http://slidepdf.com/reader/full/hockperformance-mgmt 70/152

 Actual units of input 190,000 78,000 Actual units of output 20,000 20,000

The direct materials efficiency variance for April is:

 A. $240,000 favorable.B. $760,000 unfavorable.

C. $240,000 unfavorable.D. $156,000 favorable.

A. The quantity variance (also called the efficiency or usage variance) is calculated as: (Actual Quantity −Standard Quantity for Actual Output) × Standard Price. The actual quantity is 190,000. The standard quantityallowed for the output of 20,000 units of product is 200,000 lb. (20,000 × 10 lb. standard quantity of material perunit of finished product). The standard price is $24. The direct materials efficiency variance is (190,000 −200,000) × $24 = $(240,000) favorable. The variance is favorable because the actual quantity used inproduction is less than the standard quantity allowed for the actual output.

B. This is the direct materials price variance. However, the question asks for the direct materials efficiency variance.See the correct answer for a complete explanation.

C. The direct materials efficiency variance is favorable because the actual quantity used in production is less than thestandard quantity allowed for the actual output. See the correct answer for a complete explanation.

D. This is the direct labor rate variance. The question asks for the direct materials efficiency variance. See the correctanswer for a complete explanation.

Question 106 - CMA 1294 3.29 - Manufacturing Input Variances -- Materials and Labor 

Water Control Inc. manufactures water pumps and uses a standard cost system. The standard factory overhead costsper water pump are based on direct labor hours and are as follows:

Variable overhead (4 hours at $8/hour) - $32Fixed overhead (4 hours at $5/hour*) - $20Total overhead cost per unit - $52* Based on a capacity of 100,000 direct labor hours per month.

The following additional information is available for the month of November:

22,000 pumps were produced although 25,000 had been scheduled for production.94,000 direct labor hours were worked at a total cost of $940,000.The standard direct labor rate is $9 per hour.The standard direct labor time per unit is 4 hours.Variable overhead costs were $740,000.Fixed overhead costs were $540,000.

The direct labor price variance for November was

 A. $60,000 favorable.B. $54,000 unfavorable.C. $148,000 unfavorable.D. $94,000 unfavorable.

 A.

This is not the correct answer. See the correct answer for a complete explanation.

Part 1 : 10/05/14 22:20:20

(c) HOCK international, page 70

Page 71: Hock_Performance Mgmt

8/16/2019 Hock_Performance Mgmt

http://slidepdf.com/reader/full/hockperformance-mgmt 71/152

Page 72: Hock_Performance Mgmt

8/16/2019 Hock_Performance Mgmt

http://slidepdf.com/reader/full/hockperformance-mgmt 72/152

C. Yield variance and mix variance.D. Volume variance and mix variance.

 A. The total material quantity (efficiency) and labor efficiency variances can be broken down into the two subvariances,but the spending variance is not one of them.

B. The total material quantity (efficiency) and labor efficiency variances can be broken down into the two subvariances,

but the price variance is not one of them.

C. The total material quantity (efficiency) and labor efficiency variances can be broken down into twosubvariances: the mix and the yield variances. The mix variance is the part of the quantity variance thatresults because the mix of material actually used was different from the mix that was supposed to have beenused. (For example, including more corn and less wheat in the cereal than the standard called for). The yieldvariance results from the difference between the total quantity of the inputs that were actually used toproduce the actual output and the total standard quantity that should have been used to produce the actualoutput.

D. The total material quantity (efficiency) and labor efficiency variances can be broken down into the two subvariances,but the volume variance is not one of them.

Question 109 - CMA 1291 3.10 - Responsibility Centers and Reporting Segments

 A segment of an organization is referred to as a service center if it has

 A. Responsibility for combining the raw materials, direct labor, and other factors of production into a final output.B. Authority to make decisions affecting the major determinants of profit including the power to choose its markets andsources of supply.C. Responsibility for developing markets and selling the output of the organization.D. Authority to provide specialized support to other units within the organization.

 A. This is the definition of a production center, not a service center. See the correct answer for a complete explanation.

B. This is the definition of a profit center, not a service center. See the correct answer for a complete explanation.

C. This is the definition of a revenue center, not a service center. See the correct answer for a complete explanation.

D. A service center provides specialized support services to other departments of the organization.

Question 110 - ICMA 10.P1.121 - Responsibility Centers and Reporting Segments

Most firms allocate corporate and other support costs to divisions and departments for all of the following reasonsexcept to

 A. fix accountability and evaluate profit centers.B. remind profit center managers that earnings must be adequate to cover some share of the indirect costs.C. create competition between divisions and departments and their managers.D. stimulate profit center managers to put pressure on central managers to control service costs.

 A. Allocating internal service and support costs to divisions and departments that are profit centers makes it possible tosee whether profit centers are generating adequate profits and makes them accountable for their performance.

B. Internal allocation of support costs is done because if a company calculates its cost of production but does not

Part 1 : 10/05/14 22:20:20

(c) HOCK international, page 72

Page 73: Hock_Performance Mgmt

8/16/2019 Hock_Performance Mgmt

http://slidepdf.com/reader/full/hockperformance-mgmt 73/152

include the costs of its service departments, it will calculate a cost of production that is less than the actual total cost. As a result of this incorrect calculation, the company’s pricing decisions will not be correct, and in a worst-casescenario, the company may sell the product for less than it actually costs to produce it. Allocating service costs todivisions and departments reminds profit center managers that earnings must be adequate to cover the indirect costs,and allocating service costs provides the needed information to those making pricing decisions.

C. A spirit of competition can be very motivating and result in achievement of company goals, so this is a reason to

allocate corporate and support costs to divisions and departments.

D. This is not a reason to allocate service and support costs to divisions and departments. The managers ofthe service departments should not be held accountable to the user departments to control their costs butrather to top management.

Question 111 - CIA 1191 IV.16 - Manufacturing Input Variances -- Materials and Labor 

 A company producing a single product employs the following direct material cost standard for each unit of output:

3 pounds of material x $4 per pound = $12 per output unit

Data regarding the operations for the current month are as follows:Planned production 26,000 units Actual production 23,000 units Actual purchases of direct materials (75,000 pounds) $297,000Direct materials used in Production 70,000 pounds

What would be the amount of the direct materials purchase price variance and direct materials quantity variance that the company would recognize for the month?

 A. Price variance - $3,000 favorable / Quantity variance - $4,000 unfavorableB. Price variance - $3,000 favorable / Quantity variance - $24,000 unfavorableC. Price variance - $3,120 favorable / Quantity variance - $32,000 favorableD. Price variance - $2,800 favorable / Quantity variance - $4,000 unfavorable

A.

The purchase price variance is calculated as follows: (Actual Price − Standard Price) × Actual Quantity. Sincethe purchase price variance is required, the price to use for the actual price is the price per unit of the unitspurchased instead of the price per unit of the units used; and the actual quantity is the number of unitspurchased, not the number of units that were put into production. The actual price is $3.96 per pound($297,000 ÷ 75,000). The standard price is $4.00 per pound. The actual quantity purchased is 75,000 pounds.The purchase price variance is ($3.96 − $4.00) × 75,000 = $(3,000) favorable.

The quantity variance is calculated as follows: (Actual Quantity − Standard Quantity for Actual Output ) ×Standard Price. The actual quantity is the actual quantity used, not the quantity purchased. The standardquantity is the standard quantity allowed for the actual output. The actual quantity is 70,000. The standard

quantity allowed for the actual level of output is 69,000 pounds (3 lb.× 23,000 units). The standard price is $4.Hence, the quantity variance is (70,000 − 69,000) × $4 = $4,000 unfavorable.

B.

The quantity variance is incorrect because it is calculated as (the actual quantity purchased of 75,000 minus the actualquantity used of 70,000) multiplied by the standard price per unit of $4.

The correct formula to use for the quantity variance is (Actual Quantity − Standard Quantity for Actual Output ) ×Standard Price. The actual quantity is the actual quantity used, not the quantity purchased. The standard quantity is

Part 1 : 10/05/14 22:20:20

(c) HOCK international, page 73

Page 74: Hock_Performance Mgmt

8/16/2019 Hock_Performance Mgmt

http://slidepdf.com/reader/full/hockperformance-mgmt 74/152

the standard quantity allowed for the actual output.

C.

The price variance is calculated as follows: (Actual Price − Standard Price) × Actual Quantity. Since the purchase pricevariance is required, the price to use for the actual price is the price per unit of the units purchased instead of the priceper unit of the units used; and the actual quantity is the number of units purchased. This price variance is incorrect

because the figure used in the formula is not the actual quantity of units purchased but is instead the standardquantity for the planned production (26,000 units x 3 pounds per unit).

The quantity variance is calculated as follows: (Actual Quantity − Standard Quantity for Actual Output ) × StandardPrice. The actual quantity is the actual quantity used, not the quantity purchased. The standard quantity is the standardquantity allowed for the actual output. This quantity variance is incorrect because it uses the standard quantity allowedfor the planned production (26,000 × 3 pounds) instead of the standard quantity allowed for the actual production.

D. The purchase price variance is incorrect because it is calculated using the quantity used in production instead of thequantity purchased. When a question asks for the purchase price variance, the actual quantity to use is the quantitypurchased, and the actual price is the actual price per unit for the quantity purchased.

Question 112 - ICMA 10.P1.095 - Manufacturing Input Variances -- Materials and Labor 

MinnOil performs oil changes and other minor maintenance services (e.g., tire pressure checks) for cars. The companyadvertises that all services are completed within 15 minutes for each service. On a recent Saturday, 160 cars wereserviced resulting in the following labor variances: rate, $19 unfavorable; efficiency, $14 favorable. If MinnOil's standardlabor rate is $7 per hour, determine the actual wage rate per hour and the actual hours worked.

 A. Wage Rate $6.67/hour, Hours Worked 42.71.B. Wage Rate $7.45/hour, Hours Worked 42.00.C. Wage Rate $7.50/hour; Hours Worked 38.00.D. Wage Rate $6.55/hour; Hours Worked 42.00.

 A.

This is not the correct answer. Please see the correct answer for an explanation.

We have been unable to determine how to calculate this incorrect answer choice. If you have calculated it, please letus know how you did it so we can create a full explanation of why this answer choice is incorrect. Please send us anemail at [email protected]. Include the full Question ID number and the actual incorrect answer choice --not its letter, because that can change with every study session created. The Question ID number appears in the upperright corner of the ExamSuccess screen. Thank you in advance for helping us to make your HOCK study materialsbetter.

B. This answer results from solving the Quantity Variance formula for the variable AQ using a positive QuantityVariance in the formula instead of a negative one. The Quantity (Efficiency) Variance is favorable. Since it is a cost, afavorable variance is represented by a negative amount. Therefore, the Quantity Variance used in the formula should

be negative. See the correct answer for a complete explanation.

C.

This is an example of price and quantity variances applied to a service business. To answer this question, wemust work out both the price (rate) variance and the quantity (efficiency) variance. The price variance formulais (AP − SP) × AQ = Price Variance. The quantity variance formula is (AQ − SQ) × SP = Quantity Variance. Sincethese are cost items, a negative variance is a favorable variance (actual is lower than standard) and a positivevariance is an unfavorable one (actual is higher than standard).

Part 1 : 10/05/14 22:20:20

(c) HOCK international, page 74

Page 75: Hock_Performance Mgmt

8/16/2019 Hock_Performance Mgmt

http://slidepdf.com/reader/full/hockperformance-mgmt 75/152

The Standard Quantity (SQ) is the standard for the number of hours to service 160 cars. The standard is 0.25hours per car multiplied by 160 cars, which is 40 hours.

The Standard Price (SP) is given as $7 per hour.

The Price Variance is $19 unfavorable.

The Quantity Variance is ($14) favorable.

Whenever we have all of the values we need except one, we can set up a simple equation and solve for themissing variable. We have all of the values for the Quantity Variance formula except for AQ. So the firstequation will be:

(AQ − 40) × 7 = (14)7 AQ − 280 = (14)7 AQ = 266AQ = 38, and this is the actual hours worked.

Now, we can take that value for AQ and use it in the Price Variance formula to solve for AP:

(AP − 7) × 38 = 1928AP − 266 = 1938AP = 285AP = 7.50, and this is the actual wage rate.

D. This answer results from two errors: (1) Solving the Quantity Variance formula for the variable AQ using a positiveQuantity Variance in the formula instead of a negative one. The Quantity (Efficiency) Variance is favorable. Since it is acost, a favorable variance is represented by a negative amount. Therefore, the Quantity Variance used in the formulashould be negative. And (2) Solving the Price Variance formula for the variable AP using a negative Price Variance inthe formula instead of a positive one. The Price (Rate) Variance is unfavorable. Since it is a cost, an unfavorablevariance is represented by a positive amount. Therefore, the Price Variance used in the formula should be positive.See the correct answer for a complete explanation.

Question 113 - CMA 693 3.12 - Performance Measures

Edith Carolina, president of the Deed Corporation, requires a minimum return on investment of 8% for any project to beundertaken by her company. The company is decentralized, and leaves investment decisions up to the discretion of thedivision managers as long as the 8% return is expected to be realized. Michael Sanders, manager of the CosmeticsDivision, has had a return on investment of 14% for his division for the past 3 years and expects the division to havethe same return in the coming year. Sanders has the opportunity to invest in a new line of cosmetics which is expectedto have a return on investment of 12%.

If the Deed Corporation evaluates managerial performance using residual income based on the corporate minimumrequired rate of return, what will be the preference for taking on the proposed cosmetics line by Edith Carolina andMichael Sanders?

 A. Carolina will accept / Sanders will acceptB. Carolina will accept / Sanders will rejectC. Carolina will reject / Sanders will rejectD. Carolina will reject / Sanders will accept

A. Residual income measures the dollar return of an investment. Since the return on this investment is 12%and the required return is 8%, both of these projects will have a positive residual income and both Carolinaand Sanders would accept the proposal.

Part 1 : 10/05/14 22:20:20

(c) HOCK international, page 75

Page 76: Hock_Performance Mgmt

8/16/2019 Hock_Performance Mgmt

http://slidepdf.com/reader/full/hockperformance-mgmt 76/152

B. Residual income measures the dollar return of an investment. Since the return on this investment is 12% and therequired return is 8%, both of these projects will have a positive residual income and both Carolina and Sanders wouldaccept the proposal.

C. Residual income measures the dollar return of an investment. Since the return on this investment is 12% and therequired return is 8%, both of these projects will have a positive residual income and both Carolina and Sanders wouldaccept the proposal.

D. Residual income measures the dollar return of an investment. Since the return on this investment is 12% and therequired return is 8%, both of these projects will have a positive residual income and both Carolina and Sanders wouldaccept the proposal.

Question 114 - CMA 694 3.27 - Responsibility Centers and Reporting Segments

The Stonebrook Company uses a performance reporting system that reflects the company's decentralization ofdecision making. The departmental performance reports show actual costs incurred during the period againstbudgeted costs. Any variances from the budget are assigned to the individual department manager who controls thecosts. Stonebrook is using a type of system called

 A. Responsibility accounting.B. Transfer-pricing accounting.C. Flexible budgeting.D. Activity-based budgeting.

A. Responsibility accounting is a system in which an individual is held accountable (responsible) for thingsthat he or she is able to control. It is this person's responsibility to address any issues that arise related tothese items that they control.

B. Transfer prices are the prices used in the 'sale' of goods within the company. See the correct answer for a completeexplanation.

C. Flexible budgeting is the adaptation of variable revenues and expenses in the static budget to reflect the actual levelof activity achieved during the period. See the correct answer for a complete explanation.

D. Activity based budgeting is a budgeting concept in which the budget is built using activities that cause costs to beincurred. See the correct answer for a complete explanation.

Question 115 - CMA 694 3.24 - Responsibility Centers and Reporting Segments

Decentralized firms can delegate authority and yet retain control and monitor managers' performance by structuring theorganization into responsibility centers. Which one of the following organizational segments is most like an independentbusiness?

 A. Profit center.B. Cost center.C. Revenue center.D. Investment center.

 A. A profit center is responsible for both revenues and costs, but a profit center is not responsible for the return on theinvestment made into it.

B. A cost center is responsible only for costs and not revenues. Therefore, it is not like an independent business.

Part 1 : 10/05/14 22:20:20

(c) HOCK international, page 76

Page 77: Hock_Performance Mgmt

8/16/2019 Hock_Performance Mgmt

http://slidepdf.com/reader/full/hockperformance-mgmt 77/152

C. A revenue center is not responsible for costs, and in that respect does not resemble an independent business.

D. An investment center is the most like an independent business because it is responsible for revenues,costs and investment; and it is measured on its return on investment as well as on its level of profit.

Question 116 - ICMA 10.P1.110 - Manufacturing Input Variances -- Overhead

Cordell Company uses a standard cost system. On January 1 of the current year, Cordell budgeted fixedmanufacturing overhead cost of $600,000 and production at 200,000 units. During the year, the firm produced 190,000units and incurred fixed manufacturing overhead of $595,000. The production volume variance for the year was

 A. $30,000 unfavorable.B. $25,000 unfavorable.C. $5,000 unfavorable.D. $10,000 unfavorable.

A.

The fixed overhead production volume variance is Budgeted Fixed Overhead − Fixed Overhead Applied. FixedOverhead Applied is the fixed overhead application rate per unit multiplied by the number of units actuallyproduced. The fixed overhead application rate per unit is $600,000 budgeted FOH ÷ 200,000 budgetedproduction, or $3 per unit. A total of 190,000 units were produced, so the amoiunt of fixed overhead appliedwas $3 × 190,000 = $570,000.

Budgeted Fixed Overhead was $600,000. Thus, the Fixed Overhead Production-Volume Variance was$600,000 − $570,000 = $30,000. Because the budgeted fixed overhead was greater than the applied fixedoverhead, this means that the actual volume produced was lower than the budgeted volume and so thevariance is unfavorable.

B. This is the incurred fixed overhead minus fixed overhead applied. The fixed overhead production-volume variance isBudgeted Fixed Overhead − Fixed Overhead Applied.

C. This is budgeted fixed overhead minus incurred fixed overhead. The fixed overhead production-volume variance isBudgeted Fixed Overhead − Fixed Overhead Applied.

D. This is the difference between the number of units budgeted for the year and the number of units actually producedduring the year. The fixed overhead production-volume variance is Budgeted Fixed Overhead − Fixed Overhead Applied.

Question 117 - CMA 1296 3.17 - Responsibility Centers and Reporting Segments

In theory, the optimal method for establishing a transfer price is

 A. Budgeted cost with or without a markup.B. Flexible budget cost.C. Incremental cost.D. Market price.

 A. Cost based transfer pricing is perhaps the best method of transfer pricing if the department that is buying is notrequired to buy from another internal department. However, if the buying department is unable to choose its supplier,the manager of the supplying internal department will not have any motivation to control costs, because he or she willknow that the costs will simply be charged to the next department.

Part 1 : 10/05/14 22:20:20

(c) HOCK international, page 77

Page 78: Hock_Performance Mgmt

8/16/2019 Hock_Performance Mgmt

http://slidepdf.com/reader/full/hockperformance-mgmt 78/152

B. Cost based transfer pricing is perhaps the best method of transfer pricing if the department that is buying is notrequired to buy from another internal department. However, if the buying department is unable to choose its supplier,the manager of the supplying internal department will not have any motivation to control costs, because he or she willknow that the costs will simply be charged to the next department.

C. Cost based transfer pricing is perhaps the best method of transfer pricing if the department that is buying is notrequired to buy from another internal department. However, if the buying department is unable to choose its supplier,

the manager of the supplying internal department will not have any motivation to control costs, because he or she willknow that the costs will simply be charged to the next department.

D. The transfer price is the price charged by one unit of the company to another unit of the same company forthe services or goods produced by the first unit and "sold" to the second unit. The goal in setting a transferprice is that the method used will stimulate the department managers — both selling and buying — to do whatwill provide the greatest benefit to the company as a whole, rather than to act in their own interest. When thereis an external market for the product, market price is almost always the best transfer price to use.

Question 118 - CMA 1294 3.24 - Manufacturing Input Variances -- Materials and Labor 

Tower Company planned to produce 3,000 units of its single product, Titactium, during November. The standardspecifications for one unit of Titactium include 6 pounds of materials at $0.30 per pound. Actual production inNovember was 3,100 units of Titactium. The accountant computed a favorable materials purchase price variance of$380 and an unfavorable materials quantity variance of $120. Based on these variances, one could conclude that

 A. More materials were used than were purchased.B. The actual usage of materials was less than the standard allowed.C. The actual cost of materials was less than the standard cost.D. More materials were purchased than were used.

 A. A difference between the quantity of materials purchased and used has nothing to do with a favorable materialspurchase price variance of $380 and an unfavorable materials quantity variance of $120. The materials efficiencyvariance is the difference between the actual quantity of material used and the standard quantity of materials allowed

multiplied by the standard price. The materials purchase price variance uses the actual quantity of materials purchased instead of the quantity of materials used.

B. An unfavorable materials quantity variance of $120 indicates that more materials were used than the standardallowed, not less.

C.

A favorable materials purchase price variance of $380 indicates that the actual price was less than thestandard price. We also can prove it by using the quantity variance formula to find the actual quantity of directmaterials used and then the price variance formula to calculate the actual price.

1. Use the quantity variance formula to find the Actual Quantity of direct materials used.(AQ − SQ) × SP = Quantity VarianceThe standard quantity allowed is 6 pounds per unit. Therefore, the standard quantity allowed for the actualproduction of 3,100 units is 3,100 × 6, or 18,600.(AQ − 18,600) × $0.30 = $120.3AQ − 5,580 = 120.3AQ = 5,700AQ = 19,000

2. Use the price variance formula to find the Actual Price of direct materials purchased.(AP − SP) × AQ

Part 1 : 10/05/14 22:20:20

(c) HOCK international, page 78

Page 79: Hock_Performance Mgmt

8/16/2019 Hock_Performance Mgmt

http://slidepdf.com/reader/full/hockperformance-mgmt 79/152

(AP − $0.30) × 19,000 = $(380)19,000AP − 5,700 = $(380)19,000AP = $5,320AP = $0.28

 The standard price was $0.30, so the actual price of $0.28 was lower than the standard price.

D. A difference between the quantity of materials purchased and used has nothing to do with a favorable materialspurchase price variance of $380 and an unfavorable materials quantity variance of $120. The materials efficiencyvariance is the difference between the actual quantity of material used and the standard quantity of material allowed forthe actual output, multiplied by the standard price. The materials purchase price variance uses the actual quantity ofmaterials purchased instead of the quantity of materials used.

Question 119 - CIA 592 IV.18 - Manufacturing Input Variances -- Materials and Labor 

The following is a standard cost variance analysis report on direct labor cost for a division of a manufacturing company.

Job

Actual Hours at

Actual Wages

Actual Hours at

Standard Wages

Standard Hours at

Standard Wages213 $ 3,243 $ 3,700 $ 3,100215 15,345 15,675 15,000217 6,754 7,000 6,600219 19,788 18,755 19,250221 3,370 3,470 2,650Totals $48,500 $48,600 $46,600

What is the total flexible budget direct labor variance for the division?

 A. $1,900 unfavorable.B. $2,000 unfavorable.C. $100 favorable.D. $1,900 favorable.

A.

The flexible budget amount is always the amount of cost allowed or budgeted for the actual activity. Theamount allowed for the actual activity is the amount allowed for one unit multiplied by the number of unitsactually produced.

The cost of direct labor allowed for one unit has two components: hourly rate and number of hours allowed toproduce one unit. The total flexible budget cost is the budgeted hourly rate multiplied by the number of hoursallowed (budgeted) to produce one unit multiplied by the number of units actually produced. That isrepresented in this question by the total of the column labeled "Standard Hours at Standard Wages."

The actual direct labor cost paid is the total of the column titled "Actual Hours at Actual Wages."

The question asks for the total flexible budget direct labor variance. The total flexible budget direct laborvariance is the actual cost minus the budgeted (or standard) cost for the actual activity.

Therefore, the total flexible budget direct labor variance is the total of the column titled "Actual Hours atActual Wages" ($48,500) minus the total of the column titled "Standard Hours at Standard Wages" ($46,600), or$1,900 unfavorable.

The total flexible budget direct labor variance is the labor rate variance and the labor efficiency variancecombined. Let's make up some detail behind the totals in each of the columns and work out the direct labor

Part 1 : 10/05/14 22:20:20

(c) HOCK international, page 79

Page 80: Hock_Performance Mgmt

8/16/2019 Hock_Performance Mgmt

http://slidepdf.com/reader/full/hockperformance-mgmt 80/152

rate variance and the direct labor efficiency variance to illustrate what each of the column totals represents.

Let's say the standard wage is $20 per hour. The total cost of the standard hours allowed at the standard wagefor the actual output (which is the flexible budget amount) is $46,600, the total of the third column. Therefore,at a standard wage rate per hour of $20, the standard number of hours allowed for the actual output wouldhave been 2,330, because $46,600 ÷ $20 = 2,330.

The cost for the total actual hours at the standard wages allowed for the actual output was $48,600, the totalof the second column. We are still using the standard wage rate of $20 per hour, so this tells us that the totalactual hours used to produce the actual output was 2,430 hours, because $48,600 ÷ $20 = 2,430.

The cost for the actual number of hours used at the actual wages was $48,500, the total of the first column.The actual number of hours used that we calculated above is 2,430. Therefore, the actual wage rate must havebeen $19.959 ($48,500 ÷ 2,430). There is a slight rounding problem with this number, but we can still use it forthis illustration.

Now, let's calculate the direct labor rate variance. The DL rate variance is (AP – SP) × AQ. Plugging in ournumbers, we have ($19.959 – $20) × 2,430, which equals $(100) rounded. Because it is a negative number for acost item, it is a favorable variance. That is also the difference between the $48,500 total cost for actual hoursat actual wages (first column) and the $48,600 total cost for actual hours at standard wages (second column).

The direct labor efficiency variance is (AQ – SQ) × SP, or (2,430 – 2,330) × $20 = $2,000 Unfavorable. That isalso the difference between the total cost of $48,600 for the actual hours at standard wages (second column)and $46,600 for the standard number of hours allowed at the standard wages (third column).

So, the difference between column 1 and column 2 is the direct labor rate variance. It is the amount of the totalvariance that was caused by the difference between the actual wage rate paid and the standard wage rate. Thedifference between column 2 and column 3 is the direct labor efficiency variance. That is the amount of thevariance caused by the difference between the actual number of hours used for the actual production and thestandard number of hours allowed for the actual production.

The difference between column 1 and column 3 is the total direct labor flexible budget variance, which is$48,500 - $46,600 = $1,900 U. It is also the rate variance of $(100) plus the efficiency variance of $2,000. That isthe variance amount this question is asking for.

B.

This is the difference between the Actual Hours at Standard Wages and Standard Hours at Standard Wages. This isthe direct labor efficiency variance. The formula for the direct labor efficiency variance is (AQ − SQ) × SP. That formulacan also be written as (AQ × SP) − (SQ × SP). (AQ × SP) is equal to "Actual Hours at Standard Wages," while (SP × AQ) is equal to "Standard Hours at Standard Wages."

The direct labor efficiency variance is one component of the total flexible budget direct labor variance. The othercomponent is the direct labor rate variance. However, the question asks for the total flexible budget direct laborvariance.

C.

This is the difference between the Actual Hours at Actual Wages and Actual Hours at Standard Wages. This is thedirect labor rate variance. The formula for the direct labor rate variance is (AP − SP) × AQ. That formula can also bewritten as (AP × AQ) − (SP × AQ). (AP × AQ) is equal to "Actual Hours at Actual Wages," while (SP × AQ) is equal to"Actual Hours at Standard Wages."

The direct labor rate variance is one component of the total flexible budget direct labor variance. The other componentis the direct labor efficiency variance. However, the question asks for the total flexible budget direct labor variance.

D. The flexible budget variance is unfavorable, because the actual cost incurred is greater than the standard cost forthe actual level of output (the flexible budget).

Part 1 : 10/05/14 22:20:20

(c) HOCK international, page 80

Page 81: Hock_Performance Mgmt

8/16/2019 Hock_Performance Mgmt

http://slidepdf.com/reader/full/hockperformance-mgmt 81/152

Question 120 - ICMA 10.P1.127 - Responsibility Centers and Reporting Segments

With respect to a firm's transfer pricing policy, an advantage of using a dual pricing arrangement is that it

 A. promotes goal congruence between the supplying and buying subunits of the firm.B. exposes the supplying subunit to the discipline of market prices.C. simplifies tax calculations when the buying and supplying subunits are taxed in different jurisdictions.D. provides an incentive for the supplying subunit to control costs.

A.

Goal congruence is defined as "aligning the goals of two or more groups." As used in this question, it meansthe goals of the individual managers are aligned with those of the other and with the goals of the organizationas a whole. In other words, the managers, while each acting in their own best interests, will also be acting inthe best interest of each other and of the organization as a whole.

With respect to transfer pricing, it is in the best interest of the organization that the products or services be

bought and sold internally rather than from outside. The cost to the organization as a whole will be lowerbecause it will not include a profit paid to the outside firm over and above its costs. The organization canessentially get the goods or services at cost.

Thus, anything that encourages the organization's divisions to buy from one another instead of from outsidewill be good for the firm. A dual rate transfer price provides both the buying and the selling division with anadvantageous price. The selling division receives the market price for the sale, while the buying division getsa purchase price that is lower than it would be if it were to purchase outside. Thus, use of a dual rate transferprice promotes goal congruence between the two managers and also between the two managers and theorganization as a whole.

B. Dual rate pricing is a transfer pricing method in which the internal selling and the purchasing departments eachrecord the transaction at different prices. For the seller (the supplying subunit), the price will probably be the market

price. That is an advantageous transfer price for the selling department because it is equal to the market price theseller would receive from selling its product to an outside customer. Sales at the market price are all subject to thediscipline of market prices; internal transfer transactions made at the market price to the internal buyer are no differentfrom sales made at the market price to an external buyer.

C. A dual pricing arrangement does not simplify tax calculations when the buying and supplying subunits are taxed indifferent jurisdictions.

D. Dual rate pricing is a transfer pricing method in which the internal selling and the purchasing departments eachrecord the transaction at different prices. There is nothing in this arrangement that would provide an incentive for thesupplying subunit (the seller) to control costs, because the transfer price on the seller's side will probably be the marketprice the company would charge an outside customer.

Question 121 - CIA 590 IV.15 - Manufacturing Input Variances -- Materials and Labor 

 A manager prepared the following table by which to analyze labor costs for the month:Actual Hours at

Actual RateActual Hours atStandard Rate

Standard Hours atStandard Rate

$10,000 $9,800 $8,820

What variance was $980?

Part 1 : 10/05/14 22:20:20

(c) HOCK international, page 81

Page 82: Hock_Performance Mgmt

8/16/2019 Hock_Performance Mgmt

http://slidepdf.com/reader/full/hockperformance-mgmt 82/152

 A. Labor efficiency variance.B. Labor spending variance.C. Volume variance.D. Labor rate variance.

A.

To solve this question, we need to calculate all the suggested variances possible from this set of data. It'sbetter to start with most simple ones. The labor efficiency variance is: (Actual Hours − Standard Hours forActual Output) × Standard Rate, or (Actual Hours × Standard Rate) − (Standard Hours for Actual Output ×Standard Rate).

The actual hours times the standard rate is $9,800. The standard hours times the standard rate is $8,820. Thedifference is $980 unfavorable.

B.

There is no such thing as a labor spending variance. Labor variances are either labor efficiency variances or labor ratevariances.

The variable overhead spending variance is related to variable overhead and is the difference between the actualapplication rate and the standard application rate multiplied by the actual quantity of the application base (level ofactivity). This can also be calculated as the actual variable overhead incurred − (the actual usage of the applicationbase × the standard variable overhead rate). This cannot be determined using the given set of data.

C. The volume variance is related to the fixed overhead and is the difference between the budgeted amount of fixedoverhead and the amount of fixed overhead applied (standard rate × standard input for the actual level of output). Thevolume variance cannot be determined using the given set of data.

D.

To solve this question, we need to calculate all the suggested variances possible from this set of data. It's better tostart with most simple ones. The labor rate variance is: (Actual Rate − Standard Rate) × Actual Hours or (Actual Rate × Actual Hours) − (Standard Rate × Actual Hours).

The actual rate times the actual hours is $10,000. The standard rate times the actual hours is $9,800. The differenceis $200 unfavorable. This is not the result we are looking for.

Question 122 - ICMA 10.P1.091 - Manufacturing Input Variances -- Overhead

Of the following pairs of variances found in a flexible budget report, which pair is most likely to be related?

 A. Material usage variance and labor efficiency variance.B. Labor rate variance and variable overhead efficiency variance.C. Material price variance and variable overhead efficiency variance.D. Labor efficiency variance and fixed overhead volume variance.

A.

A material usage variance can be caused by labor factors, and a labor efficiency variance can be caused bymaterial quality factors.

If employees are new or untrained, an unfavorable labor efficiency variance can result. The untrainedemployees may also cause more direct material spoilage, which will result in an unfavorable material usage

Part 1 : 10/05/14 22:20:20

(c) HOCK international, page 82

Page 83: Hock_Performance Mgmt

8/16/2019 Hock_Performance Mgmt

http://slidepdf.com/reader/full/hockperformance-mgmt 83/152

variance. It can work the other way, as well. Knowledgeable and efficient employees can create both afavorable labor efficiency variance and a favorable material usage variance.

Furthermore, an unfavorable material usage variance can be caused by inferior materials. The inferiormaterials that the employees have to work with can also cause an unfavorable labor efficiency variance aswell, because it may take longer to produce the product using inferior materials.

B.

It is not likely that a labor rate variance and a variable overhead efficiency variance would be related. A labor ratevariance results from a difference between the actual hourly rate paid for direct labor used and the budgeted hourlyrate. The variable overhead efficiency variance is the amount of the total variance caused by a different usage of theallocation base (either direct labor hours or machine hours) than was expected. Even if direct labor hours are beingused as the allocation base, the usage of the allocation base is a quantity variance whereas the labor rate variance is aprice variance. The labor rate variance and the variable overhead efficiency variance are not related, and it is veryunlikely that one would cause the other.

C. It is not likely that a material price variance and a variable overhead efficiency variance would be related. A materialprice variance results from a difference between the actual price per unit of the direct materials used and the budgetedprice per unit. The variable overhead efficiency variance is the amount of the total variance caused by a different usageof the allocation base (either direct labor hours or machine hours) than was expected. The material price variance and

the variable overhead efficiency variance are not related, and it is very unlikely that one would cause the other.

D.

It is not likely that a labor efficiency variance and a fixed overhead volume variance would be related. A labor efficiencyvariance results from a difference between the actual number of labor hours required for the actual output and thestandard number of labor hours allowed for the actual output. The fixed overhead volume variance (also called thefixed overhead production-volume variance) is a measure of capacity utilization. It is caused by the actual productionlevel being different from the production level used to calculate the budgeted fixed overhead rate. The labor efficiencyvariance and the fixed overhead volume variance are not related, and it is very unlikely that one would cause the other.

Question 123 - CIA 1188 IV.23 - Responsibility Centers and Reporting Segments

The price that one division of a company charges another division for goods or services provided is called the

 A. Transfer price.B. Outlay price.C. Market price.D. Distress price.

A. The transfer price is the price charged by one unit of the company to another unit of the same company forthe services or goods produced by the first unit and "sold" to the second unit.

B. The outlay price is one approach to determining a transfer price.

C. The market price is determined purely by the forces of supply and demand without interference from an outsidesource. This concept assumes that markets are efficient, which is not always true in practice. Market price is one ofmany possible approaches to determining a transfer price.

D. The distress price is a reduced price (sometimes significantly) that occurs in urgent sales. Distress price is not anapproach to determining a transfer price.

Part 1 : 10/05/14 22:20:20

(c) HOCK international, page 83

Page 84: Hock_Performance Mgmt

8/16/2019 Hock_Performance Mgmt

http://slidepdf.com/reader/full/hockperformance-mgmt 84/152

Question 124 - CMA 1293 3.24 - Manufacturing Input Variances -- Materials and Labor 

ChemKing uses a standard costing system in the manufacture of its single product. The 35,000 units of raw material ininventory were purchased for $105,000, and two units of raw material are required to produce one unit of final product.In November, the company produced 12,000 units of product. The standard cost for material allowed for the output was$60,000, and there was an unfavorable quantity variance of $2,500.

The materials price variance for the units used in November was

 A. $6,000 unfavorable.B. $2,500 unfavorable.C. $12,000 unfavorable.D. $12,500 unfavorable.

 A. This answer results from using the actual quantity of units produced as the AQ in the materials price varianceformula, instead of the actual quantity of materials used. See the correct answer for a complete explanation.

B. $2,500 Unfavorable is the quantity variance, which is given in the question.

C. This answer results from using the standard amount of materials for the actual quantity produced as the AQ in thematerials price variance formula, instead of the actual quantity of materials used. See the correct answer for a

complete explanation.

D.

The formula for calculating the materials price variance is (AP − SP) × AQ. The actual price for raw materialswas $105,000 ÷ 35,000 units in inventory, or $3.00 per unit. The standard price is $2.50 per unit of rawmaterials ($60,000 ÷ 12,000 units actually produced ÷ 2 units of materials per unit produced). The ActualQuantity in the formula is the actual quantity of the raw materials that were used in producing the 12,000finished units. It is not the actual quantity of product produced, nor is it the standard quantity of materials forthe actual quantity produced.

So to calculate the price variance, we need to know how many units of materials were actually used inproduction, and the question does not tell us that. However, it does tell us that there was an unfavorable

quantity variance. So to figure out how many units of materials were actually used, we first have to work outthe quantity variance, which the question says is an unfavorable $2,500, to solve for the AQ in that formula.

The quantity variance formula is (AQ − SQ) × SP. Since the question tells us that the materials standard is twounits of raw materials for each unit produced, we know that the standard quantity of materials for 12,000 unitswas 24,000 units. The actual quantity we do not know yet. The standard price was $2.50 per unit of rawmaterial ($60,000 standard allowed ÷ 12,000 units actually produced ÷ 2 units of materials per unit produced).Therefore, our formula is:

(AQ − 24,000) × 2.50 = 2,500

Solving for AQ, we get AQ = 25,000, as follows:

2.5AQ − 60,000 = 2,5002.5AQ = 62,500AQ = 25,000

Now, we can put the actual quantity of materials used into the materials price variance formula and calculatethe materials price variance.

(AP − SP) × AQ

($3.00 − $2.50) × 25,000 = $12,500 Unfavorable

Part 1 : 10/05/14 22:20:20

(c) HOCK international, page 84

Page 85: Hock_Performance Mgmt

8/16/2019 Hock_Performance Mgmt

http://slidepdf.com/reader/full/hockperformance-mgmt 85/152

Question 125 - CMA 681 4.2 - Performance Measures

The imputed interest rate used in the residual income approach for performance measurement and evaluation canbest be characterized as the

 A. Historical weighted average cost of capital for the company.B. Marginal after-tax cost of new equity capital.C. Average return on investment that has been earned by the company over a particular period.D. Target return on investment set by management.

 A. The historical weighted average cost of capital for the company may be used in residual income calculation only if itis set as a target rate of return. Otherwise, this past figure is irrelevant to the evaluation of future managerialperformance.

B. The marginal after-tax cost of new equity capital may be used in the residual income calculation only if it is set as atarget rate of return. Otherwise, this past figure is irrelevant to the evaluation of future managerial performance.

C. The average return on investment that has been earned by the company over a particular period may be used in

residual income calculation only if it is set as a target rate of return. Otherwise, this past figure is irrelevant to theevaluation of future managerial performance.

D. In the calculation of residual income, management normally sets a desired target rate of return that it wantsto achieve. The target rate of return may be set based prior experience or any other estimation depending onmanagement's decision.

Question 126 - ICMA 10.P1.102 - Manufacturing Input Variances -- Materials and Labor 

Frisco Company recently purchased 108,000 units of raw material for $583,200. Three units of raw materials arebudgeted for use in each finished good manufactured, with the raw material standard set at $16.50 for each completedproduct. Frisco manufactured 32,700 finished units during the period just ended and used 99,200 units of rawmaterial. If management is concerned about the timely reporting of variances in an effort to improve cost control andbottom-line performance, the materials purchase price variance should be reported as

 A. $10,800 unfavorable.B. $10,800 favorable.C. $6,050 unfavorable.D. $9,920 favorable.

 A. The variance is unfavorable, not favorable.

B.

The formula for the price variance is (AP – SP) × AQ. Because this is asking for the purchase price variance,we use the actual quantity purchased, not the actual quantity used, for AQ.

The actual price is $583,200 ÷ 108,000 = $5.40 per unit of raw material.The standard price is $16.50 per completed product unit ÷ 3 units of raw material used per completed productunit = $5.50The actual quantity purchased is 108,000.

The purchase price variance = ($5.40 – $5.50) × 108,000 = $(10,800) favorable.

Part 1 : 10/05/14 22:20:20

(c) HOCK international, page 85

Page 86: Hock_Performance Mgmt

8/16/2019 Hock_Performance Mgmt

http://slidepdf.com/reader/full/hockperformance-mgmt 86/152

C. This is the materials quantity variance, not the materials purchase price variance.

D. This is the materials price variance, calculated using the amount of direct materials used during the period.However, the question asks for the materials purchase price variance. The materials purchase price variance iscalculated using the quantity of materials purchased during the period, not the quantity of materials used.

Question 127 - CIA 1190 IV.20 - Responsibility Centers and Reporting Segments

 A limitation of transfer prices based on actual cost is that they

 A. Can lead to suboptimal decisions for the company as a whole.B. Lack clarity and administrative convenience.C. Charge inefficiencies to the department that is transferring the goods.D. Must be adjusted by some markup.

A. The basic issue of transfer prices is how much should one unit of a company charge another unit of thesame company for its goods or services. The goal in setting a transfer price is that the method used willstimulate the department managers to do what will provide the greatest benefit to the company as a whole,rather than to act in their own interest. A transfer price based on actual cost does not motivate managers touse resources more efficiently, which can lead to suboptimal decisions for the company as a whole.

B. Cost-based transfer price provide clarity and administrative convenience.

C. Inefficiencies are charged to the department that is receiving, not transferring, the goods.

D. In cost-based transfer pricing, costs do not need to be adjusted by any markup.

Question 128 - ICMA 10.P1.118 - Manufacturing Input Variances -- Materials and Labor 

During the month of May, Tyler Company experienced a significant unfavorable material efficiency variance in theproduction of its single product at one of Tyler’s plants. Which one of the following reasons would be least likely toexplain why the unfavorable variance arose?

 A. Workers used were less skilled than expected.B. Actual production was lower than planned production.C. Replacement production equipment had just been installed.D. Inferior materials were purchased.

 A. Less-skilled workers can explain an unfavorable material efficiency (quantity) variance, because less-skilled workersmake more mistakes and can cause more of the work to have to be discarded, including the direct materials that havegone into the discarded work.

B. This, by itself, would not be likely to explain an unfavorable material efficiency (quantity) variance. There isnothing about the production level that should unfavorably impact the amount of direct materials used foreach unit produced. However, inferior materials, less-skilled workers and workers learning to use newproduction equipment could explain an unfavorable materials quantity variance.

C. New production equipment can explain an unfavorable material efficiency (quantity) variance, because workersworking on new equipment experience a learning curve. They could make mistakes that could cause their work,including the direct materials used, to have to be discarded; and more of this could occur than normal.

D. Inferior materials in the production process could explain a significant unfavorable material efficiency (quantity)

Part 1 : 10/05/14 22:20:20

(c) HOCK international, page 86

Page 87: Hock_Performance Mgmt

8/16/2019 Hock_Performance Mgmt

http://slidepdf.com/reader/full/hockperformance-mgmt 87/152

variance, because an increased amount of the materials would be defective and thus unusable.

Question 129 - CMA 696 3.23 - Manufacturing Input Variances -- Materials and Labor 

 Ardmore Enterprises uses a standard cost system in its small appliance division. The standard cost of manufacturingone unit of Zeb is as follows:Materials - 60 pounds at $1.50 per pound $ 90Labor - 3 hours at $12 per hour 36Factory overhead - 3 hours at $8 per hour 24Total standard cost per unit $150

The budgeted variable factory overhead rate is $3 per labor hour, and the budgeted fixed factory overhead is $27,000per month. During May, Ardmore produced 1,650 units of Zeb compared with a normal capacity of 1,800 units. Theactual cost per unit was as follows:Materials (purchased and used) - 58 pounds at $1.65 per pound $ 95.70Labor - 3.1 hours at $12 per hour 37.20Factory overhead - $39,930 per 1,650 units 24.20

Total actual cost per unit $157.10

The materials price variance for May is

 A. $14,850 unfavorable.B. $14,355 unfavorable.C. $14,355 favorable.D. $14,850 favorable.

 A. The price variance is calculated as follows: (Actual Price − Standard Price) × Actual Quantity. This incorrect answerresults from using the standard quantity instead of the actual quantity. See the correct answer for a completeexplanation.

B. The price variance is calculated as follows: (Actual Price − Standard Price) × Actual Quantity. The actual

price was $1.65 per pound and the standard price was $1.50 per pound. The actual quantity used was 95,700pounds (1,650 units produced × 58 pounds per unit). Putting these numbers into the formula, we get ($1.65 −$1.50) × 95,700 = $14,355 unfavorable.

C. The variance is unfavorable, as the actual price per pound of material was greater than the standard price. See thecorrect answer for a complete explanation.

D. The variance is unfavorable as the actual price per pound of material was greater than the standard price. Inaddition, this answer is incorrect because the standard quantity was used in the variance formula instead of the actualquantity. See the correct answer for a complete explanation.

Question 130 - CMA 697 3.29 - Performance Measures

Listed below is selected financial information for the Western Division of the Hinzel Company for last year.

 Account Amount

(thousands) Average working capital $ 625General and administrative expenses 75Net sales 4,000 Average plant and equipment 1,775

Part 1 : 10/05/14 22:20:20

(c) HOCK international, page 87

Page 88: Hock_Performance Mgmt

8/16/2019 Hock_Performance Mgmt

http://slidepdf.com/reader/full/hockperformance-mgmt 88/152

Cost of goods sold 3,525

If Hinzel treats the Western Division as an investment center for performance measurement purposes, what is thebefore-tax return on investment (ROI) for last year?

 A. 22.54%B. 34.78%

C. 19.79%D. 16.67%

 A. ROI is calculated as net income divided by average total assets (or investments). This answer does not includeworking capital in average total assets.

B. ROI is calculated as net income divided by average total assets (or investments). This answer results from dividingnet income by average plant and equipment − average working capital. Average working capital should be added toaverage plant and equipment, not subtracted from it.

C. ROI is calculated as net income divided by average total assets (or investments). This answer fails to reduce netincome for general and administrative costs.

D. ROI is calculated as net income divided by average total assets (or investments). Net income equals net

sales minus COGS and G&A expenses, or $400,000 ($4,000,000 − $3,525,000 − $75,000). Average assets areequal to the sum of average plant and equipment plus working capital, or $2,400,000 ($625,000 + $1,775,000).Now we can calculate ROI: $400,000 ÷ $2,400,000 = 16.67%.

Question 131 - CIA 589 IV.15 - Responsibility Centers and Reporting Segments

In a responsibility accounting system, managers are accountable for

 A. Product costs but not for period costs.B. Incremental costs.C. Variable costs but not for fixed costs.D. Costs over which they have significant influence.

 A. In a responsibility accounting system, managers are responsible for the costs that they control, whether they areproduct or period costs.

B. In a responsibility accounting system, managers are responsible for the costs that they control, whether they areincremental costs or not.

C. In a responsibility accounting system, managers are responsible for the costs that they control, whether they arefixed or variable.

D. In a responsibility accounting system, managers are responsible for the costs that they control. Costs overwhich a manager has significant influence are costs that they control.

Question 132 - ICMA 10.P1.116 - Manufacturing Input Variances -- Materials and Labor 

Fortune Corporation’s Marketing Department recently accepted a rush order for a nonstock item from a valuedcustomer. The Marketing Department filed the necessary paperwork with the Production Department, whichcomplained greatly about the lack of time to do the job the right way. Nevertheless, the Production Departmentaccepted the manufacturing commitment and filed the required paperwork with the Purchasing Department for the

Part 1 : 10/05/14 22:20:20

(c) HOCK international, page 88

Page 89: Hock_Performance Mgmt

8/16/2019 Hock_Performance Mgmt

http://slidepdf.com/reader/full/hockperformance-mgmt 89/152

needed raw materials. A purchasing clerk temporarily misplaced the paperwork. By the time the paperwork wasfound, it was too late to order from the company’s regular supplier. A new supplier was located, and that vendorquoted a very attractive price. The materials arrived and were rushed into production, bypassing the normal inspectionprocesses (as directed by the Production Department supervisor) to make up for lost time. Unfortunately, the goodswere of low quality and created considerable difficulty for Fortune’s assembly-line personnel. Which of the followingbest indicates the responsibility for the materials usage variance in this situation?

 A. Purchasing and Marketing.B. Purchasing.C. Marketing and Production.D. Purchasing, Marketing, and Production.

 A. The purchasing department and the marketing department have some of the responsibility, but they are not the onlydepartments that are responsible for the variance.

B. The purchasing department has some responsibility, but it is not the only department that is responsible for thevariance.

C. The marketing department and the production department have some of the responsibility, but they are not the onlydepartments that are responsible for the variance.

D. Purchasing, marketing and production all have some responsibility for the variance. The purchasingdepartment is responsible for misplacing the paperwork and not ordering the raw materials in a timelymanner, necessitating the search for a new supplier. The marketing department is responsible because theyaccepted a rush order for a nonstock item without checking first with the production department aboutwhether it was feasible to fulfill the order under the terms the customer required. The production departmentsupervisor is responsible for rushing the raw materials received into production without inspecting them first.

Question 133 - CIA 1190 IV.18 - Sales and Market Variances

The following exhibit reflects a summary of performance for a single item of a retail store's inventory for April.

 Actual

Results

FlexibleBudgetVariances

FlexibleBudget

Static(Master)Budget

Sales (units) 11,000 11,000 12,000Revenue (sales) $208,000$(12,000) U $220,000 $240,000Variable costs 121,000 (11,000)U 110,000 120,000Contribution Margin $ 87,000 $(23,000) U $110,000 $120,000Fixed costs 72,000 -- 72,000 72,000Operating Income $ 15,000 $(23,000) U $ 38,000 $ 48,000

The sales volume variance is

 A. $1,000 F.

B. $11,000 F.C. $10,000 U.D. $12,000 U.

 A.

The sales volume variance measures the impact of the difference in sales volume between the actual results and thestatic budget. The sales volume variance for the contribution margin is the flexible budget contribution margin minusthe static budget contribution margin. The sales volume variance is unfavorable as the flexible budget contributionmargin is lower than the static (master) budget contribution margin.

Part 1 : 10/05/14 22:20:20

(c) HOCK international, page 89

Page 90: Hock_Performance Mgmt

8/16/2019 Hock_Performance Mgmt

http://slidepdf.com/reader/full/hockperformance-mgmt 90/152

B. The sales volume variance measures the impact of the difference in sales volume between the actual results andthe static budget. The sales volume variance for the contribution margin is the flexible budget contribution marginminus the static budget contribution margin. The sales volume variance is unfavorable as the flexible budgetcontribution margin is lower than the static (master) budget contribution margin.

C.

The sales volume variance measures the impact of the difference in sales volume between the actual resultsand the static budget. The sales volume variance for a single product (or for a single product firm) can becalculated for each variable income and expense item as well as for the contribution margin. If a questiondoes not specify which line to use, as this one does not, use the contribution margin line.

For the contribution margin line, it is calculated as follows: (Actual Sales Volume − Budgeted Sales Volume) ×Budgeted Contribution per Unit, or (AQ − SQ) × SP. The "AQ," actual quantity, is 11,000. The "SQ," budgetedquantity, is 12,000. The "SP," budgeted price, is the budgeted contribution margin per unit. That is $10 per unit($110,000 ÷ 11,000 or $120,000 ÷ 12,000). Thus the variance is (11,000 − 12,000) × $10, or ($10,000). A negativevariance for an income line or for the contribution margin line is an unfavorable variance, because it meansthe actual was lower than the budget.

We can also calculate the variance using the amounts given in the variance report. The sales volume variance

for the contribution margin is the flexible budget contribution margin minus the static budget contributionmargin. Actual sales volume times budgeted contribution per unit is the flexible budget contribution margin,i.e. $110,000. Budgeted sales volume times budgeted contribution per unit is the static (master) budgetcontribution margin, i.e. $120,000. $110,000 − $120,000 equals ($10,000) unfavorable.

D. The sales volume variance measures the impact of the difference in sales volume between the actual results andthe static budget. The sales volume variance for the contribution margin is the flexible budget contribution marginminus the static budget contribution margin. This answer is the flexible budget variance for revenue, which is not ananswer to the question.

Question 134 - CMA 696 3.22 - Manufacturing Input Variances -- Materials and Labor 

 Ardmore Enterprises uses a standard cost system in its small appliance division. The standard cost of manufacturingone unit of Zeb is as follows:Materials - 60 pounds at $1.50 per pound $ 90Labor - 3 hours at $12 per hour 36Factory overhead - 3 hours at $8 per hour 24Total standard cost per unit $150

The budgeted variable factory overhead rate is $3 per labor hour, and the budgeted fixed factory overhead is $27,000per month. During May, Ardmore produced 1,650 units of Zeb compared with a normal capacity of 1,800 units. Theactual cost per unit was as follows:Materials (purchased and used) - 58 pounds at $1.65 per pound $ 95.70Labor - 3.1 hours at $12 per hour 37.20

Factory overhead - $39,930 per 1,650 units 24.20Total actual cost per unit $157.10

The total materials quantity variance for May is

 A. $14,355 favorable.B. $4,950 favorable.C. $4,950 unfavorable.D. $14,355 unfavorable.

Part 1 : 10/05/14 22:20:20

(c) HOCK international, page 90

Page 91: Hock_Performance Mgmt

8/16/2019 Hock_Performance Mgmt

http://slidepdf.com/reader/full/hockperformance-mgmt 91/152

 A. The actual quantity of material used (95,700) was less than the standard quantity (99,000). This means thatvariance is favorable. See the correct answer for a complete explanation.

B. The quantity variance (also called the efficiency or usage variance) is calculated as: (Actual Quantity −Standard Quantity for Actual Output) × Standard Price. The actual quantity of material used was 58 lb. per unitof finished product or 95,700 units in total (1,650 × 58 lb.). The standard quantity to produce 1,650 units equals99,000 units (1,650 × 60 lb.). The standard price is $1.50. Therefore, the quantity variance is (95,700 − 99,000) ×

$1.50 = $(4,950) favorable. The actual quantity used was less than the standard quantity for the actual output,which means that the variance is favorable.

C. The quantity variance is favorable because the quantity of material actually used was less than the standard quantityfor the actual output. See the correct answer for a complete explanation.

D. This is the material price variance. See the correct answer for a complete explanation.

Question 135 - CIA 588 IV.19 - Responsibility Centers and Reporting Segments

The alpha division of a company, which is operating at capacity, produces and sells 1,000 units of a certain electroniccomponent in a perfectly competitive market. Revenue and cost data are as follows:Sales $50,000Variable costs 34,000Fixed costs 12,000

The minimum transfer price that should be charged to the beta division of the same company for each component is:

 A. $12B. $34C. $46D. $50

 A. Because the alpha division is operating at capacity, the minimum price it will charge an internal division is the market

price. See the correct answer for a complete explanation.

B. Because the alpha division is operating at capacity, the minimum price it will charge an internal division is the marketprice. See the correct answer for a complete explanation.

C. Because the alpha division is operating at capacity, the minimum price it will charge an internal division is themarket price. See the correct answer for a complete explanation.

D. Because the alpha division is operating at capacity, the minimum price it will charge an internal division isthe market price. The market price per unit is $50. If they were to charge less than the market price they wouldbe losing money since they could sell that same unit to a different customer at a higher price.

Question 136 - CIA 1193 IV.20 - Responsibility Centers and Reporting Segments

 A and B are autonomous divisions of a corporation. They have no beginning or ending inventories, and the number ofunits produced is equal to the number of units sold. Following is financial information relating to the two divisions:  D i v i s i o n

  A BSales $150,000$400,000Other revenue 10,000 15,000

Part 1 : 10/05/14 22:20:20

(c) HOCK international, page 91

Page 92: Hock_Performance Mgmt

8/16/2019 Hock_Performance Mgmt

http://slidepdf.com/reader/full/hockperformance-mgmt 92/152

Direct materials 30,000 65,000Direct labor 20,000 40,000Variable factory overhead 5,000 15,000Fixed factory overhead 25,000 55,000Variable selling and administrative expense 15,000 30,000Fixed sell ing and administrative expense 35,000 60,000Central corporate expenses (allocated) 12,000 20,000

What is the total contribution to corporate profits generated by Division A before allocation of central corporateexpenses?

 A. $80,000B. $18,000C. $20,000D. $30,000

 A. In this answer total S&A expenses are not deducted. However, those expenses should be included in thecalculation.

B. In this answer the central corporate expenses allocated are deducted. However, those expenses should not be

included in calculation.

C. In this answer the other revenues are not added. These revenues should be included in the calculation.

D. Department A's contribution to the corporate profits is calculated as all revenues minus all expensesexcept the allocated central corporate expenses. It is calculated as follows: $150,000 + $10,000 − $30,000 −$20,000 − $5,000 − $25,000 − $15,000 − $35,000 = $30,000.

Question 137 - CIA 1193 IV.18 - Responsibility Centers and Reporting Segments

One department of an organization, Final Assembly, is purchasing subcomponents from another department, Materials

Fabrication. The price that wil l be charged to Final Assembly by Materials Fabrication is to be determined. Outsidemarket prices for the subcomponents are available. Which of the following is the most correct statement regarding amarket-based transfer price?

 A. Corporate politics is more of a factor in a market-based transfer price than with other methods.B. Marginal production cost transfer prices provide incentives to use otherwise idle capacity.C. Market transfer prices provide an incentive to use otherwise idle capacity.D. Overall long term competitiveness is enhanced with a market-based transfer price.

 A. In all methods of transfer pricing, corporate politics will play a role. In market based transfer pricing, corporatepolitics are probably a little bit less as the market is setting the price and the market is outside the influence of thecompany, or of individuals within it.

B. If the marginal cost of production is used as the transfer price there is no incentive to use otherwise idle facilities

because there will not be any contribution received from the units that are produced. Also, marginal costs of productionwould not be the transfer price if a market based transfer price is used.

C. A transfer price equal to the market price will provide incentives for the producing department to increaseproduction, but will not provide any incentive for the purchasing department to purchase internally.

D. This is a very general, but true, statement. A market based transfer price will lead to the greatest good forthe company in the long-term. Other, artificial, transfer prices will lead to behavior that is not in the bestinterest of the company as a whole.

Part 1 : 10/05/14 22:20:20

(c) HOCK international, page 92

Page 93: Hock_Performance Mgmt

8/16/2019 Hock_Performance Mgmt

http://slidepdf.com/reader/full/hockperformance-mgmt 93/152

Question 138 - ICMA 10.P1.101 - Manufacturing Input Variances -- Materials and Labor 

 A company has a direct labor price variance that is favorable. Of the following the most serious concern the companymay have about this variance is that

 A. the production manager may not be using human resources as efficiently as possible.B. actual production is less than budgeted production.C. the cause of the favorable variance may result in other larger unfavorable variances in the value chain.D. the circumstances giving rise to the favorable variance will not continue in the future.

 A. The labor efficiency variance measures whether human resources are being used efficiently. The labor price (rate)variance does not.

B. A direct labor price variance is not caused by actual production volume being less than budgeted production volume.

C.

A company’s "value chain" is its chain of activities for transforming inputs into the outputs that customers

value. This process of transformation includes all of the primary activities (business functions) that add valueto the product or service, as well as support activities. It involves functions from R&D through production tomarketing and sales, and on to customer service, including the activities that play supporting roles such asinformation systems, materials management, and human resources.

When a company has a favorable labor price (rate) variance, it is because the company has paid hourly wagerates that are lower than the standard wage rates. Usually, that occurs when the company has employedworkers who are less qualified than the workers they expected to have. Use of less qualified workers can leadto a higher probability that the products produced and sold will be of lower quality and have more defects.Lower quality products and higher defective rates will lead to increased costs for customer service, as thecustomers who buy the products will have more difficulty with them. It can also lead to higher warranty costs,because more products will need to be repaired or replaced for customers during the warranty period.

The amount gained from the favorable labor price variance, and possibly even more, may be lost throughgreater unfavorable variances further down the value chain.

D. There is no reason to believe that circumstances giving rise to the favorable variance will not continue in th future.

Question 139 - ICMA 10.P1.094 - Manufacturing Input Variances -- Overhead

Lee Manufacturing uses a standard cost system with overhead applied based on direct labor hours. The manufacturingbudget for the production of 5,000 units for the month of June included 10,000 hours of direct labor at $15 per hour, or$150,000. During June, 4,500 units were produced, using 9,600 direct labor hours, incurring $39,360 of variableoverhead, and showing a variable overhead efficiency variance of $2,400 unfavorable. The standard variable overhead

rate per direct labor hour was

 A. $4.10B. $3.85C. $6.00D. $4.00

 A. This is the actual incurred variable overhead of $39,360 divided by the actual number of direct labor hours used(9,600).

Part 1 : 10/05/14 22:20:20

(c) HOCK international, page 93

Page 94: Hock_Performance Mgmt

8/16/2019 Hock_Performance Mgmt

http://slidepdf.com/reader/full/hockperformance-mgmt 94/152

Page 95: Hock_Performance Mgmt

8/16/2019 Hock_Performance Mgmt

http://slidepdf.com/reader/full/hockperformance-mgmt 95/152

B. Production supervisors found several significant fluctuations in manufacturing volume, with short-term increases inoutput being followed by rapid, sustained declines.C. A review of performance reports revealed the presence of many unfavorable efficiency variances.D. Management noted that minimal incentive bonuses have been paid in recent periods.

A.

"Tight" standards are standards that are very demanding. For example, if a job can be done in two hours onlyif everything works perfectly and absolutely nothing goes wrong, then a labor standard of two hours may betoo tight because things can go wrong. Machines can break down, jobs can get interrupted, there could be apower failure, or any number of things could delay the completion of the job.

Whether the budgeting process was well-defined or not well defined is not relevant to the standards beingused within the company. Furthermore, if the standards were developed using a bottom-up philosophy, thereis little chance that they will be too tight. If anything, they might be too loose, because the people closest tothe production process might build more extra time than necessary into the labor standards, in order to makesure the standards can be met. Therefore, a bottom-up philosophy of budgeting would not result in too-tightlabor standards, so this is inconsistent with the consultant’s conclusion.

B.

"Tight" standards are standards that are very demanding. For example, if a job can be done in two hours only ifeverything works perfectly and absolutely nothing goes wrong, then a labor standard of two hours may be too tightbecause things can go wrong. Machines can break down, jobs can get interrupted, there could be a power failure, orany number of things could delay the completion of the job.

Fluctuations in manufacturing volume with short-term increases in output being followed by rapid, sustained declines inoutput can indicate an all-out effort to produce the quantities demanded followed by a reining in of costs in order tomeet unrealistic labor standards. Therefore, this is consistent with the consultant’s conclusion, not inconsistent.

C.

"Tight" standards are standards that are very demanding. For example, if a job can be done in two hours only ifeverything works perfectly and absolutely nothing goes wrong, then a labor standard of two hours may be too tight

because things can go wrong. Machines can break down, jobs can get interrupted, there could be a power failure, orany number of things could delay the completion of the job.

Many unfavorable efficiency (quantity of labor used) variances could be a sign of an unrealistic standard. Therefore,this is consistent with the consultant's conclusion, not inconsistent. If the company continually uses more hours thananticipated, revisions to the labor standard may be necessary.

D.

"Tight" standards are standards that are very demanding. For example, if a job can be done in two hours only ifeverything works perfectly and absolutely nothing goes wrong, then a labor standard of two hours may be too tightbecause things can go wrong. Machines can break down, jobs can get interrupted, there could be a power failure, orany number of things could delay the completion of the job.

Incentive bonuses are generally based on successful performance which can be measured using standards. Ifbonuses are not being paid out, performance has not been what was expected and perhaps the expectations havebeen unrealistic. Therefore, this is consistent with the consultant's conclusion, not inconsistent.

Question 141 - CMA 693 3.19 - Manufacturing Input Variances -- Overhead

Tiny Tykes Corporation had the following activity relating to its fixed and variable overhead for the month of July.

Part 1 : 10/05/14 22:20:20

(c) HOCK international, page 95

Page 96: Hock_Performance Mgmt

8/16/2019 Hock_Performance Mgmt

http://slidepdf.com/reader/full/hockperformance-mgmt 96/152

Actual costs  Fixed overhead $120,000Variable overhead 80,000 

Flexible budgetVariable overhead 90,000

 Applied costsFixed overhead 125,000 Variable overhead spending variance 2,000FProduction volume variance 5,000U

If the budgeted rate for applying variable manufacturing overhead was $20 per direct labor hour, how efficient orinefficient was Tiny Tykes Corporation in terms of using direct labor hours as an activity base?

 A. 100 direct labor hours efficient.B. 400 direct labor hours efficient.C. 100 direct labor hours inefficient.D. 400 direct labor hours inefficient.

 A. This is the Variable Overhead Spending Variance divided by the budgeted rate for applying variable manufacturingoverhead of $20 per direct labor hour. Since the question is asking about the company's efficiency, the answer to thisquestion is the Variable Overhead Efficiency Variance divided by the budgeted rate for applying variablemanufacturing overhead of $20 per direct labor hour.

B.

The answer to this question is the Variable Overhead Efficiency Variance divided by the budgeted rate forapplying variable manufacturing overhead of $20 per direct labor hour. This will tell us the difference betweenthe actual quantity of direct labor hours used for the actual output and the standard quantity of direct laborhours allowed for the actual output. That, in turn, will tell us how efficient or inefficient the company's use ofdirect labor was.

Another way of looking at it is this: Since (AQ − SQ) × SP equals the Variable Overhead Efficiency Variance,the Variable Overhead Efficiency Variance divided by the standard price ($20) equals the difference betweenAQ and SQ. And the difference between AQ and SQ is what we need to find to answer the question.

The Variable Overhead Efficiency Variance is the difference between the Total Variable Overhead Variance andthe Variable Overhead Spending Variance (because the Variable Overhead Spending Variance plus theVariable Overhead Efficiency Variance equals the Total Variable Overhead Variance.)

The Total Variable Overhead Variance is equal to actual incurred variable overhead of $80,000 minus flexiblebudget variable overhead of $90,000, or $(10,000) Favorable.The problem tells us that the Variable OverheadSpending Variance is $(2,000) Favorable.

Therefore, the Variable Overhead Efficiency Variance must be $(10,000) minus $(2,000), which is $(8,000)Favorable.

The Variable Overhead Efficiency Variance of $(8,000) divided by the budgeted rate for applying variablemanufacturing overhead of $20 per direct labor hour equals (400). This means the actual quantity of directlabor hours (AQ) used for the actual output was 400 hours less than the standard quantity of direct laborhours allowed for the actual output (SQ).

Thus, Tiny Tykes was 400 hours efficient in its use of direct labor hours.

C. This is the Variable Overhead Spending Variance divided by the budgeted rate for applying variable manufacturing

Part 1 : 10/05/14 22:20:20

(c) HOCK international, page 96

Page 97: Hock_Performance Mgmt

8/16/2019 Hock_Performance Mgmt

http://slidepdf.com/reader/full/hockperformance-mgmt 97/152

overhead of $20 per direct labor hour calculated using a positive (unfavorable) variance amount. Since the question isasking about the company's efficiency, the answer to this question is the Variable Overhead Efficiency Variancedivided by the budgeted rate for applying variable manufacturing overhead of $20 per direct labor hour. Furthermore,the Variable Overhead Efficiency Variance is a negative (favorable) amount.

D. This is the Variable Overhead Efficiency Variance divided by the budgeted rate for applying variable manufacturingoverhead of $20 per direct labor hour calculated using a positive (unfavorable) variance amount. The Variable

Overhead Efficiency Variance is a negative (favorable) amount.

Question 142 - CMA 691 3.29 - Performance Measures

The selection of the denominator in the return on investment (ROI) formula is critical to the measure's effectiveness.Which denominator is crit icized because it combines the effects of operating decisions made at one level of theorganization with financing decisions made at another organizational level?

 A. Total assets employed.B. Total assets available.C. Shareholders' equity.

D. Working capital.

 A. This does not take into account the source of financing since it includes all assets that are employed, no matter thesource of the financing.

B. This does not take into account the source of financing since it includes all assets that are available, no matter thesource of financing.

C. Shareholders' equity is not a good denominator to use in the ROI calculation because it takes into accountdecisions made about financing that are not made at the manager level. The decision to use equity for thefinancing of assets is one that is made at the highest levels in the organization.

D. This does not take into account the source of financing of assets, as it will include those assets financed by debtand equity.

Question 143 - CIA 589 IV.14 - Sales and Market Variances

The following data are available for July:  Budget ActualSales 40,000 units 42,000 unitsSelling price $6 per unit$5.70 per unitVariable cost$3.50 per unit$3.40 per unit

What is the sales quantity variance for July?

 A. $5,000 favorable.B. $12,000 unfavorable.C. $12,600 unfavorable.D. $4,600 favorable.

A.

The sales quantity variance, which is the same thing as the sales volume variance, measures the impact of thedifference in sales volume between the actual results and the static budget. The sales quantity/volume

Part 1 : 10/05/14 22:20:20

(c) HOCK international, page 97

Page 98: Hock_Performance Mgmt

8/16/2019 Hock_Performance Mgmt

http://slidepdf.com/reader/full/hockperformance-mgmt 98/152

variance for a single product or for a single product firm can be calculated for each variable income andexpense item as well as for the contribution margin. If a question does not specify which line to use, as thisone does not, use the contribution margin line.

For the contribution margin line, it is calculated as follows: (Actual Sales Volume − Budgeted Sales Volume) ×Budgeted Contribution per Unit, or (AQ − SQ) × SP. The "AQ," actual quantity, is 42,000. The "SQ," budgetedquantity, is 40,000. The "SP," budgeted price, is the budgeted contribution margin per unit. That is $2.50 per

unit ($6 − $3.50). Thus the variance is (42,000 − 40,000) × $2.50, or $5,000. A positive variance for an incomeline or for the contribution margin line is a favorable variance, because it means the actual was higher thanthe budget.

B. The variance is favorable as the quantity actually sold (42,000) is greater than the budgeted quantity (40,000). Seethe correct answer for a complete explanation.

C. The variance is favorable as the quantity actually sold (42,000) is greater than the budgeted quantity (40,000). Seethe correct answer for a complete explanation.

D.

The sales quantity variance, which is the same thing as the sales volume variance, measures the impact of thedifference in sales volume between the actual results and the static budget. The sales quantity/volume variance for asingle product or for a single product firm can be calculated for each variable income and expense item as well as forthe contribution margin. If a question does not specify which line to use, as this one does not, use the contributionmargin line.

For the contribution margin line, it is calculated as follows: (Actual Sales Volume − Budgeted Sales Volume) ×Budgeted Contribution per Unit, or (AQ − SQ) × SP. The "AQ," actual quantity, is 42,000. The "SQ," budgeted quantity,is 40,000. The "SP," budgeted price, is the budgeted contribution margin per unit. That is $2.50 per unit ($6 − $3.50).

This answer results from using the actual contribution margin per unit instead of the budgeted contribution margin perunit.

Question 144 - CIA 1191 IV.17 - Responsibility Centers and Reporting Segments

The receipt of raw materials used in the manufacture of products and the shipping of finished goods to customers isunder the control of the warehouse supervisor. The warehouse supervisor's time is spent approximately 60% onreceiving activities and 40% on shipping activities. Separate staffs are employed for the receiving and shippingoperations. The labor-related costs for the warehousing function are as follows:Warehouse supervisor's salary $ 40,000Receiving clerks' wages 75,000Shipping clerks' wages 55,000Employee benefit costs (30% of wage and salary costs) 51,000

  Total labor-related costs $221,000

The company employs a responsibility accounting system for performance reporting purposes. The costs are classifiedon the report as period or product costs. The total labor-related costs that would be listed on the responsibilityaccounting performance report as product costs under the control of the warehouse supervisor for the warehousingfunction would be

 A. $130,000B. $169,000C. $97,500D. $128,700

 A. This answer is a total of the receiving clerks' wages and the shipping clerks' wages. The answer to this question

Part 1 : 10/05/14 22:20:20

(c) HOCK international, page 98

Page 99: Hock_Performance Mgmt

8/16/2019 Hock_Performance Mgmt

http://slidepdf.com/reader/full/hockperformance-mgmt 99/152

should be the total product costs that the warehouse supervisor can control.

B. This answer is a total of the receiving clerks' wages and the shipping clerks' wages, plus their related benefits. Theanswer to this question should be the total product costs that the warehouse supervisor can control.

C.

In a responsibility accounting system, the only costs that a manager should be evaluated on are those coststhat the manager can control. The answer to this question must include only costs that are (1) product costsand (2) costs that the warehouse supervisor can control.

The warehouse supervisor can control the receiving clerks' wages and the shipping clerks' wages. However,this problem asks for the total labor-related costs that would be listed on the responsibility accountingperformance report as product costs.

Product costs include all the costs that are involved in acquiring or making a product. These costs are alsocalled inventoriable costs. For a manufacturer, product costs include direct materials, direct labor, andmanufacturing overhead. Receiving costs are product costs because they are necessary for manufacturingthe product. Labor costs for receiving are classified as indirect labor, and so they are consideredmanufacturing overhead and are a product cost.

Although receiving costs are product costs, shipping costs are selling costs, and so they are classified asperiod costs, not product costs. Therefore, even though the warehouse supervisor controls the shippingclerks' wages, those wages are period costs, not product costs, so they must be excluded.

The supervisor's salary is a product cost because it is indirect labor. However, the supervisor cannot controlhis or her own salary (it is controlled by someone higher up in the organization) and so it is not included as aproduct cost under the control of the warehouse supervisor.

Thus, the only labor-related costs listed that are (1) product costs and (2) costs that the warehouse supervisorcan control are the costs for the receiving clerks. Those will include $75,000 for the receiving clerks' wagesplus 30% of the receiving clerks' wages for benefits, which is $22,500 ($75,000 × 0.30), for a total of $97,500.

D. This answer results from including the receiving clerks' wages and related benefits, plus 60% of the supervisor's

salary and related benefits. This incorrectly assumes that the supervisor's salary is controllable by the supervisor. Theanswer to this question should be the total product costs that the warehouse supervisor can control; and thewarehouse supervisor is not able to control his/her own salary.

Question 145 - CMA 694 3.20 - Responsibility Centers and Reporting Segments

Under a standard cost system, the materials price variances are usually the responsibility of the

 A. Purchasing manager.B. Production manager.C. Cost accounting manager.D. Sales manager.

A. The materials price variance is the difference between the actual price and the standard price of materialmultiplied by the quantity actually consumed by production. The purchasing manager is responsible forpurchasing activity where price is negotiated and is therefore usually held responsible for the material pricevariance.

B. The materials price variance is the difference between the actual price and the standard price of material multipliedby the quantity actually consumed by production. The production manager is not involved in the purchasing activitywhere the price is negotiated.

Part 1 : 10/05/14 22:20:20

(c) HOCK international, page 99

Page 100: Hock_Performance Mgmt

8/16/2019 Hock_Performance Mgmt

http://slidepdf.com/reader/full/hockperformance-mgmt 100/152

C. The materials price variance is the difference between the actual price and the standard price of material multipliedby the quantity actually consumed by production. The cost accounting manager is not involved in the purchasingactivity where the price is negotiated.

D. The materials price variance is the difference between the actual price and the standard price of material multipliedby the quantity actually consumed by production. The sales manager is not involved in the purchasing activity where

the price is negotiated.

Question 146 - CIA 589 IV.16 - Responsibility Centers and Reporting Segments

Division A of a company is currently operating at 50% capacity. It produces a single product and sells all its productionto outside customers for $13 per unit. Variable costs are $7 per unit, and fixed costs are $6 per unit at the currentproduction level. Division B, which currently purchases this product from an outside supplier for $12 per unit, would liketo purchase the product from Division A. Division A will operate at 80% capacity to meet outside customers' andDivision B's demand. What is the minimum price that Division A should charge Division B for this product?

 A. $12.00 per unit.

B. $9.60 per unit.C. $7.00 per unit.D. $13.00 per unit.

 A. When operating below capacity, the minimum price that the producing department will sell for is the variable cost ofproduction.

B. When operating below capacity, the minimum price that the producing department will sell for is the variable cost ofproduction.

C. When operating below capacity, the minimum price that the producing department will sell for is thevariable cost of production. This is $7 in this question.

D. When operating below capacity, the minimum price that the producing department will sell for is the variable cost ofproduction.

Question 147 - CMA 693 3.20 - Manufacturing Input Variances -- Overhead

Tiny Tykes Corporation had the following activity relating to its fixed and variable overhead for the month of July.Actual costs  Fixed overhead $120,000Variable overhead 80,000 

Flexible budget

Variable overhead 90,000 

Applied costsFixed overhead 125,000 Variable overhead spending variance 2,000FProduction volume variance 5,000U

The fixed overhead efficiency variance is

Part 1 : 10/05/14 22:20:20

(c) HOCK international, page 100

Page 101: Hock_Performance Mgmt

8/16/2019 Hock_Performance Mgmt

http://slidepdf.com/reader/full/hockperformance-mgmt 101/152

 A. $3,000 unfavorable.B. $5,000 favorable.C. $3,000 favorable.D. Never a meaningful variance.

 A. There is no such thing as a fixed overhead efficiency variance because fixed costs are not related to levels of outputand therefore are unable to be used efficiently or inefficiently.

B. There is no such thing as a fixed overhead efficiency variance because fixed costs are not related to levels of outputand therefore are unable to be used efficiently or inefficiently.

C. There is no such thing as a fixed overhead efficiency variance because fixed costs are not related to levels of outputand therefore are unable to be used efficiently or inefficiently.

D. There is no such thing as a fixed overhead efficiency variance because fixed costs are not related to levelsof output and therefore are unable to be used efficiently or inefficiently.

Question 148 - ICMA 10.P1.139 - Performance Measures

Performance results for four geographic divisions of a manufacturing company are shown below.

DivisionTarget Returnon Investment

 Actual Returnon Investment

Return onSales

 A 18% 18.1% 8%B 16% 20.0% 8%C 14% 15.8% 6%D 12% 11.0% 9%

The division with the best performance is

 A. Division C.

B. Division B.C. Division A.D. Division D.

 A. Division C's actual return on investment exceeded its target return on investment, but not by as much as some otherdivisions. Furthermore, its return on sales was not as great as that of some other divisions. Therefore, it cannot qualifyas the division with the best performance.

B. Division B's actual return on investment exceeded its target return on investment by the greatest amount,among the divisions that did exceed their targets. In addition, among the divisions that exceeded their targets,Division B had one of the highest returns on sales. Therefore, it is the division with the best overallperformance.

C. Division A's actual return on investment exceeded its target return on investment and among the divisions that didexceed their targets, it had one of the highest returns on sales. However, it did not exceed its target return oninvestment by as much as some other divisions did. Therefore, it cannot qualify as the division with the bestperformance.

D. Division D's actual return on investment was below its target return on investment. Its return on sales was thehighest of all the divisions, but since it did not meet its target return on investment, it cannot qualify as the division withthe best performance.

Part 1 : 10/05/14 22:20:20

(c) HOCK international, page 101

Page 102: Hock_Performance Mgmt

8/16/2019 Hock_Performance Mgmt

http://slidepdf.com/reader/full/hockperformance-mgmt 102/152

Question 149 - CMA 1290 3.6 - Manufacturing Input Variances -- Overhead

Franklin Glass Works' production budget for the year ended November 30 was based on 200,000 units. Each unitrequires two standard hours of labor for completion. Total overhead was budgeted at $900,000 for the year, and thefixed overhead rate was estimated to be $3.00 per unit. Both fixed and variable overhead are assigned to the producton the basis of direct labor hours. The actual data for the year ended November 30 are presented as follows.

 Actual production in units 198,000 Actual direct labor hours 440,000 Actual variable overhead $352,000 Actual fixed overhead $575,000

Franklin's variable overhead efficiency variance for the year is

 A. $66,000 unfavorable.B. $33,000 favorable.C. $35,520 favorable.D. $33,000 unfavorable.

 A. This answer was calculated using an incorrect labor rate of $1.50. It was caused by using the wrong number ofhours budgeted (200,000) for production. In fact, 400,000 hours was scheduled for production, as each unit requirestwo standard hours of labor and production was planned to be 200,000 units.

B.

The variable overhead efficiency variance is calculated as: (Actual Activity Level of VOH application base actually used− Standard Activity Level of application base allowed for Actual Output) × Standard Application Rate. A positive result isan unfavorable variance, while a negative result is a favorable variance, since this is a cost item. The variance is anunfavorable variance, not a favorable one.

C.

This is not the correct answer. Please see the correct answer for a complete explanation.

We have been unable to determine how to calculate this incorrect answer choice. If you have calculated it, please letus know how you did it so we can create a full explanation of why this answer choice is incorrect. Please send us anemail at [email protected]. Include the full Question ID number and the actual incorrect answer choice --not its letter, because that can change with every study session created. The Question ID number appears in the upperright corner of the ExamSuccess screen. Thank you in advance for helping us to make your HOCK study materialsbetter.

D.

The variable overhead efficiency variance determines the amount of the total variance that was caused by adifferent usage of the allocation base (machine hours or direct labor hours) than was expected for the actualoutput (i.e. the standard hours for the actual output). The variable overhead efficiency variance is calculatedas: (Actual Activity Level of VOH application base actually used − Standard Activity Level of application baseallowed for Actual Output) × Standard Application Rate.

Total budgeted overhead (variable and fixed) was $900,000. The fixed overhead rate was estimated to be $3.00per unit, thus the budgeted amount of fixed overhead for the budgeted level of production of 200,000 unitswas $600,000 ($3 × 200,000). Therefore, the budgeted amount of variable overhead was $300,000 ($900,000 −$600,000). Each unit requires two standard hours of labor, hence the variable overhead rate per hour was$0.75 ($300,000 ÷ (200,000 × 2)). The standard hours were 396,000 (198,000 × 2 hours per unit) and the actualhours were 440,000. Now we can calculate the variable overhead efficiency variance as follows: (440,000 −396,000) × $0.75 = 33,000 unfavorable variance.

Part 1 : 10/05/14 22:20:20

(c) HOCK international, page 102

Page 103: Hock_Performance Mgmt

8/16/2019 Hock_Performance Mgmt

http://slidepdf.com/reader/full/hockperformance-mgmt 103/152

Question 150 - CMA 687 4.16 - Sales and Market Variances

The following information is available for the Mitchelville Products Company for the month of July.

 Master Budget Actual

Units 4,000 3,800Sales revenue $60,000$53,200Variable manufacturing costs 16,000 19,000Fixed manufacturing costs 15,000 16,000Variable selling and administrative expense 8,000 7,600Fixed selling and administrative expense 9,000 10,000

The contribution margin volume variance for the month of July would be

 A. $6,800 unfavorable.B. $1,800 unfavorable.C. $400 unfavorable.D. $200 favorable.

 A. This is the difference between actual and budgeted revenue. See the correct answer for a complete explanation.

B. The contribution margin volume variance is calculated as follows: (Actual Quantity − Budgeted Quantity) ×Budgeted Unit Contribution Margin. The actual quantity is 3,800 and the budgeted quantity is 4,000. Thebudgeted contribution margin per unit is $9 [($60,000 sales revenue − $16,000 variable manufacturing costs −$8,000 variable S&A costs) ÷ 4,000 units]. Therefore, the contribution margin volume variance is (3,800 − 4,000)× $9 = $(1,800) unfavorable.

C. This is the difference between the actual and budgeted variable S&A costs. This number does not mean anything.

D. This is the difference between budgeted and actual units. See the correct answer for a complete explanation.

Question 151 - CIA 587 IV.15 - Responsibility Centers and Reporting Segments

Overtime conditions and pay were recently set by the personnel department. The production department has justreceived a request for a rush order from the sales department. The production department protests that additionalovertime costs will be incurred as a result of the order. The sales department argues that the order is from animportant customer. The production department processes the order. To control costs, which department should neverbe charged with the overtime costs generated as a result of the rush order?

 A. Shared by production department and sales department.B. Production department.C. Personnel department.D. Sales department.

 A. The overtime premium should be charged to the department that is responsible for the overhead being worked. Ifthe reason that overtime is worked is simply that the volume of work requires it, the overtime premium is charged tooverhead and allocated to all of the units produced. Given the situation in this question, it is reasonable that theovertime premium be charged to both the production and sales departments.

B. The overtime premium should be charged to the department that is responsible for the overhead being worked. Ifthe reason that overtime is worked is simply that the volume of work requires it, the overtime premium is charged tooverhead and allocated to all of the units produced. Given the situation in this question, it is reasonable that at leastsome of the overtime premium be charged to the production department.

Part 1 : 10/05/14 22:20:20

(c) HOCK international, page 103

Page 104: Hock_Performance Mgmt

8/16/2019 Hock_Performance Mgmt

http://slidepdf.com/reader/full/hockperformance-mgmt 104/152

C. The overtime premium should be charged to the department that is responsible for the overhead beingworked. If the reason that overtime is worked is simply that the volume of work requires it, the overtimepremium is charged to overhead and allocated to all of the units produced. Because the personnel departmentwould not have been able to cause or prevent the overtime work, the costs should not be charged to thepersonnel department.

D. The overtime premium should be charged to the department that is responsible for the overhead being worked. Ifthe reason that overtime is worked is simply that the volume of work requires it, the overtime premium is charged tooverhead and allocated to all of the units produced. Given the situation in this question, it is reasonable that at leastsome of the overtime premium be charged to the sales department.

Question 152 - ICMA 10.P1.085 - Variance Analysis Concepts

 A major disadvantage of a static budget is that

 A. it is more difficult to develop than a flexible budget.B. variances tend to be smaller than when flexible budgeting is used.

C. variances are more difficult to compute than when flexible budgeting is used.D. it is made for only one level of activity.

 A. A static budget is not more difficult to develop than a flexible budget. A flexible budget is based on the static budget,but sales and production activity assumptions used to develop the static budget are changed to actual sales andproduction activity for the flexible budget, and the budgeted amounts that vary with activity are adjusted accordingly inthe flexible budget.

B. Variances would probably tend to be larger when actual is compared to the static budget, because the variancesinclude variances due to variations between budgeted activity levels and actual activity levels. When actual amountsare compared with flexible budget amounts, the variances will not include variances caused by variations betweenactual and budgeted activity, whereas when the static budget is used, the variances will include variances caused byvariations between actual and budgeted activity.

C. Variances are not more difficult to compute when a static budget rather than when a flexible budget is used tocompare actual to budgeted amounts.

D.

A static budget is developed for one specific activity (sales or production) level. When variance reportscomparing actual results to budgeted results in the static budget are prepared and causes for the variancesare reported, one of the causes will always be that actual volume was different from planned sales volume,because actual activity will never be exactly equal to the budgeted activity.

Since variances due to volume variations are expected, it does not make much sense to continue reportingthem on the variance report as causes of variances. It is more important to focus on variances caused byother factors.

For example, a variance caused by an increase in the cost of direct labor above what is expected for the actualproduction could signal a problem in production and should be investigated. But an increase in the cost ofdirect labor above what is expected for the budgeted production that is caused by increased production is nota production problem, if the cost of the direct labor per unit is equal to the budgeted amount per unit for thenumber of units actually produced.

The use of a static budget can create difficulty in isolating the causes of variances that need to beinvestigated from those that need no investigation.

Part 1 : 10/05/14 22:20:20

(c) HOCK international, page 104

Page 105: Hock_Performance Mgmt

8/16/2019 Hock_Performance Mgmt

http://slidepdf.com/reader/full/hockperformance-mgmt 105/152

Question 153 - CMA 1294 3.26 - Manufacturing Input Variances -- Overhead

Water Control Inc. manufactures water pumps and uses a standard cost system. The standard factory overhead costsper water pump are based on direct labor hours and are as follows:

Variable overhead (4 hours at $8/hour) - $32Fixed overhead (4 hours at $5/hour*) - $20Total overhead cost per unit - $52* Based on a capacity of 100,000 direct labor hours per month.

The following additional information is available for the month of November:

22,000 pumps were produced although 25,000 had been scheduled for production.94,000 direct labor hours were worked at a total cost of $940,000.The standard direct labor rate is $9 per hour.The standard direct labor time per unit is 4 hours.Variable overhead costs were $740,000.Fixed overhead costs were $540,000.

The fixed overhead spending variance for November was:

 A. $40,000 unfavorable.B. $240,000 unfavorable.C. $70,000 unfavorable.D. $460,000 unfavorable.

A.

The fixed overhead budget/spending variance is the difference between the actual fixed overhead incurredand the budgeted fixed overhead costs. The actual amount of fixed overhead costs was $540,000.

The budgeted amount of fixed overhead was $500,000 ($5 of fixed overhead per labor hour multiplied by100,000 budgeted labor hours). We use the budgeted amount of labor hours (100,000) because we need tocalculate what the total budgeted fixed overhead amount was that the company used in its calculation of thecost per direct labor hour. To calculate the budgeted cost per direct labor hour, the company divided the totalbudgeted fixed cost by the budgeted number of labor hours. Therefore, to find the budgeted fixedmanufacturing cost, we reverse the process and multiply the budgeted cost per labor hour by the number ofbudgeted labor hours. The amount of budgeted fixed cost is the same for any level of production. Budgetedfixed cost is the same in the flexible budget as it is in the static budget. So in calculating the budgeted fixedmanufacturing overhead, it does not matter what the actual level of production was.

The fixed overhead budget/spending variance is $40,000 unfavorable ($540,000 − $500,000). The actual fixedoverhead costs incurred were higher than the budgeted amount, so the variance is unfavorable.

B. This amount is the difference between the actual variable overhead and the budgeted fixed overhead. See thecorrect answer for a complete explanation.

C.

This answer results from using the actual amount of labor hours used (94,000) to calculate the budgeted fixedoverhead costs instead of the budgeted labor hours (100,000).

We use the budgeted amount of labor hours (100,000) because we need to calculate what the total budgeted fixedoverhead amount was that the company used in its calculation of the cost per direct labor hour. To calculate thebudgeted cost per direct labor hour, the company divided the total budgeted fixed cost by the budgeted number oflabor hours. Therefore, to find the budgeted fixed manufacturing cost, we reverse the process and multiply the

Part 1 : 10/05/14 22:20:20

(c) HOCK international, page 105

Page 106: Hock_Performance Mgmt

8/16/2019 Hock_Performance Mgmt

http://slidepdf.com/reader/full/hockperformance-mgmt 106/152

budgeted cost per labor hour by the number of budgeted labor hours. The amount of budgeted fixed cost is the samefor any level of production. Budgeted fixed cost is the same in the flexible budget as it is in the static budget. So incalculating the budgeted fixed manufacturing overhead, it does not matter what the actual level of production was.

D.

This is not the correct answer. Please see the correct answer for a complete explanation.

We have been unable to determine how to calculate this incorrect answer choice. If you have calculated it, please letus know how you did it so we can create a full explanation of why this answer choice is incorrect. Please send us anemail at [email protected]. Include the full Question ID number and the actual incorrect answer choice --not its letter, because that can change with every study session created. The Question ID number appears in the upperright corner of the ExamSuccess screen. Thank you in advance for helping us to make your HOCK study materialsbetter.

Question 154 - CMA 1289 4.3 - Manufacturing Input Variances -- Overhead

 A fixed overhead volume variance based on standard direct labor hours measures:

 A. Fixed overhead efficiency.B. Deviation from standard direct labor hour capacity.C. Fixed overhead use.D. Deviation from the normal, or denominator, level of direct labor hours.

 A. There is no fixed overhead efficiency variance because fixed costs are not related to levels of output and thereforeare unable to be used efficiently or inefficiently.

B. The fixed overhead volume variance does not relate to the capacity of standard direct labor hours.

C. The fixed overhead volume variance does not relate to the fixed overhead usage.

D.

The fixed overhead volume variance (also called the fixed overhead production-volume variance) is thebudgeted amount of fixed overhead (in the static budget) minus the amount of fixed overhead applied(standard rate × standard input for the actual level of output).

The budgeted amount of fixed overhead is what was projected as the total amount of fixed overhead to beincurred during the period at the beginning of the year when the budget was developed.

Traditionally, overhead is applied to individual products throughout the year using some standard allocationbasis, usually direct labor hours, machine hours, materials cost, units of production, or some similar measurethat can be measured and calculated. The measure used is called the activity base. A predetermined rate iscalculated by dividing the budgeted amount of manufacturing overhead by the budgeted activity level. Thebudgeted activity level is the number of units of the activity base (labor hours, machine hours, etc.) allowed

for the expected production during the coming year. As production continues throughout the year, the amountof the activity base allowed for the amount of product actually produced is multiplied by the predeterminedrate to calculate the amount of overhead to be applied.

The fixed overhead volume variance is caused by the actual production level being different from theproduction level (called the denominator level) used to calculate the budgeted fixed overhead rate.

If the amount of overhead applied is greater than the budgeted amount, the variance will be favorable becauseit means that the actual level of production was greater than the budgeted level of production. That is goodbecause it means the facilities are being more fully utilized. If the amount of overhead applied is less than the

Part 1 : 10/05/14 22:20:20

(c) HOCK international, page 106

Page 107: Hock_Performance Mgmt

8/16/2019 Hock_Performance Mgmt

http://slidepdf.com/reader/full/hockperformance-mgmt 107/152

budgeted overhead, the variance will be unfavorable because it means that actual production was lower thanthe budgeted production level.

Question 155 - CMA 1290 3.7 - Manufacturing Input Variances -- Overhead

Franklin Glass Works' production budget for the year ended November 30 was based on 200,000 units. Each unitrequires two standard hours of labor for completion. Total overhead was budgeted at $900,000 for the year, and thefixed overhead rate was estimated to be $3.00 per unit. Both fixed and variable overhead are assigned to the producton the basis of direct labor hours. The actual data for the year ended November 30 are presented as follows. Actual production in units 198,000 Actual direct labor hours 440,000 Actual variable overhead $352,000 Actual fixed overhead $575,000

Franklin's variable overhead spending variance for the year is:

 A. $20,000 favorable.

B. $20,000 unfavorable.C. $22,000 unfavorable.D. $19,800 favorable.

 A.

This is not the correct answer. Please see the correct answer for a complete explanation.

We have been unable to determine how to calculate this incorrect answer choice. If you have calculated it, please letus know how you did it so we can create a full explanation of why this answer choice is incorrect. Please send us anemail at [email protected]. Include the full Question ID number and the actual incorrect answer choice --not its letter, because that can change with every study session created. The Question ID number appears in the upperright corner of the ExamSuccess screen. Thank you in advance for helping us to make your HOCK study materialsbetter.

B.

This is not the correct answer. Please see the correct answer for a complete explanation.

We have been unable to determine how to calculate this incorrect answer choice. If you have calculated it, please letus know how you did it so we can create a full explanation of why this answer choice is incorrect. Please send us anemail at [email protected]. Include the full Question ID number and the actual incorrect answer choice --not its letter, because that can change with every study session created. The Question ID number appears in the upperright corner of the ExamSuccess screen. Thank you in advance for helping us to make your HOCK study materialsbetter.

C.

The variable overhead spending variance is the difference between the actual variable overhead cost per unitof the allocation base actually used (the actual overhead costs divided by the actual usage of the allocationbase) and the standard variable overhead application rate, multiplied by the actual quantity used of theapplication base. (Actual Cost per Hour [MH or DLH] Actually Used − Standard Application Rate) × ActualQuantity [MH or DLH] Used.)

This variance is also the difference between the actual amount of variable overhead incurred and the standardamount of variable overhead allowed for the actual quantity of the variable overhead allocation base used forthe output produced.

Part 1 : 10/05/14 22:20:20

(c) HOCK international, page 107

Page 108: Hock_Performance Mgmt

8/16/2019 Hock_Performance Mgmt

http://slidepdf.com/reader/full/hockperformance-mgmt 108/152

To use the formula — (Actual Cost per Hour [MH or DLH] Actually Used − Standard Application Rate) × ActualQuantity [MH or DLH] Used — we need to find the actual application rate and the standard application rate.The actual quantity of the application base (direct labor hours) used is given in the problem as 440,000.

Actual Cost per Hour Actually Used: The actual variable overhead incurred was $352,000. The actual quantityof direct labor hours used was 440,000. Therefore, the actual cost per hour actually used was $352,000 ÷440,000 = $0.80.

Standard Application Rate: This one is a little more complicated. Total budgeted overhead is given in theproblem as $900,000, but that includes both fixed and variable overhead. We need to figure out what thebudgeted variable overhead was. We have enough information to figure out what budgeted fixed overheadwas, so we can calculate budgeted fixed overhead and subtract it from budgeted total overhead to findbudgeted variable overhead. The fixed overhead application rate is given in the problem as $3.00 per unit, andthe budgeted production is given as 200,000 units. Therefore, total budgeted fixed overhead must have been$3.00 × 200,000 units, which is equal to $600,000.

Since total budgeted overhead was $900,000 and total budgeted fixed overhead was $600,000, total budgetedvariable overhead must have been $900,000 − $600,000, or $300,000. Since production was budgeted to be200,000 units, standard (budgeted) variable overhead per unit produced was $300,000 ÷ 200,000, which is$1.50 per unit. Overhead is applied on the basis of direct labor hours, and 2 direct labor hours are allowed for

each unit produced. So the standard variable overhead rate per direct labor hour allowed for production is$1.50 ÷ 2, which is $0.75.

Actual quantity of application base used: This is given as 440,000 direct labor hours.

The variable overhead spending variance is therefore:

(0.80 − 0.75) × 440,000 = 22,000

Since overhead is a cost, a positive variance is an Unfavorable variance. We said that the variable overheadspending variance is also the difference between the actual amount of variable overhead incurred and thestandard amount of variable overhead allowed for the actual quantity of the variable overhead allocation baseused for the output produced. Actual variable overhead incurred is given in the problem as $352,000. Thestandard amount of variable overhead allowed for the actual quantity of the variable overhead allocation baseused for the output produced is the standard variable overhead application rate of $0.75 (calculated above)multiplied by the actual direct labor hours used, which is 440,000. The result, $330,000, is subtracted from$352,000 to calculate the variance: $22,000 Unfavorable.

D.

This is not the correct answer. Please see the correct answer for a complete explanation.

We have been unable to determine how to calculate this incorrect answer choice. If you have calculated it, please letus know how you did it so we can create a full explanation of why this answer choice is incorrect. Please send us anemail at [email protected]. Include the full Question ID number and the actual incorrect answer choice --not its letter, because that can change with every study session created.The Question ID number appears in the upperright corner of the ExamSuccess screen. Thank you in advance for helping us to make your HOCK study materials

better.

Question 156 - CMA 696 3.26 - Responsibility Centers and Reporting Segments

Parkside Inc. has several divisions that operate as decentralized profit centers. Parkside's Entertainment Divisionmanufactures video arcade equipment using the products of two of Parkside's other divisions. The Plastics Divisionmanufactures plastic components, one type that is made exclusively for the Entertainment Division, while other less

Part 1 : 10/05/14 22:20:20

(c) HOCK international, page 108

Page 109: Hock_Performance Mgmt

8/16/2019 Hock_Performance Mgmt

http://slidepdf.com/reader/full/hockperformance-mgmt 109/152

complex components are sold to outside markets. The products of the Video Cards Division are sold in a competitivemarket; however, one video card model is also used by the Entertainment Division. The actual costs per unit used bythe Entertainment Division are presented below.

 Plastic

ComponentsVideoCards

Direct material $1.25 $2.40Direct labor 2.35 3.00

Variable overhead 1.00 1.50Fixed overhead .40 2.25Total cost $5.00 $9.15

The Plastics Division sells its commercial products at full cost plus a 25% markup and believes the proprietary plasticcomponent made for the Entertainment Division would sell for $6.25 per unit on the open market. The market price ofthe video card used by the Entertainment Division is $10.98 per unit.

 A per-unit transfer price from the Video Cards Division to the Entertainment Division at full cost, $9.15, would

 A. Provide no profit incentive for the Video Cards Division to control or reduce costs.B. Satisfy the Video Cards Division's profit desire by allowing recovery of opportunity costs.C. Allow evaluation of both divisions on a competitive basis.

D. Encourage the Entertainment Division to purchase video cards from an outside source.

A. This transfer price does not provide a profit incentive to the Video Cards Division, as it covers only thecosts of production and does not allow a profit. It also does not encourage the Video Cards Division to reducecosts, as the full cost transfer price is being passed on to the Entertainment Division.

B. This transfer price does not satisfy the Video Cards Division's profit desire by allowing the recovery of opportunitycosts because the transfer price is a full cost price and it does not include any profit for the Video Cards Division.

C. This transfer price does not allow for the evaluation of both divisions on a competitive basis. The EntertainmentDivision would be 'buying' video cards from the Video Cards Division at a 'full cost' transfer price; but it would be sellingits product at market price. Therefore, the Entertainment Division would have an opportunity to be more profitable thanthe Video Cards Division would.

D. A transfer price of full cost would be beneficial to the Entertainment Division, the buying department, as it is lessthan market price.

Question 157 - ICMA 10.P1.115 - Manufacturing Input Variances -- Overhead

When using a flexible budgeting system, the computation for the variable overhead spending variance is the differencebetween

 A. the amount applied to work-in-process and actual variable overhead.B. actual variable overhead and actual inputs times the budgeted rate.C. the previously budgeted amount and actual inputs times the budgeted rate.

D. actual variable overhead and the previously budgeted amount.

 A. This is the definition of the total variable overhead variance. The total variable overhead variance is further brokendown into the variable overhead spending variance and the variable overhead efficiency variance. This question asksfor the the computation for the variable overhead spending variance.

B. The variable overhead spending variance is the difference between actual variable overhead incurred andbudgeted variable overhead based on the inputs (labor or machine hours) actually used. The budgetedvariable overhead based on the inputs actually used is the budgeted rate per labor hour or machine hour(whichever is being used as the allocation base) multiplied by the actual number of hours used for the actual

Part 1 : 10/05/14 22:20:20

(c) HOCK international, page 109

Page 110: Hock_Performance Mgmt

8/16/2019 Hock_Performance Mgmt

http://slidepdf.com/reader/full/hockperformance-mgmt 110/152

output.

C. The difference between the previously budgeted amount and actual inputs times the budgeted rate is not a definitionof the computation of any variable overhead variance. Furthermore, the term "previously budgeted amount" of variableoverhead does not adequately describe any variable overhead budget amount. The term does not specify whether it isa cost per unit or a total cost, and if a total cost, whether it is a static budget amount or a flexible budget amount.

D. The difference between actual variable overhead and the previously budgeted variable overhead is not a usefuldefinition of the computation of any variable overhead variance. "Previously budgeted amount" of variable overheaddoes not adequately describe any variable overhead budget amount. The term does not specify whether it is a cost perunit or a total cost, and if a total cost, whether it is a static budget amount or a flexible budget amount.

Question 158 - CMA 1291 3.16 - Variance Analysis Concepts

Folsom Fashions sells a line of women's dresses. Folsom's performance report for November follows.Actual Budget

Dresses sold 5,000 6,000Sales $ 235,000 $ 300,000Variable costs (145,000) (180,000)Contribution margin $ 90,000 $ 120,000Fixed costs (84,000) (80,000)Operating income $ 6,000 $ 40,000

The company uses a flexible budget to analyze its performance and to measure the effect on operating income of thevarious factors affecting the difference between budgeted and actual operating income.

The variable cost flexible budget variance for November is

 A. $5,000 unfavorable.B. $5,000 favorable.C. $4,000 unfavorable.

D. $4,000 favorable.

 A. The actual cost incurred was lower than the budgeted amount for the actual activity, so the variance is favorable.

B.

The flexible budget variance for variable costs is the difference between the actual variable cost incurred andthe budgeted amount for the actual activity. The budgeted variable cost per unit sold is $180,000 in budgetedtotal variable costs ÷ 6,000 budgeted units sold, or $30 per unit. Since 5,000 dresses were sold, the budgetedamount for the actual activity is $30 × 5,000, or $150,000.

The actual total variable cost incurred was $145,000. Therefore, the flexible budget variance for variable costsis the difference between $150,000 and $145,000, or $5,000. The variance is favorable because the actual cost

incurred ($145,000) was lower than the budgeted amount for the actual activity ($150,000).

C. This is the fixed cost variance. However, the question asks for the variable cost variance.

D. This is the difference between the actual and budgeted fixed cost. However, the question asks for the variable costflexible budget variance.

Part 1 : 10/05/14 22:20:20

(c) HOCK international, page 110

Page 111: Hock_Performance Mgmt

8/16/2019 Hock_Performance Mgmt

http://slidepdf.com/reader/full/hockperformance-mgmt 111/152

Question 159 - CIA 594 III.72 - Manufacturing Input Variances -- Materials and Labor 

 A company manufactures one product and has a standard cost system. In April the company had the followingexperience:  Direct MaterialsDirect Labor  Actual $/unit of input (lbs. & hrs.) $28 $18Standard price/unit of input $24 $20

Standard inputs allowed per unit of output 10 4 Actual units of input 190,000 78,000 Actual units of output 20,000 20,000

The direct materials price variance for April is

 A. $156,000 favorable.B. $760,000 favorable.C. $760,000 unfavorable.D. $240,000 unfavorable.

 A. This is the direct labor rate variance. However, the question asks for the direct materials price variance. See thecorrect answer for a complete explanation.

B. The variance is unfavorable because the actual price ($28) is higher than standard ($24). See the correct answer fora complete explanation.

C. The direct materials price variance is calculated as follows: (Actual Price − Standard Price) × ActualQuantity. All the components of the formula are in the data given. The actual price is $28. The standard priceis $24. The actual quantity is 190,000. Therefore, the direct materials price variance is ($28 − $24) × 190,000 =$760,000 unfavorable. Because the actual price was higher than the standard, the variance is unfavorable.

D. This is the materials efficiency variance. However, the question asks for the materials price variance. See thecorrect answer for a complete explanation.

Question 160 - CMA 1296 3.21 - Responsibility Centers and Reporting Segments

David Rogers, purchasing manager at Fairway Manufacturing Corporation, was able to acquire a large quantity of rawmaterial from a new supplier at a discounted price. Marion Conner, inventory supervisor, is concerned because thewarehouse has become crowded and some things had to be rearranged. Brian Jones, vice president of production, isconcerned about the quality of the discounted material. However, the Engineering Department tested the new rawmaterial and indicated that it is of acceptable quality. At the end of the month, Fairway experienced a favorablematerials usage variance, a favorable labor usage variance, and a favorable materials price variance. The usagevariances were solely the result of a higher yield from the new raw material. The favorable materials price variancewould be considered the responsibility of the

 A. Inventory supervisor.B. Vice president of production.C. Engineering manager.D. Purchasing manager.

 A. The inventory supervisor is not responsible for the price variance, because he or she does not participate inpurchasing activity and usually can't influence the price of materials.

B. The vice president of production was concerned only about the quality of new material in this situation. The vicepresident does not participate in purchasing activity and usually can't influence the price of materials.

C. The engineering manager does not participate in purchasing activity and usually can't influence the price of

Part 1 : 10/05/14 22:20:20

(c) HOCK international, page 111

Page 112: Hock_Performance Mgmt

8/16/2019 Hock_Performance Mgmt

http://slidepdf.com/reader/full/hockperformance-mgmt 112/152

materials.

D. The material price variance is the difference between the actual price and the standard price times theactual quantity of materials consumed by production. The purchasing manager is usually responsible for thematerial price variance, as the price is determined during the purchasing activity.

Question 161 - ICMA 10.P1.124 - Responsibility Centers and Reporting Segments

Manhattan Corporation has several divisions that operate as decentralized profit centers. At the present time, theFabrication Division has excess capacity of 5,000 units with respect to the UT-371 circuit board, a popular item in manydigital applications. Information about the circuit board follows.Market price $48Variable selling/distribution costs on external sales 5Variable manufacturing cost 21Fixed manufacturing cost 10

Manhattan’s Electronic Assembly Division wants to purchase 4,500 circuit boards either internally, or else use a similar

board in the marketplace that sells for $46. The Electronic Assembly Division’s management feels that if the firstalternative is pursued, a price concession is justified, given that both divisions are part of the same firm. The bestprocess to determine the price ultimately charged by the Fabrication Division to the Assembly Division for the circuitboard is to

 A. establish the price through negotiations between the Fabrication's and Electronic Assembly's Division management.B. establish the price by an arbitration committee.C. establish the price by top management.D. set the price equal to the price that would be charged if the Fabrication Department had no excess capacity.

A. The best process for establishing a transfer price between different divisions is for the managers of therespective divisions to negotiate the price. The best price is usually the market price when a market priceexists, but that still needs to be acceptable to both division managers. And if the market price is notacceptable to one or both managers for some reason, a different price should be negotiated (without violating

any tax laws of the countries and/or other taxing jurisdictions involved.

B. Any transfer price that is imposed on the managers of the two divisions may not be acceptable to one or bothmanagers.

C. Any transfer price that is imposed on the managers of the two divisions may not be acceptable to one or bothmanagers.

D. Any transfer price that is imposed on the managers of the two divisions may not be acceptable to one or bothmanagers.

Question 162 - CMA 692 3.21 - Manufacturing Input Variances -- Materials and Labor 

Jackson Industries employs a standard cost system in which direct materials inventory is carried at standard cost.Jackson has established the following standards for the prime costs of one unit of product.

 StandardQuantity

StandardPrice

StandardCost

Direct materials 5 pounds$3.60/pound $18.00Direct labor 1.25 hours 12.00/hour 15.00  $33.00

Part 1 : 10/05/14 22:20:20

(c) HOCK international, page 112

Page 113: Hock_Performance Mgmt

8/16/2019 Hock_Performance Mgmt

http://slidepdf.com/reader/full/hockperformance-mgmt 113/152

During May, Jackson purchased 125,000 pounds of direct materials at a total cost of $475,000. The total factory wagesfor May were $364,000, 90% of which were for direct labor. Jackson manufactured 22,000 units of product during Mayusing 108,000 pounds of direct materials and 28,000 direct labor hours.

The direct labor usage (efficiency) variance for May is

 A. $6,000 unfavorable.

B. $5,850 unfavorable.C. $5,850 favorable.D. $6,000 favorable.

A. The direct labor efficiency variance is calculated as: (Actual Hours − Standard Hours for Actual Output) ×Standard Rate. Actual hours is 28,000. The standard hours allowed for the actual level of output is 27,500hours (1.25 hours per unit × 22,000 units produced). The standard labor rate is $12. Therefore, the direct laborefficiency variance is (28,000 − 27,500) × $12 = $6,000 unfavorable. The variance is unfavorable because theactual hours exceed the standard hours, and this is a cost variance.

B. The labor efficiency variance is calculated as: (Actual Hours − Standard Hours for Actual Output) × Standard Rate.This answer results from multiplying by the actual rate ($11.70) instead of the standard rate.

C. The actual hours exceed the standard amount, which means the variance is unfavorable. See the correct answer fora complete explanation.

D. The actual hours exceed the standard amount, which means the variance is unfavorable. See the correct answer fora complete explanation.

Question 163 - CMA 1294 3.22 - Responsibility Centers and Reporting Segments

If a manufacturing company uses responsibility accounting, which one of the following items is least likely to appear ina performance report for a manager of an assembly line?

 A. Supervisory salaries.B. Materials.C. Repairs and maintenance.D. Equipment depreciation.

 A. In accordance with responsibility accounting, a manager should be held responsible only for those factors which heor she can control. The assembly line manager will have input into determining the salaries of supervisors under him orher. Therefore, supervisory salaries are controllable by the manager, so they should be included in the assembly linemanager's performance report.

B. In accordance with responsibility accounting, a manager should be held responsible only for those factors which heor she can control. The assembly line manager will have control over the materials used and therefore those costsshould be included in the performance report of the assembly line manager.

C. In accordance with responsibility accounting, a manager should be held responsible only for those factors which heor she can control. The assembly line manager will have control over repairs and maintenance and therefore thosecosts should be included in the performance report of the assembly line manager.

D. In accordance with responsibility accounting, a manager should be held responsible only for those factorswhich he or she can control. Costs of fixed assets and depreciation methods and policies are not under thecontrol of an assembly line manager, because those things are decided at a higher level. Therefore, equipmentdepreciation expense would not normally be included in a performance report for the assembly line managerunder responsibility accounting.

Part 1 : 10/05/14 22:20:20

(c) HOCK international, page 113

Page 114: Hock_Performance Mgmt

8/16/2019 Hock_Performance Mgmt

http://slidepdf.com/reader/full/hockperformance-mgmt 114/152

Question 164 - ICMA 10.P1.141 - Performance Measures

Oakmont Company has two divisions, Household Appliances and Construction Equipment. The manager of theHousehold Appliances Division is evaluated on the basis of return on investment (ROI). The manager of theConstruction Equipment Division is evaluated on the basis of residual income. The cost of capital has been 12% andthe return on investment has been 16% for the two divisions. Each manager is currently considering a project with a14% rate of return. According to the current evaluation system for managers, which manager(s) would have incentiveto undertake the project?

 A. Neither manager would have incentive to undertake the project.B. Both managers would have incentive to undertake the project.C. The manager of the Construction Equipment Division would have incentive to undertake the project while themanager of the Household Appliances Division would not have incentive to undertake the project.D. The manager of the Household Appliances Division would have incentive to undertake the project while themanager of the Construction Equipment Division would not have incentive to undertake the project.

 A. One of the managers would have incentive to undertake the project.

B. The manager of the Household Appliances Division would not have incentive to undertake the project becausehe/she is evaluated on the basis of return on investment, and a project with a 14% rate of return would decrease the16% return on investment the division is currently achieving.

C.

The manager of the Construction Equipment Division would have incentive to undertake the project becausehe/she is evaluated on the basis of residual income, and a project with a 14% rate of return would increaseresidual income.

The manager of the Household Appliances Division would not have incentive to undertake the projectbecause he/she is evaluated on the basis of return on investment, and a project with a 14% rate of returnwould decrease the 16% return on investment the division is currently achieving.

However, since both projects have rates of return that are greater than the company's cost of capital, bothprojects will increase the value of the company, so both should be accepted by their respective managers.

D.

The manager of the Household Appliances Division would not have incentive to undertake the project because he/sheis evaluated on the basis of return on investment, and a project with a 14% rate of return would decrease the 16%return on investment the division is currently achieving.

The manager of the Construction Equipment Division would have incentive to undertake the project because he/she isevaluated on the basis of residual income, and a project with a 14% rate of return would increase residual income.

Question 165 - CMA 696 3.28 - Responsibility Centers and Reporting Segments

Parkside Inc. has several divisions that operate as decentralized profit centers. Parkside's Entertainment Divisionmanufactures video arcade equipment using the products of two of Parkside's other divisions. The Plastics Divisionmanufactures plastic components, one type that is made exclusively for the Entertainment Division, while other lesscomplex components are sold to outside markets. The products of the Video Cards Division are sold in a competitivemarket; however, one video card model is also used by the Entertainment Division. The actual costs per unit used bythe Entertainment Division are presented below.

Part 1 : 10/05/14 22:20:20

(c) HOCK international, page 114

Page 115: Hock_Performance Mgmt

8/16/2019 Hock_Performance Mgmt

http://slidepdf.com/reader/full/hockperformance-mgmt 115/152

 Plastic

ComponentsVideoCards

Direct material $1.25 $2.40Direct labor 2.35 3.00Variable overhead 1.00 1.50Fixed overhead .40 2.25Total cost $5.00 $9.15

The Plastics Division sells its commercial products at full cost plus a 25% markup and believes the proprietary plasticcomponent made for the Entertainment Division would sell for $6.25 per unit on the open market. The market price ofthe video card used by the Entertainment Division is $10.98 per unit.

 Assume that the Plastics Division has excess capacity and it has negotiated a transfer price of $5.60 per plasticcomponent with the Entertainment Division. This price will

 A. Cause the Plastics Division to reduce the number of commercial plastic components it manufactures.B. Encourage the Entertainment Division to seek an outside source for plastic components.C. Motivate both divisions as estimated profits are shared.D. Demotivate the Plastics Division causing mediocre performance.

 A. Because this price is above the variable costs of production, the Plastics Division would have no reason to reduceproduction because even what it sells internally generates a contribution.

B. Because the price is lower than the market price, the Entertainment Division will not seek an outside supplier.

C. Because the Plastics Division has excess capacity, this price is a mutually motivating price. The price forthe purchasing division is less than it would pay in the market and the price for the selling division is higherthan its variable costs of production. So, both divisions end up in a better position because of this price.

D. Given that the transfer price is higher than the variable costs of production, there is no reason for the producingdivision to be demotivated.

Question 166 - ICMA 10.P1.214 - Responsibility Centers and Reporting Segments

Jones Tax Company has three divisions - Compliance, Tax Planning, and Financial Consulting. Based on the divisionaldata presented below, which one of the allocation bases for common company expenses would likely have the least negative behavioral impact on the Financial Consulting Division manager?  ComplianceTax PlanningFinancial ConsultingRevenues $4,500,000 $6,000,000 $4,500,000Variable e xpenses 1,500,000 3,750,000 2,250,000No. of employees 68 76 56

 A. Equal sharing.B. Number of employees.

C. Revenues.D. Contribution margin.

 A. Managers tend to prefer allocation bases for common costs that do not cause their division to be charged more thanwhat they regard as their fair share of common costs. If common costs are allocated equally to each division, themanager of the Financial Consulting Division will feel it is unfair, because the Financial Consulting Division's share oftotal revenue, total variable expenses and the total contribution margin is only 30%. An allocation of 1/3 (33 1/3%) ofthe common costs would seem unfair and might be de-motivating.

B.

Part 1 : 10/05/14 22:20:20

(c) HOCK international, page 115

Page 116: Hock_Performance Mgmt

8/16/2019 Hock_Performance Mgmt

http://slidepdf.com/reader/full/hockperformance-mgmt 116/152

Managers tend to prefer allocation bases for common costs that do not cause their division to be chargedmore than what they regard as their fair share of common costs. If they feel they are being charged morebecause their revenues or their contribution margin is higher than other divisions', they will feel they arebeing punished for doing well. A manager might even cut back on sales efforts or make other changes thatwould have a negative effect on the company in an effort to cut the amount of common costs being allocatedto his or her division.

Allocating common costs on the basis of number of employees would likely have the least negative behavioralimpact on the Financial Consulting Division's manager, because the Financial Consulting Division has feweremployees than the other two divisions have. As a result, the amount of common costs allocated to theFinancial Consulting Division will be less than the amount allocated to the other divisions.

Furthermore, if allocating common costs on the basis of number of employees causes the manager of theFinancial Consulting Division to look at the possibility of cutting the number of employees in the division, thatcould be a positive behavioral impact. If the company is able to function well with fewer employees, then itshould do so, because profits will increase as a result. So from the standpoint of management, workforcereductions are positive. Reduction of the workforce would be a negative thing only if the workforce werereduced so much that the company's operations were harmed — i.e. if it were not be able to provide productand service adequately to its customers as a result.

C. Managers tend to prefer allocation bases for common costs that do not cause their division to be charged more thanwhat they regard as their fair share of common costs. If they feel they are being charged more than they should bebecause their revenues too high, they will feel they are being punished for doing well. A manager might even cut backon sales efforts, which would have a negative effect on the company, in an effort to cut the amount of common costsbeing allocated to his or her division.

D. Managers tend to prefer allocation bases for common costs that do not cause their division to be charged more thanwhat they regard as their fair share of common costs. If they feel they are being charged more than they should bebecause their contribution margin is too high, they will feel they are being punished for doing well. A manager mighteven make changes that would cause the contribution margin to decrease, which would have a negative effect on thecompany, in an effort to cut the amount of common costs being allocated to his or her division.

Question 167 - CMA 697 3.23 - Manufacturing Input Variances -- Materials and Labor 

The controller for Durham Skates is reviewing the production cost report for July. An analysis of direct materials costsreflects an unfavorable flexible budget variance of $25. The plant manager believes this is excellent performance on aflexible budget for 5,000 units of direct materials. However, the production supervisor is not pleased with this resultbecause he claims to have saved $1,200 in materials cost on actual production using 4,900 units of direct materials.The standard materials cost is $12 per unit. Actual materials used for the month amounted to $60,025.

If the direct materials variance is investigated further, it will reflect a price variance of 

 A. $1,200 favorable.B. $1,225 unfavorable.C. $2,500 favorable.

D. Zero.

 A. The actual price is higher than the standard price, which gives us an unfavorable price variance. See the correctanswer for a complete explanation.

B. The price variance is calculated as follows: (Actual Price − Standard Price) × Actual Quantity. The actualmaterials price is $12.25 ($60,025 ÷ 4,900). The standard materials price is $12.00 per unit. The actual quantityis 4,900 units of direct materials. Hence, the material price variance is ($12.25 − $12.00) × 4,900 = $1,225unfavorable. The actual price is higher than the standard price, so the price variance is unfavorable.

Part 1 : 10/05/14 22:20:20

(c) HOCK international, page 116

Page 117: Hock_Performance Mgmt

8/16/2019 Hock_Performance Mgmt

http://slidepdf.com/reader/full/hockperformance-mgmt 117/152

C. The actual price is higher than the standard price, which gives us an unfavorable price variance. See the correctanswer for a complete explanation.

D. The actual price is higher than the standard price, which gives us an unfavorable price variance. See the correctanswer for a complete explanation.

Question 168 - CMA 695 3.28 - Responsibility Centers and Reporting Segments

In responsibility accounting, a center's performance is measured by controllable costs. Controllable costs are bestdescribed as including

 A. Only discretionary costs.B. Only those costs that the manager can influence in the current time period.C. Those costs about which the manager is knowledgeable and informed.D. Direct material and direct labor, only.

 A. Other costs besides discretionary costs may also be controllable by a manager.

B. A definition of a controllable cost is a cost that the manager is able to influence in the time period underconsideration.

C. Just because a manager has knowledge and information about a cost does not mean that the cost is controllable bythat manager.

D. Other types of costs (for example, marketing) are costs that may be controllable by a manager.

Question 169 - ICMA 10.P1.128 - Responsibility Centers and Reporting Segments

Manhattan Corporation has several divisions that operate as decentralized profit centers. At the present time, theFabrication Division has excess capacity of 5,000 units with respect to the UT-371 circuit board, a popular item in manydigital applications. Information about the circuit board follows.Market price $48Variable selling/distribution costs on external sales 5Variable manufacturing cost 21Fixed manufacturing cost 10

Manhattan’s Electronic Assembly Division wants to purchase 4,500 circuit boards either internally, or else use a similarboard in the marketplace that sells for $46. The Electronic Assembly Division’s management feels that if the firstalternative is pursued, a price concession is justified, given that both divisions are part of the same firm. To optimizethe overall goals of Manhattan, the minimum price to be charged for the board from the Fabrication Division to theElectronic Assembly Division should be

 A. $21.B. $46.C. $26.D. $31.

A.

There are two important points to note in this question. One, the Fabrication Division has excess capacity thatis adequate to manufacture all of the 4,500 units of UT-371 that the Electronic Assembly wants to purchase.And two, the question asks for the minimum price, not the best price. "Minimum" means the very lowest price

Part 1 : 10/05/14 22:20:20

(c) HOCK international, page 117

Page 118: Hock_Performance Mgmt

8/16/2019 Hock_Performance Mgmt

http://slidepdf.com/reader/full/hockperformance-mgmt 118/152

that the Fabrication Division must receive to avoid having a loss on the internal sale.

The Fabrication Division will not have any variable selling and distribution costs on the internal order, and itstotal fixed manufacturing cost will be the same whether it accepts the internal order or not. Since theFabrication Division has enough excess capacity to produce the order without having to give up any externalorders, there will be no opportunity cost. Therefore, the very lowest price that the Fabrication Division mustreceive is its variable manufacturing cost of $21. The Fabrication Division will break even if the price it

receives for the internal order from the Electronic Assembly Division is equal to its variable manufacturingcost.

B. The question asks for the minimum price, not the best price. "Minimum" means the very lowest price that theFabrication Division must receive to avoid having a loss on the internal sale. This is not the minimum price theFabrication Division must receive. This is the quote the Electronic Assembly Division has received from the outsidesupplier.

C. The question asks for the minimum price, not the best price. "Minimum" means the very lowest price that theFabrication Division must receive to avoid having a loss on the internal sale. This is not the minimum price because itincludes the variable selling and distribution costs that the Fabrication Division will not have to pay.

D. The question asks for the minimum price, not the best price. "Minimum" means the very lowest price that theFabrication Division must receive to avoid having a loss on the internal sale. This is not the minimum price because itincludes an allocation for the fixed manufacturing cost. The total fixed manufacturing cost that the Fabrication Divisionwill have to pay will be the same whether it takes this order or not.

Question 170 - CMA 695 3.10 - Introduction to Variance Analysis and Standard Costs

 A standard costing system is most often used by a firm in conjunction with

 A. Management by objectives.B. Target (hurdle) rates of return.C. Flexible budgets.

D. Participative management programs.

 A. MBO can be used in conjunction with a standard costing system, but they are not necessarily related. The primaryfocus of MBO is goal congruence; it is a behavioral, communication-oriented, responsibility approach system.

B. The target (hurdle) rate of return is a capital budgeting function. It is not concerned with standard costs.

C.

A standard cost is an estimate of the cost the company expects to incur in the production process. Without astandard cost, the analysis of actual activities and results is very difficult because there is no standard tomeasure the performance against. Standard costs are best used with a flexible budgeting system in order toprovide the most useful variance analysis. The flexible budget will enable the company to identify differencesfrom the budget that are not simply due to the actual quantity produced or sold being different from the

budgeted quantity. 

D. Participative management lies in the area of communications and does not relate to standard costs.

Question 171 - CIA 1191 IV.19 - Responsibility Centers and Reporting Segments

Part 1 : 10/05/14 22:20:20

(c) HOCK international, page 118

Page 119: Hock_Performance Mgmt

8/16/2019 Hock_Performance Mgmt

http://slidepdf.com/reader/full/hockperformance-mgmt 119/152

 A carpet manufacturer maintains a retail division consisting of stores stocking its brand and other brands, and amanufacturing division that makes carpets and pads. An outside market exists for carpet padding material in which allpadding produced can be sold. The proper transfer price for padding transferred from the manufacturing division to theretail division is

 A. Variable manufacturing division production cost.B. The market price at which the retail division could purchase padding.

C. Variable manufacturing division production cost plus allocated fixed factory overhead.D. Variable manufacturing division production cost plus variable selling and administrative cost.

 A. When there is an external market for the product, market price is almost always the best transfer price to use.

B. The basic issue of transfer prices is how much should one unit of a company charge another unit of thesame company for its goods or services. The goal in setting a transfer price is that the method used willstimulate the department managers to do what will provide the greatest benefit to the company as a whole,rather than to act in their own interest. Since an outside market exists for padding produced in which allpadding produced can be sold, the best transfer price is the market price.

C. When there is an external market for the product, market price is almost always the best transfer price to use.

D. When there is an external market for the product, market price is almost always the best transfer price to use.

Question 172 - CMA 692 3.13 - Responsibility Centers and Reporting Segments

Ordinarily, the most appropriate basis on which to evaluate the performance of a division manager is the division's

 A. Gross profit.B. Contribution margin.C. Net income minus the division's fixed costs.D. Net revenue minus controllable division costs.

 A. A manager's performance valuation should be based on the factors controllable by the manager. Gross profit isequal to sales revenue minus COGS. Cost of goods sold includes part of the fixed manufacturing overheads. However,fixed manufacturing overheads usually include items that are not controlled by a division manager, such asdepreciation. Thus, gross profit should not be used as a division manager performance evaluation tool.

B. A manager's performance valuation should be based on the factors controllable by the manager. Contributionmargin does not include fixed costs and some of the fixed costs may be controllable by the division manager.

C. A manager's performance valuation should be based on the factors controllable by the manager. Net income minusthe division's fixed costs would (a) include a reduction for the fixed costs twice, since fixed costs are a reduction to netincome; and (b) only some of the fixed costs, not all, may be controllable by the division manager.

D. A manager's performance valuation should be based on the factors controllable by the manager. Acontribution income statement that presents net revenue minus controllable division costs can be used to

isolate the controllable costs of a business unit from its non-controllable costs such as depreciation orallocated central costs. According to the contribution income statement approach to evaluation, a divisionmanager usually controls the division's revenues, variable costs and a portion of its fixed costs.

Question 173 - CMA 687 4.18 - Manufacturing Input Variances -- Materials and Labor 

Baxter Corporation's master budget calls for the production of 5,000 units of product monthly. The master budget

Part 1 : 10/05/14 22:20:20

(c) HOCK international, page 119

Page 120: Hock_Performance Mgmt

8/16/2019 Hock_Performance Mgmt

http://slidepdf.com/reader/full/hockperformance-mgmt 120/152

Page 121: Hock_Performance Mgmt

8/16/2019 Hock_Performance Mgmt

http://slidepdf.com/reader/full/hockperformance-mgmt 121/152

 A. Production and industrial engineering.B. Sales and industrial engineering.C. Purchasing and industrial engineering.D. Purchasing and sales.

A. Industrial engineers are involved in designing product and setting the standards of material usage, thusbearing a part of the responsibility for the efficiency of material usage. Production workers are those who

actually convert materials into the finished product and directly responsible for efficiency of material usage.Therefore, both of these parties will have some responsibility for the materials efficiency variance.

B. Industrial engineers are involved in designing product and setting the standards of material usage, thus bearing apart of the responsibility for the efficiency of material usage. However, sales personnel rarely can influence materialusage by production workers.

C. Industrial engineers are involved in designing product and setting the standards of material usage, thus bearing apart of the responsibility for the efficiency of material usage. However, purchasing personnel rarely can influencematerial usage by production workers.

D. Purchasing and sales personnel rarely can influence material usage by production workers.

Question 176 - CMA 1273 4.13 - Manufacturing Input Variances -- Overhead

Which of these variances is least significant for cost control?

 A. Fixed O/H volume variance.B. Labor price variance.C. Variable O/H spending variance.D. Materials quantity variance.

A. The fixed overhead volume variance is the difference between the budgeted amount of fixed overhead andthe amount of fixed overhead applied (standard rate × standard input for the actual level of output). It does not

reflect any differences between actual costs and budgeted costs. This variance concerns only the level ofproduction. If the actual level of production is lower than the budgeted level, the fixed overhead volumevariance will be unfavorable. If the actual production level is higher than the budgeted level, the fixedoverhead volume variance will be favorable. Because this variance arises from a variance in the level ofproduction, it is not very significant for cost control purposes.

B. The labor price variance is the difference in labor rates between actual and standard used in production. It is animportant area of cost control.

C. The variable overhead spending variance is the difference between the actual application rate (the actual applicationrate is calculated as the actual overhead costs ÷ the actual usage of the allocation base) and the standard applicationrate multiplied by the actual quantity of the application base. Because actual variable costs are used in this variance, itis an important area of cost control.

D. The materials quantity variance is the difference between the quantity of materials actually used in production andthe standard quantity of materials allowed for the actual level of output. It is an important area of cost control.

Question 177 - CMA 686 4.14 - Responsibility Centers and Reporting Segments

The segment margin of the Wire Division of Lerner Corporation should not include:

Part 1 : 10/05/14 22:20:20

(c) HOCK international, page 121

Page 122: Hock_Performance Mgmt

8/16/2019 Hock_Performance Mgmt

http://slidepdf.com/reader/full/hockperformance-mgmt 122/152

 A. Fixed selling expenses of the Wire Division.B. Variable selling expenses of the Wire Division.C. Net sales of the Wire Division.D. The Wire Division's fair share of the salary of Lerner Corporation's president.

 A. Segment margin includes sales, variable costs, controllable fixed costs and traceable fixed costs. Fixed sellingexpenses are controllable and should be included in the segment margin of this division.

B. Segment margin includes sales, variable costs, controllable fixed costs and traceable fixed costs. Variable sellingexpenses should be included in the segment margin of this division.

C. Segment margin includes sales, variable costs, controllable fixed costs and traceable fixed costs. Sales should beincluded in the segment margin of this division.

D. Segment margin includes sales, variable costs, controllable fixed costs and traceable fixed costs. Theallocation of the president's salary is an untraceable fixed cost and should not be included in segment marginof this division.

Question 178 - ICMA 10.P1.136 - Performance Measures

 Albert Hathaway recently joined Brannen University as the chief information officer of the University ComputingServices Department. His assigned task is to help reduce the recurrent problem of cost overruns due to uncontrolledcomputer usage by the user community, while at the same time not curtailing the use of information technology forresearch and teaching. To ensure goal congruence, which one of the following algorithms should be used to allocatethe cost of the University Computing Services Department to other departments within the university?

 A. Budgeted rate times actual hours of computer usage.B. Actual rate times budgeted hours of computer usage.C. Actual rate times actual hours of computer usage.D. Budgeted rate times budgeted hours of computer usage.

A. So that user departments can know the hourly rate they will be charged in advance, the budgeted hourlyrate should be used. To encourage them to limit their computer usage, they should be charged for their actualhours of computer usage.

B. If the actual rate is used, the user departments will not be able to know in advance what hourly rate they will becharged, because the hourly rate can only be determined after the usage takes place and the costs for it are known.Furthermore, charging user departments for their budgeted hours of computer usage will not encourage them to limittheir usage.

C. If the actual rate is used, the user departments will not be able to know in advance what hourly rate they will becharged, because the hourly rate can only be determined after the usage takes place and the costs for it are known.

D. Charging user departments for their budgeted hours of computer usage will not encourage them to limit their usage.

Question 179 - ICMA 10.P1.111 - Manufacturing Input Variances -- Overhead

Highlight Inc. uses a standard cost system and applies factory overhead to products on the basis of direct labor hours.If the firm recently reported a favorable direct labor efficiency variance, then the

 A. fixed overhead volume variance must be unfavorable.B. variable overhead spending variance must be favorable.

Part 1 : 10/05/14 22:20:20

(c) HOCK international, page 122

Page 123: Hock_Performance Mgmt

8/16/2019 Hock_Performance Mgmt

http://slidepdf.com/reader/full/hockperformance-mgmt 123/152

C. variable overhead efficiency variance must be favorable.D. direct labor rate variance must be unfavorable.

 A.

The direct labor efficiency variance is (AQ − SQ) × SP, or (AQ × SP) − (SQ × SP). If the variance is favorable, the firstnumber will be lower than the second number and the variance will be negative because AQ is lower than SQ; and it

means the actual quantity of labor used was lower than the standard quantity allowed for the actual output.

The fixed overhead production-volume variance is budgeted fixed overheads minus the fixed overhead applied.

Since these two variances have no information in common, there are no grounds for drawing any conclusions aboutthe fixed overhead production volume variance from the direct labor efficiency variance.

B.

The direct labor efficiency variance is (AQ − SQ) × SP, or (AQ × SP) − (SQ × SP). If the variance is favorable, the firstnumber will be lower than the second number and the variance will be negative because AQ is lower than SQ; and itmeans the actual quantity of labor used was lower than the standard quantity allowed for the actual output.

The variable overhead spending variance is actual total variable overhead incurred minus budgeted variable overhead

based on inputs actually used.

The difference between these two variances is that the second number of the direct labor efficiency variance is (SQ ×SP), or the standard cost based on the standard quantity for the actual output, whereas the second number of thevariable overhead spending variance is budgeted variable overhead based on inputs actually used (not standardinputs).

Since the two formulas are essentially different, no conclusions can be drawn about whether or not a favorable directlabor efficiency variance means the variable overhead spending variance must also be favorable.

C.

The direct labor efficiency variance is (AQ − SQ) × SP, or (AQ × SP) − (SQ × SP). If the variance is favorable,the first number will be lower than the second number and the variance will be negative because AQ is lowerthan SQ; and it means the actual quantity of labor used was lower than the standard quantity allowed for theactual output.

The variable overhead efficiency variance is budgeted variable overhead based on inputs actually used minusstandard variable overhead applied to production. So if the actual quantity of labor used was lower than thestandard quantity allowed for the actual output, then the first number will be lower than the second numberand the variance will be negative.

So if the direct labor efficiency variance is favorable, the variable overhead efficiency variance will also befavorable.

D.

The direct labor efficiency variance is (AQ − SQ) × SP, or (AQ × SP) − (SQ × SP). If the variance is favorable, the firstnumber will be lower than the second number and the variance will be negative because AQ is lower than SQ; and itmeans the actual quantity of labor used was lower than the standard quantity allowed for the actual output.

The direct labor rate variance is the price variance for direct labor. The formula is (AP − SP) × AQ, or (AP × AQ) − (SP× AQ).

Since these two variances are calculated in completely different ways, there are no grounds for drawing anyconclusions about the direct labor rate variance from the direct labor efficiency variance.

Part 1 : 10/05/14 22:20:20

(c) HOCK international, page 123

Page 124: Hock_Performance Mgmt

8/16/2019 Hock_Performance Mgmt

http://slidepdf.com/reader/full/hockperformance-mgmt 124/152

Question 180 - CMA 694 3.23 - Manufacturing Input Variances -- Materials and Labor 

 A favorable materials price variance coupled with an unfavorable materials usage variance would most likely result from

 A. The purchase and use of higher than standard quality materials.

B. Product mix production changes.C. Machine efficiency problems.D. The purchase of lower than standard quality materials.

 A. The purchase and use of higher than standard quality materials would most likely result in an unfavorable materialsprice variance and either not significantly affect the material usage variance or would lead to favorable materialefficiency variance.

B. Product mix production changes would not cause the materials price variance.

C. Machine efficiency problems would not cause the materials price variance.

D. The favorable materials price variance means that the material was purchased for a lower price thanplanned. This could happen due to a quantity discount, lower quality materials, or some other reason. Theunfavorable materials usage variance means that the production process used more material than had beenplanned. This could happen due to a lower skill level of workers, poor material quality or some other reason.However, these two variances together are most likely caused by the same reason. This reason could be thepurchase of lower than standard quality materials.

Question 181 - CIA 1183 IV.5 - Responsibility Centers and Reporting Segments

 A company has two divisions, A and B, each operated as a profit center. A charges B $35 per unit for each unittransferred to B. Other data follow: A's variable cost per unit $30

 A's fixed costs $10,000 A's annual sales to B 5,000 units A's sales to outsiders 50,000 units

 A is planning to raise its transfer price to $50 per unit. Division B can purchase units at $40 each from outsiders, butdoing so would idle A's facilities now committed to producing units for B. Division A cannot increase its sales tooutsiders. From the perspective of the company as a whole, from whom should Division B acquire the units, assumingB's market is unaffected?

 A. Division A, despite the increased transfer price.B. Division A, but only until fixed costs are covered, then from outside vendors.C. Division A, but only at the variable cost per unit.D. Outside vendors.

A.

When the variable costs plus the opportunity cost of lost outside sales for the internal supplier are lower thanthe price of the external supplier, the company as a whole is better off if the buying department buysinternally, even if the transfer price is higher than an outside supplier's price.

That is because the income to the selling division is equal to the cost to the buying division. These are calledintercompany sales , and the two items are cancelled out when the company's financial statements areconsolidated. So the only cost to the company is the variable cost to produce the item being sold internally, if

Part 1 : 10/05/14 22:20:20

(c) HOCK international, page 124

Page 125: Hock_Performance Mgmt

8/16/2019 Hock_Performance Mgmt

http://slidepdf.com/reader/full/hockperformance-mgmt 125/152

the selling division has the capacity to produce it. If the selling division has another, outside buyer, the cost tosell to the buying division instead of to the outside buyer will also include the lost contribution margin thatcould have been earned by selling to the outside buyer.

In this case, there is no opportunity cost of lost sales, because the question tells us that purchasing fromoutside would idle A's facilities now committed to producing units for B, and Division A cannot increase itssales to outsiders. Therefore, the total cost of producing the units internally is the variable cost of $30, which

is lower than the outside price of $40. Thus, from the perspective of the company as a whole, Division Bshould continue purchasing from Division A, despite the increased transfer price, because the cost to thecompany as a whole will be only $30 versus the $40 it would cost to purchase outside.

In reality, Department B should not simply accept the new transfer price of $50. The managers of the twodivisions should agree on a transfer price that is acceptable to both of them. It may be that seniormanagement will need to get involved in order to make that happen, if the two division heads cannot agree.However, that is not given as an answer choice, and of the answer choices given, this is the best choice.

B. The fixed costs are not relevant, since they will be the same regardless of where Division B gets the parts.

C. There are many acceptable methods of determining a transfer price. The best method for a particular situationdepends upon the circumstances. This question asks what is the best course of action from the perspective of the

company as a whole. Whenever the variable costs plus the opportunity cost of lost outside sales for the internalsupplier are lower than the price of the external supplier, the company as a whole is better off if the buying departmentbuys internally, even if the transfer price is higher than an outside supplier's price. In this case, there is no opportunitycost of lost sales, because the question tells us that purchasing from outside would idle A's facilities now committed toproducing units for B, and Division A cannot increase its sales to outsiders. Therefore, the total cost of producing theunits internally is the variable cost of $30, which is lower than the outside price of $40. Thus, in this situation, thetransfer cost used is not relevant to the decision.

D. Purchasing from outside in this situation will cause the company to pay more for each unit and to lose somecontribution to its fixed costs from the internal sales. When the variable costs plus the opportunity cost of lost outsidesales for the internal supplier are lower than the price of the external supplier, the company as a whole is better off ifthe buying department buys internally, even if the transfer price is higher than an outside supplier's price. In this case,there is no opportunity cost of lost sales, because the question tells us that purchasing from outside would idle A'sfacilities now committed to producing units for B, and Division A cannot increase its sales to outsiders. Therefore, the

total cost of producing the units internally is the variable cost of $30, which is lower than the outside price of $40.

Question 182 - CIA 596 III.90 - Manufacturing Input Variances -- Materials and Labor 

 A manufacturing cell's partial productivity can be measured using data on

 A. Inventory turnover.B. Inventory shrinkage.C. Scrap.D. Direct materials usage.

 A. Inventory turnover measures how many times inventory on hand is completely sold and replaced during a period.This is a measure not only of production but also purchasing.

B. Shrinkage occurs when inventory is lost through deterioration, obsolescence, theft, damage or waste. It does notreflect on manufacturing productivity.

C. A partial productivity measure is usually a ratio of output to the quantity of a single input (material, labor, capital).Scrap cannot be used to measure a manufacturing cell's partial productivity, because it is not an output nor is it aninput.

Part 1 : 10/05/14 22:20:20

(c) HOCK international, page 125

Page 126: Hock_Performance Mgmt

8/16/2019 Hock_Performance Mgmt

http://slidepdf.com/reader/full/hockperformance-mgmt 126/152

D. A partial productivity measure is usually a ratio of output to the quantity of a single input (material, labor,capital). This ratio measures the productivity of that one input to the production process. Examples of partialproductivity measures are number of units produced per pound of direct material, or per direct labor hour.Direct material usage is a partial productivity measure of a manufacturing cell because direct material is oneinput.

Question 183 - ICMA 10.P1.086 - Variance Analysis Concepts

 Arkin Co.’s controller has prepared a flexible budget for the year just ended, adjusting the original static budget for theunexpected large increase in the volume of sales. Arkin’s costs are mostly variable. The controller is pleased to notethat both actual revenues and actual costs approximated amounts shown on the flexible budget. If actual revenuesand actual costs are compared with amounts shown on the original (static) budget, what variances would arise?

 A. Revenue variances would be unfavorable and cost variances would be favorable.B. Both revenue variances and cost variances would be favorable.C. Both revenue variances and cost variances would be unfavorable.D. Revenue variances would be favorable and cost variances would be unfavorable.

 A.

Sales that are higher than expected will result in positive variances on a static budget report for both revenues andexpenses/costs, because both actual revenue and actual expenses or costs will be greater than budgeted.

However, a positive variance for a revenue item is favorable, whereas a positive variance for an expense or cost itemis unfavorable.

B. Sales that are higher than expected will result in positive variances on a static budget report, because both actualrevenue and actual expense or costs will be greater than budgeted. However, a positive variance for a revenue item isfavorable, whereas a positive variance for an expense or cost item is unfavorable.

C.

Sales that are higher than expected will result in positive variances on a static budget report for both revenues andexpenses/costs, because both actual revenue and actual expenses or costs will be greater than budgeted.

However, a positive variance for a revenue item is favorable, whereas a positive variance for an expense or cost itemis unfavorable.

D.

Sales that are higher than expected will result in positive variances on a static budget report for bothrevenues and expenses/costs, because both actual revenue and actual expenses or costs will be greater thanbudgeted.

For a revenue item, a positive variance (actual revenue is greater than budgeted revenue) is a favorable

variance. For an expense or cost item, a positive variance (actual is greater than budgeted) is an unfavorablevariance.

Question 184 - CMA 1293 3.23 - Manufacturing Input Variances -- Materials and Labor 

ChemKing uses a standard costing system in the manufacture of its single product. The 35,000 units of raw material ininventory were purchased for $105,000, and two units of raw material are required to produce one unit of final product.

Part 1 : 10/05/14 22:20:20

(c) HOCK international, page 126

Page 127: Hock_Performance Mgmt

8/16/2019 Hock_Performance Mgmt

http://slidepdf.com/reader/full/hockperformance-mgmt 127/152

In November, the company produced 12,000 units of product. The standard cost for material allowed for the output was$60,000, and there was an unfavorable quantity variance of $2,500.

The units of material used to produce November output totaled

 A. 12,500 units.B. 12,000 units.

C. 25,000 units.D. 23,000 units.

 A. This answer is based on the assumption that only one unit of raw material is required to produce one unit of finishedproduct. In fact, two units of raw material are required.

B. 12,000 is the number of units of finished product produced.

C.

In this question, we need to solve the materials quantity variance formula for AQ. The variance formula is (AQ− SQ) × SP. SQ can be calculated because the company produced 12,000 units, and two units of raw materialsare required for each unit of output. Therefore, the standard quantity for the actual output was 24,000 units.The standard price per unit of raw material can also be calculated, because the standard cost allowed for the

actual output was $60,000. Since the standard quantity for the actual output was 24,000 units, the standardprice per unit of raw materials was $60,000 ÷ 24,000, or $2.50. We are given the quantity variance as $2,500unfavorable. Therefore, the formula is:

(AQ − 24,000) × $2.50 = $2,500

Solving for AQ, we get AQ = 25,000, as follows:

2.5AQ − 60,000 = 2,5002.5AQ = 62,500AQ = 25,000

D. This answer is based on the assumption that the materials quantity variance was favorable. It was, in fact,

unfavorable.

Question 185 - CMA 694 3.28 - Responsibility Centers and Reporting Segments

DigitalTech uses an accounting system that charges costs to the manager who has the authority to make decisionsincurring the costs. For example, if a sales manager authorizes a rush order that results in additional manufacturingcosts, these additional costs are charged to the sales manager. This type of accounting system is known as

 A. Functional accounting.B. Transfer-pricing accounting.C. Contribution accounting.

D. Responsibility accounting.

 A. Functional accounting assigns costs based on the function that is performed. See the correct answer for a completeexplanation.

B. Transfer prices are the prices used in the 'sale' of goods within the company. See the correct answer for a completeexplanation.

C. Contribution margin analysis focuses on the variable revenues and costs and their behavior as activity levelschange. The selling price minus variable costs is the contribution margin. Fixed costs are then subtracted from the

Part 1 : 10/05/14 22:20:20

(c) HOCK international, page 127

Page 128: Hock_Performance Mgmt

8/16/2019 Hock_Performance Mgmt

http://slidepdf.com/reader/full/hockperformance-mgmt 128/152

contribution to determine the profit.

D. Responsibility accounting is a system in which an individual is held accountable (responsible) for thingsthat he or she is able to control. It is this person's responsibility to address any issues that arise related tothese costs that they control.

Question 186 - CMA 1292 3.13 - Responsibility Centers and Reporting Segments

When comparing performance report information for top management with that for lower-level management,

 A. Lower-level management reports are likely to contain more quantitative data and less financial data.B. Top management reports are more detailed.C. Top management reports show control over fewer costs.D. Lower-level management reports are typically for longer time periods.

A. Lower-level management reports are likely to contain more quantitative data and less financial data. This isbecause lower-level management is responsible for day-to-day operations and their reports containinformation about units of production, number of hours worked, etc. Top management, on the other hand, isconcerned about company's strategic goals, objectives and overall financial results.

B. Top management reports are less detailed than those for lower management.

C. Top management reports show control over more costs.

D. Lower-level management reports are typically for shorter time periods.

Question 187 - CMA 693 3.15 - Manufacturing Input Variances -- Materials and Labor 

The budgeted output for the month of May was for 9,000 units with direct material at $15 per unit. Direct labor wasbudgeted at 45 minutes per unit for a total of $81,000. Actual output for the month was 8,500 units with $127,500 indirect material and $77,775 in direct labor expense. The direct labor standard of 45 minutes was maintainedthroughout the month. Variance analysis of the performance for the month of May would show a(n)

 A. Unfavorable direct labor efficiency variance of $1,275.B. Favorable material usage variance of $7,500.C. Favorable direct labor efficiency variance of $1,275.D. Unfavorable direct labor price variance of $1,275.

 A.

Because the actual and budgeted hours were the same, there was no direct labor efficiency variance. The formula is:(AQ − SQ) × SP or (Actual Hours − Standard Hours for Actual Output) × Standard Rate. We know that the company

maintained the direct labor standard of 45 minutes, or 0.75 of an hour per unit, throughout the month. The standardlabor hour rate is $12 per hour, calculated as follows: 9,000 units planned × 0.75 of an hour per unit = total hoursplanned of 6,750. A total cost of $81,000 was planned, so therefore, the standard hourly rate must be $81,000 ÷ 6,750,or $12 per hour.

The Standard Hours for the Actual Output is 0.75 × 8,500, or 6,375 hours. Since we are told that the direct laborstandard was maintained, the Actual Hours must be the same as the Standard Hours, or 6,375.

Putting the numbers into the formula, we get (6,375 − 6,375) × $12 = 0. However, it is not really necessary to gothrough all those calculations. Because the problem states that the direct labor standard was maintained throughout

Part 1 : 10/05/14 22:20:20

(c) HOCK international, page 128

Page 129: Hock_Performance Mgmt

8/16/2019 Hock_Performance Mgmt

http://slidepdf.com/reader/full/hockperformance-mgmt 129/152

the period, we can know that the direct labor efficiency variance must be zero.

B.

This problem does not give enough information to enable calculation of a materials usage variance. The materialsquantity, or usage, variance is the difference between the actual quantity of materials that were used for the actualoutput and the standard quantity of materials allowed for the actual output, multiplied by the standard price per unit of

direct materials.

The problem does not give us either the standard quantity of materials allowed per unit of output or the actual quantityof materials used per unit of output. Nor does it give us the standard price per unit of direct materials. We are givenonly the standard price for the total direct materials per unit of finished product. The standard price for directmaterials per unit of finished product is not the same thing as the standard price per unit of direct materials.

Therefore, we have no way of telling whether the actual quantity used was different from the quantity planned for theoutput; and we have no way of telling whether the actual price per unit of direct materials was different from theplanned price per unit. By dividing the actual material cost by the actual number of units produced and the plannedmaterials costs by the planned number of units, we can determine that the actual direct materials cost was $15 perfinished unit of product and the planned direct materials cost was also $15 per finished unit of product, so we know thatthe total direct materials variance (price and quantity) was zero. However, that could consist of any variances at all forprice and quantity that would net to zero.

Since there is just not enough information given in this problem to permit a calculation of the materials usage variance(or the materials price variance, either), it cannot be said that the material usage variance was $7,500 favorable.

C.

Because the budgeted and actual hours were the same, there was no direct labor efficiency variance. The formula is:(AQ − SQ) × SP or (Actual Hours − Standard Hours for Actual Output) × Standard Rate. We know that the companymaintained the direct labor standard of 45 minutes, or 0.75 of an hour per unit, throughout the month. The standardlabor hour rate is $12 per hour, calculated as follows: 9,000 units planned × 0.75 of an hour per unit = total hoursplanned of 6,750. A total cost of $81,000 was planned, so therefore, the standard hourly rate must be $81,000 ÷ 6,750,or $12 per hour.

The Standard Hours for the Actual Output is 0.75 × 8,500, or 6,375 hours. Since we are told that the direct laborstandard was maintained, the Actual Hours must be the same as the Standard Hours, or 6,375.

Putting the numbers into the formula, we get (6,375 − 6,375) × $12 = 0. However, it is not really necessary to gothrough all those calculations. Because the problem states that the direct labor standard was maintained throughoutthe period, we can know that the direct labor efficiency variance must be zero.

D.

The direct labor price variance is calculated (AP − SP) × AQ or (Actual Rate − Standard Rate) × Actual Hours.The standard labor rate per hour is $12, calculated as follows: the labor standard is 45 minutes per unit, or0.75 of an hour. 9,000 units were planned, so total planned hours was 0.75 × 9,000 or 6,750 hours. Totalplanned cost was $81,000, so the standard cost per hour was $81,000 ÷ 6,750 hours, or $12 per hour.

The actual labor rate was $12.20, calculated as follows: The labor standard of 45 minutes per unit or 0.75 of anhour per unit was maintained. 8,500 units were actually produced, so the total actual hours was 0.75 × 8,500 or6,375 hours. Total actual cost was $77,775, so the actual cost per hour (the actual rate) was $77,775 ÷ 6,375hours, or $12.20 per hour.

Putting the numbers into the formula, we get ($12.20 − $12.00) × 6,375 = $1,275 Unfavorable.

Part 1 : 10/05/14 22:20:20

(c) HOCK international, page 129

Page 130: Hock_Performance Mgmt

8/16/2019 Hock_Performance Mgmt

http://slidepdf.com/reader/full/hockperformance-mgmt 130/152

Question 188 - CMA 1290 3.8 - Manufacturing Input Variances -- Overhead

Franklin Glass Works' production budget for the year ended November 30 was based on 200,000 units. Each unitrequires two standard hours of labor for completion. Total overhead was budgeted at $900,000 for the year, and thefixed overhead rate was estimated to be $3.00 per unit. Both fixed and variable overhead are assigned to the producton the basis of direct labor hours. The actual data for the year ended November 30 are presented as follows. Actual production in units 198,000

 Actual direct labor hours 440,000 Actual variable overhead $352,000 Actual fixed overhead $575,000

Franklin's fixed overhead spending variance for the year is:

 A. $5,750 favorable.B. $19,000 favorable.C. $25,000 favorable.D. $25,000 unfavorable.

 A. The fixed overhead budget/spending variance is the difference between the actual fixed overhead amount and thebudgeted fixed overhead costs. See the correct answer for a complete explanation.

B. The fixed overhead budget/spending variance is the difference between the actual fixed overhead amount and thebudgeted fixed overhead costs. See the correct answer for a complete explanation.

C. The fixed overhead budget/spending variance is the difference between the actual fixed overhead amountand the budgeted fixed overhead costs. The fixed overhead rate was estimated to be $3.00 per unit and200,000 units were scheduled for production, thus, the total budgeted fixed overhead was $600,000 ($3.00 ×200,000). Actual fixed overhead was $575,000. The fixed overhead spending variance was $25,000 favorable($575,000 − $600,000).

D. The fixed overhead budget/spending variance is the difference between the actual fixed overhead amount and thebudgeted fixed overhead costs. See the correct answer for a complete explanation.

Question 189 - CMA 1294 3.28 - Manufacturing Input Variances -- Overhead

Water Control Inc. manufactures water pumps and uses a standard cost system. The standard factory overhead costsper water pump are based on direct labor hours and are as follows:

Variable overhead (4 hours at $8/hour) - $32Fixed overhead (4 hours at $5/hour*) - $20Total overhead cost per unit - $52* Based on a capacity of 100,000 direct labor hours per month.

The following additional information is available for the month of November:

22,000 pumps were produced although 25,000 had been scheduled for production.94,000 direct labor hours were worked at a total cost of $940,000.The standard direct labor rate is $9 per hour.The standard direct labor time per unit is 4 hours.Variable overhead costs were $740,000.Fixed overhead costs were $540,000.

The variable overhead efficiency variance for November was

 A. $48,000 unfavorable.

Part 1 : 10/05/14 22:20:20

(c) HOCK international, page 130

Page 131: Hock_Performance Mgmt

8/16/2019 Hock_Performance Mgmt

http://slidepdf.com/reader/full/hockperformance-mgmt 131/152

B. $60,000 favorable.C. $96,000 unfavorable.D. $200,000 unfavorable.

A.

The variable overhead efficiency variance is calculated as follows: (Actual Activity Level of VOH allocation

base used for Actual Output − Standard Activity Level of VOH allocation base allowed for Actual Output) ×Standard Application Rate.

The actual activity level is 94,000 direct labor hours. The standard activity level is calculated as the standardof 4 hours of direct labor per unit multiplied by the 22,000 units produced during the period, or 88,000 directlabor hours. The standard application rate is $8/hour.

Therefore, the variable overhead efficiency variance is (94,000 − 88,000) × $8 = $48,000 Unfavorable. Thevariance is unfavorable because the actual activity level was greater than the standard activity level.

B. This answer uses capacity, not actual hours. See the correct answer for a complete explanation.

C. This is the difference between the capacity hours (100,000) and the standard hours allowed for the actual output(88,000) multiplied by the standard application rate ($8). See the correct answer for a complete explanation.

D. This is the difference between the actual direct labor costs of $940,000 and the actual variable overhead costs of$740,000, which does not mean anything. See the correct answer for a complete explanation.

Question 190 - CIA 593 IV.14 - Sales and Market Variances

For a single-product company, the sales volume variance is

 A. The difference between actual and master budget sales volume, times actual unit contribution margin.B. The difference between flexible budget and actual sales volume, times master budget unit contribution margin.

C. The difference between flexible budget and master budget sales volume, times actual budget unit contributionmargin.D. The difference between flexible budget and master budget sales volume, times master budget unit contributionmargin.

 A. The sales volume variance formula is: (Actual Sales Volume − Budgeted Sales Volume) × Budgeted Contributionper Unit. As we can see from the formula, the budgeted unit contribution margin is used in the calculation, not theactual contribution margin.

B. There is no difference between the flexible budget and the actual sales volume, as the actual sales volume is usedin calculating the flexible budget figures because the flexible budget uses the actual sales level.

C. The sales volume variance basic formula is: (Actual Sales Volume − Budgeted Sales Volume) × BudgetedContribution per Unit. As we can see from the formula, the budgeted unit contribution margin is used in the calculation,

not the actual unit contribution margin. The actual sales volume and the flexible budget sales volume are the sameamount, because the flexible budget uses the actual level of output.

D. The sales volume variance formula is: (Actual Sales Volume − Budgeted Sales Volume) × BudgetedContribution per Unit. The budgeted contribution per unit is defined at the beginning of the year using themaster budget. The budgeted sales volume is also defined at the beginning of the year and it is a masterbudget figure. Actual sales volume is the same thing as the flexible budget sales volume, because the flexiblebudget uses the actual level of output. Thus, the sales volume variance is the difference between the flexiblebudget and master budget sales volume, times master budget unit contribution margin.

Part 1 : 10/05/14 22:20:20

(c) HOCK international, page 131

Page 132: Hock_Performance Mgmt

8/16/2019 Hock_Performance Mgmt

http://slidepdf.com/reader/full/hockperformance-mgmt 132/152

Question 191 - ICMA 10.P1.113 - Manufacturing Input Variances -- Overhead

The JoyT Company manufactures Maxi Dolls for sale in toy stores. In planning for this year, JoyT estimated variablefactory overhead of $600,000 and fixed factory overhead of $400,000. JoyT uses a standard costing system, andfactory overhead is allocated to units produced on the basis of standard direct labor hours. The denominator level ofactivity budgeted for this year was 10,000 direct labor hours, and JoyT used 10,300 actual direct labor hours.

Based on the output accomplished during this year, 9,900 standard direct labor hours should have been used. Actualvariable factory overhead was $596,000, and actual fixed factory overhead was $410,000 for the year. Based on thisinformation, the variable overhead spending variance for JoyT for this year was

 A. $4,000 favorable.B. $22,000 favorable.C. $2,000 unfavorable.D. $24,000 unfavorable.

 A. This is the actual variable overhead of $596,000 minus estimated variable overhead of $600,000. The standardamount used should be the standard amount of variable overhead allowed for the actual quantity of the variableoverhead allocation base used for the actual output, or (10,300 × $60). See the correct answer for a full explanation.

B.

The Variable Overhead Spending Variance is the actual amount of variable overhead incurred minus thestandard amount of variable overhead allowed for the actual quantity of the variable overhead allocation baseused for the actual output.

The actual amount of variable overhead incurred is $596,000.

The standard application rate is calculated by dividing the $600,000 estimated variable overhead by the 10,000budgeted direct labor hours, which results in an application rate of $60 per direct labor hour. The companyactually used 10,300 direct labor hours. Therefore, the standard amount of variable overhead allowed for theactual quantity of the variable overhead allocation base used for the actual output was $60 × 10,300, or

$618,000.

The Variable Overhead spending Variance is $596,000 − $618,000, which equals $(22,000). Since overhead is acost, a negative variance is favorable.

C. This answer is the difference between the actual variable overhead cost incurred of $596,000 and (standard hoursfor the actual output of 9,900 × estimated overhead cost per hour of $60). The use of the actual cost is correct, but thestandard amount used should be the standard amount of variable overhead allowed for the actual quantity of thevariable overhead allocation base used for the actual output, or (10,300 × $60). See the correct answer for a fullexplanation.

D. This is the standard cost per direct labor hour of $60 multiplied by the difference between 10,300 actual DLH usedand 9,900 standard DLH for the actual output, i.e., (10,300 − 9,900) × $60. The variable overhead spending variance isthe actual amount of variable overhead incurred minus the standard amount of variable overhead allowed for the actualquantity of the variable overhead allocation base used for the actual output. See the correct answer for a fullexplanation.

Question 192 - ICMA 10.P1.119 - Responsibility Centers and Reporting Segments

Sara Bellows, manager of the telecommunication sales team, has the following department budget.Billings - long distance $350,000

Part 1 : 10/05/14 22:20:20

(c) HOCK international, page 132

Page 133: Hock_Performance Mgmt

8/16/2019 Hock_Performance Mgmt

http://slidepdf.com/reader/full/hockperformance-mgmt 133/152

Billings - phone card 75,000Billings - toll free 265,000

Her responsibility center is best described as a

 A. revenue center.B. investment center.

C. cost center.D. profit center.

A. A revenue center is responsible only for revenues. This is a sales department, and the "billings" areinvoices issued to customers for charges they have incurred for telecommunications services provided,which is revenue. A sales department is a revenue center.

B. An investment center is responsible for profit (revenues and costs) and also for invested capital such as equipmentand for providing a return on that capital. The "billings" in this question are invoices issued to customers for chargesthey have incurred for telecommunications services provided, which is revenue. However, the department budget doesnot include any items representing expenses, capital or return on capital, so it is not an investment center.

C. A cost center is responsible only for the incurrence of costs. This is a sales department, and the "billings" areinvoices issued to customers for charges they have incurred for telecommunications services provided. Therefore, thisis not a cost center.

D. A profit center is a department that is responsible for both revenues and expenses. The "billings" in this question areinvoices issued to customers for charges they have incurred for telecommunications services provided, which isrevenue. However, the department budget does not include any items representing expenses, so it is not a profit center.

Question 193 - ICMA 10.P1.099 - Manufacturing Input Variances -- Overhead

For a given time period, a company had a favorable material quantity variance, a favorable direct labor efficiencyvariance, and a favorable fixed overhead volume variance. Of the following, the one factor that could not have caused

all three variances is

 A. the purchase of more efficient machinery.B. an increase in production supervision.C. the use of lower-skilled workers.D. the purchase of higher quality materials.

 A.

The purchase of more efficient machinery could cause all three variances.

Use of more efficient machinery could lead to a favorable material quantity variance if less direct materials were ruinedin production due to better machinery being used.

The purchase of more efficient machinery could also lead to a favorable direct labor efficiency variance, becauseemployees would be able to produce more units of finished goods when the machinery they are using is more efficient.

The purchase of more efficient machinery could also cause a favorable fixed overhead volume variance, for the samereason. The fixed overhead volume variance is a measurement of actual production volume compared with expectedproduction volume. If a greater number of units is produced than was expected, the fixed overhead production-volumevariance will be favorable, and that could happen if more efficient machinery were being used, because more unitscould be produced.

B.

Part 1 : 10/05/14 22:20:20

(c) HOCK international, page 133

Page 134: Hock_Performance Mgmt

8/16/2019 Hock_Performance Mgmt

http://slidepdf.com/reader/full/hockperformance-mgmt 134/152

 An increase in production supervision could cause all three variances.

 An increase in production supervision could lead to a favorable material quantity variance if less direct materials wereruined in production due to increased supervision of production employees.

 An increase in production supervision could also lead to a favorable direct labor efficiency variance, becauseemployees would be able to produce more units of finished goods when they are being supervised more closely, since

more mistakes would be noticed and corrected sooner.

 An increase in production supervision could also cause a favorable fixed overhead volume variance, for the samereason. The fixed overhead volume variance is a measurement of actual production volume compared with expectedproduction volume. If a greater number of units is produced than was expected, the fixed overhead production-volumevariance will be favorable, and that could happen if production supervision is increased, because more units could beproduced.

C.

The use of lower-skilled workers could not be a cause of these three favorable variances.

Lower-skilled employees will make more mistakes than highly-skilled employees. That will result in morematerials being used than the standard (an unfavorable direct material quantity variance). It will also result ina greater amount of direct labor time required than the standard (an unfavorable direct labor efficiencyvariance). And it will result in an unfavorable fixed overhead production-volume variance, becauselower-skilled employees would require more time to produce each unit. In addition, a greater number of unitswould be rejected as defective. Both of these would lead to a lower production volume than was plannedwhich creates an unfavorable fixed overhead production-volume variance.

D.

The purchase of higher quality materials could cause all three variances.

Use of higher quality materials could lead to a favorable material quantity variance because less direct material wouldneed to be discarded due to defects, so less direct materials would be used.

The purchase of higher quality materials could also lead to a favorable direct labor efficiency variance, becauseemployees would be able to produce more units of finished goods when fewer units are rejected as defective due todefective materials.

The purchase of higher quality materials could also cause a favorable fixed overhead volume variance, for the samereason. The fixed overhead volume variance is a measurement of actual production volume compared with expectedproduction volume. If a greater number of units is produced than was expected, the fixed overhead production-volumevariance will be favorable, and that could happen if higher quality materials are being used because fewer units wouldbe rejected as defective due to defective materials.

Question 194 - CMA 696 3.24 - Manufacturing Input Variances -- Materials and Labor 

 Ardmore Enterprises uses a standard cost system in its small appliance division. The standard cost of manufacturingone unit of Zeb is as follows:Materials - 60 pounds at $1.50 per pound $ 90Labor - 3 hours at $12 per hour 36Factory overhead - 3 hours at $8 per hour 24Total standard cost per unit $150

The budgeted variable factory overhead rate is $3 per labor hour, and the budgeted fixed factory overhead is $27,000per month. During May, Ardmore produced 1,650 units of Zeb compared with a normal capacity of 1,800 units. The

Part 1 : 10/05/14 22:20:20

(c) HOCK international, page 134

Page 135: Hock_Performance Mgmt

8/16/2019 Hock_Performance Mgmt

http://slidepdf.com/reader/full/hockperformance-mgmt 135/152

actual cost per unit was as follows:Materials (purchased and used) - 58 pounds at $1.65 per pound $ 95.70Labor - 3.1 hours at $12 per hour 37.20Factory overhead - $39,930 per 1,650 units 24.20Total actual cost per unit $157.10

The labor rate variance for May is

 A. $1,920 favorable.B. $0C. $4,950 favorable.D. $4,950 unfavorable.

 A. This is the difference between the actual total overhead incurred and the flexible budget overhead. This is not thelabor rate variance. See the correct answer for a complete explanation.

B. The labor rate variance formula is: (Actual Rate − Standard Rate) × Actual Hours. The labor rate variance iszero as the standard rate equals the actual rate of $12.00 per hour.

C. This is the material quantity variance. The question asks for the labor rate variance. See the correct answer for acomplete explanation.

D. The labor rate variance is not unfavorable. See the correct answer for a complete explanation.

Question 195 - CMA 1294 3.20 - Responsibility Centers and Reporting Segments

Fairmount Inc. uses an accounting system that charges costs to the manager who has been delegated the authority tomake the decisions incurring the costs. For example, if the sales manager accepts a rush order that will result in higherthan normal manufacturing costs, these additional costs are charged to the sales manager because the authority toaccept or decline the rush order was given to the sales manager. This type of accounting system is known as

 A. Transfer price accounting.B. Functional accounting.C. Reciprocal allocation.D. Responsibility accounting.

 A. Transfer price accounting is related to the determination of the price to charge for intercompany sales.

B. Functional accounting allocates costs according to the function they represent. Functional accounting does not usethe responsibility for the incurrence of cost to allocate the costs.

C. Reciprocal allocation is a method of allocating the costs of service departments to other service departments andthe production departments.

D. Responsibility accounting is a system in which costs are allocated to managers and/or departments based

on who is responsible for the incurrence of the costs. This is the method described in the question.

Question 196 - CMA 693 3.26 H1 - Manufacturing Input Variances -- Overhead

Which one of the following variances is of least significance from a behavioral control perspective?

 A. Unfavorable material quantity variance amounting to 20% of the quantity allowed for the output attained.

Part 1 : 10/05/14 22:20:20

(c) HOCK international, page 135

Page 136: Hock_Performance Mgmt

8/16/2019 Hock_Performance Mgmt

http://slidepdf.com/reader/full/hockperformance-mgmt 136/152

B. Favorable labor rate variance resulting from an inability to hire experienced workers to replace retiring workers.C. Fixed overhead volume variance resulting from management's decision midway through the fiscal year to reduce itsbudgeted output by 20%.D. Unfavorable labor efficiency variance amounting to 10% more than the budgeted hours for the output attained.

 A. An unfavorable material quantity variance means that workers used more material than was budgeted. Varianceanalysis is supposed to give an answer to the question 'who is responsible for the variance'. In this situation the

excessive use of material could come from either unskilled workers who could spoil some of material or from purchaseof poor quality material, or both. So this variance is quite significant from a behavioral control perspective.

B. The result of the favorable labor rate variance caused by an inability to hire experienced workers to replace retiringworkers could be an unfavorable labor efficiency variance or even unfavorable material quantity variance (if wasteoccurs). So this variance is quite significant from a behavioral control perspective.

C. The fixed overhead volume variance is the difference between the budgeted amount of fixed overhead andthe amount of fixed overhead applied (standard rate × standard input for the actual level of output). It is notvery much involved with performance of employees, as in most cases top management is responsible for thisvariance. This variance could be caused by different reasons: the economic situation, weather or change inplanned output as it is in this question. So this variance is least significant from a behavioral controlperspective.

D. An unfavorable labor efficiency variance means that workers spent more time to perform work than was budgeted.There could be many reasons to this: work was performed by new unskilled workers, poor usage of materials ortechnology, etc. So this variance is quite significant from a behavioral control perspective.

Question 197 - CMA 692 3.14 - Responsibility Centers and Reporting Segments

The most fundamental responsibility center affected by the use of market-based transfer prices is a(n):

 A. Profit center.B. Cost center.

C. Production center.D. Investment center.

A.

A profit center is responsible for both revenues and costs (and therefore profit). A transfer price, whetherreceived as revenue or paid as a cost, does affect a profit center; and a profit center is the most fundamental(basic) responsibility center that would be affected.

An investment center, which is responsible for revenues, costs and investment, would also be affected bytransfer prices; but an investment center is not the most basic, or fundamental, responsibility center to beaffected by transfer prices.

B. A cost center is not responsible for revenues, so it would not be affected by transfer prices.

C. Production centers are not always responsible for revenues and costs so the transfer prices that are used would notalways affect a production center.

D. While an investment center would be affected by the transfer prices being used, an investment center is not themost fundamental (basic) of the centers that is affected by transfer prices.

Part 1 : 10/05/14 22:20:20

(c) HOCK international, page 136

Page 137: Hock_Performance Mgmt

8/16/2019 Hock_Performance Mgmt

http://slidepdf.com/reader/full/hockperformance-mgmt 137/152

Question 198 - CIA 592 IV.19 - Responsibility Centers and Reporting Segments

Division Z of a company produces a component that it currently sells to outside customers for $20 per unit. At itscurrent level of production, which is 60% of capacity, Division Z's fixed cost of producing this component is $5 per unitand its variable cost is $12 per unit. Division Y of the same company would like to purchase this component fromDivision Z for $10. Division Z has enough excess capacity to fill Division Y's requirements. The managers of bothdivisions are compensated based upon reported profits. Which of the following transfer prices will maximize total

company profits and be most equitable to the managers of Division Y and Division Z?

 A. $18 per unit.B. $22 per unit.C. $12 per unit.D. $20 per unit.

A. The transfer price should be between the variable costs of production ($12) and the market price ($20). $18is the best answer because it is the only answer choice that is in between these amounts.

B. The transfer price should be between the variable costs of production ($12) and the market price ($20). $22 isoutside of this range.

C. The transfer price should be between the variable costs of production ($12) and the market price ($20). While $12 is

at the edge of this range, the selling division would earn no contribution from the sale, so it wil l not gain anything fromthis contract.

D. The transfer price should be between the variable costs of production ($12) and the market price ($20). $18 is theonly option between these amounts. While $20 is at the edge of this range, the purchasing division will have no savingscompared to the outside purchase price so would not gain anything by purchasing internally.

Question 199 - CIA 594 III.44 - Responsibility Centers and Reporting Segments

 A firm prepared a segmented income statement that included the following data for its suburban marketing segment:Fixed costs controllable by the suburban marketing segment manager $150,000Fixed suburban marketing costs controllable by corporate management $250,000Fixed manufacturing costs allocated to the suburban marketing segment $110,000Variable manufacturing costs $200,000Variable selling costs $100,000Variable administrative costs $130,000Net sales $950,000

The best measure of the economic performance of the suburban marketing segment is:

 A. $370,000B. $120,000C. $520,000D. $10,000

 A. This answer does not include the fixed costs controllable by the company's management. This needs to be includedin segment performance, but it would not be included in segment manager performance.

B.

The suburban marketing segment should be measured by what it controls and by fixed costs traceable to it,even if not controlled by it. This is called the segment margin, and the expenses included in it are thesegment's variable costs, fixed costs it controls and fixed costs that are controllable by corporatemanagement.

Part 1 : 10/05/14 22:20:20

(c) HOCK international, page 137

Page 138: Hock_Performance Mgmt

8/16/2019 Hock_Performance Mgmt

http://slidepdf.com/reader/full/hockperformance-mgmt 138/152

Segment manager performance includes only costs controllable by the segment manager. Segment managerperformance would not include fixed costs controllable by the corporate management.

However, in this question we are measuring the segment, not the segment manager, because the questionasks for the best measure of the economic performance of the segment. The segment margin includes thefollowing items:

Net sales $950,000Fixed costs controllable by suburban marketing mgr. 150,000

Fixed costs controllable by corporate management 250,000Total variable costs 430,000

Segment margin $120,000

C. This answer is the contribution margin of sales minus variable costs and fails to take into account the fixed coststhat are controllable by the segment.

D. This answer incorrectly includes the allocated fixed costs. See the correct answer for a complete explanation.

Question 200 - ICMA 10.P1.137 - Performance Measures

For several years, Northern Division of Marino Company has maintained a positive residual income. Northern iscurrently considering investing in a new project that will lower the division’s overall return on investment (ROI) butincrease its residual income. What is the relationship between the expected rate of return on the new project, thefirm’s cost of capital, and the division’s current ROI?

 A. The expected rate of return on the new project is higher than the division’s current return on investment, but lowerthan the firm’s cost of capital.B. The expected rate of return on the new project is higher than the firm’s cost of capital, but lower than the division’scurrent return on investment.

C. The division’s current return on investment is higher than the expected rate of return on the new project, but lowerthan the firm’s cost of capital.D. The firm’s cost of capital is higher than the expected rate of return on the new project, but lower than the division’scurrent return on investment.

 A.

The question tells us that the expected rate of return on the new project is lower than the division’s overall return oninvestment, so the first part of this answer choice is incorrect.

The second part of this answer choice is also incorrect. The expected rate of return on the new project cannot be lowerthan the firm’s cost of capital. If it were, the project would not increase the company’s residual income. Furthermore,the firm would lose money on the project because the financing for the project (the cost of capital) would cost morethan the project would earn. The result would be that residual income would decrease, not increase.

B.

The expected rate of return on the new project must be higher than the firm’s cost of capital. If it were not, theproject would not increase the company’s residual income. If the expected rate of return were lower than thefirm’s cost of capital, the firm would lose money on the project because the financing for the project (the costof capital) would cost more than the project would earn. The result would be that residual income woulddecrease. Since we are told that the project will increase residual income, the project’s expected rate of returnmust be higher than the company’s cost of capital.

Part 1 : 10/05/14 22:20:20

(c) HOCK international, page 138

Page 139: Hock_Performance Mgmt

8/16/2019 Hock_Performance Mgmt

http://slidepdf.com/reader/full/hockperformance-mgmt 139/152

We know the expected rate of return on the new project is lower than the division’s overall return oninvestment, because the question tells us it is.

C. The question tells us that the division is maintaining a positive residual income. If the division's return on investmentwere lower than the firm's cost of capital, then it would have a negative residual income. Since it does not, its return oninvestment must be higher than the firm's cost of capital, not lower.

D. The firm's cost of capital cannot be higher than the expected rate of return on the new project. If it were, the projectwould not increase the company’s residual income. Furthermore, the firm would lose money on the project because thefinancing for the project (the cost of capital) would cost more than the project would earn. The result would be thatresidual income would decrease, not increase.

Question 201 - CMA 1294 3.27 - Manufacturing Input Variances -- Overhead

Water Control Inc. manufactures water pumps and uses a standard cost system. The standard factory overhead costsper water pump are based on direct labor hours and are as follows:

Variable overhead (4 hours at $8/hour) - $32

Fixed overhead (4 hours at $5/hour*) - $20Total overhead cost per unit - $52* Based on a capacity of 100,000 direct labor hours per month.

The following additional information is available for the month of November:

22,000 pumps were produced although 25,000 had been scheduled for production.94,000 direct labor hours were worked at a total cost of $940,000.The standard direct labor rate is $9 per hour.The standard direct labor time per unit is 4 hours.Variable overhead costs were $740,000.Fixed overhead costs were $540,000.

The variable overhead spending variance for November was

 A. $12,000 favorable.B. $48,000 unfavorable.C. $60,000 favorable.D. $40,000 unfavorable.

A.

The variable overhead spending variance is calculated as follows: (Actual VOH Cost/Unit of allocation baseactually used − Standard Application Rate) × Actual Quantity of variable overhead allocation base used for theactual output.

The actual VOH cost/unit of the allocation base actually used was the total actual variable overhead cost of

$740,000 divided by the actual number of direct labor hours worked of 94,000, or $7.8723 per direct labor hour.The standard application rate of variable overhead was $8 per direct labor hour. The actual quantity of thevariable overhead allocation base used for the actual output was 94,000.

Therefore, the variable overhead spending variance was ($7.8723 − $8) × 94,000, which equals ($12,000)favorable.

B. This is the variable overhead efficiency variance. See the correct answer for a complete explanation.

C.

Part 1 : 10/05/14 22:20:20

(c) HOCK international, page 139

Page 140: Hock_Performance Mgmt

8/16/2019 Hock_Performance Mgmt

http://slidepdf.com/reader/full/hockperformance-mgmt 140/152

The variable overhead spending variance is calculated as follows: (Actual VOH Cost/Unit of allocation base actuallyused − Standard Application Rate) × Actual Quantity of variable overhead allocation base used for the actual output.

This answer results from using the budgeted hours (100,000), not the actual hours (94,000) to calculate the actualvariable overhead cost/unit of the allocation base actually used; and also to multiply by the actual quantity of thevariable overhead allocation base used for the actual output.

D. This is the fixed overhead spending variance. See the correct answer for a complete explanation.

Question 202 - CMA 693 3.27 - Performance Measures

Which one of the following statements pertaining to the return on investment (ROI) as a performance measurement isincorrect?

 A. The use of ROI may lead managers to reject capital investment projects that can be justified by using discountedcash flow models.B. ROI relies on financial measures that are capable of being independently verified while other forms of performancemeasures are subject to manipulation.C. When the average age of assets differs substantially across segments of a business, the use of ROI may not beappropriate.D. The use of ROI can make it undesirable for a skillful manager to take on trouble-shooting assignments such asthose involving turning around unprofitable divisions.

 A. This is a true statement. Under ROI, a manager may reject a project that has a positive net present value onlybecause the return is not high enough when compared with the manager's historical ROI.

B. ROI uses many of the same measures that other methods use and is therefore susceptible to the samemanipulation as other methods are.

C. This is a true statement. When there are substantially different ages of assets in different segments, the use of ROImay not be appropriate. This is because the assets will have different book values based on their age and so each

segment would have a different basis for what may be essentially the same assets.

D. This is a true statement. If ROI is used to evaluate managers, a good manager may not want to be affiliated with adivision that is performing poorly because the poorly performing division will have a difficult time generating therequired return. And yet, a division that is performing poorly needs the skills of a good manager if it is to have anychance of turning around.

Question 203 - CMA 1294 3.30 - Manufacturing Input Variances -- Materials and Labor 

Water Control Inc. manufactures water pumps and uses a standard cost system. The standard factory overhead costsper water pump are based on direct labor hours and are as follows:

Variable overhead (4 hours at $8/hour) - $32Fixed overhead (4 hours at $5/hour*) - $20Total overhead cost per unit - $52* Based on a capacity of 100,000 direct labor hours per month.

The following additional information is available for the month of November:

22,000 pumps were produced although 25,000 had been scheduled for production.94,000 direct labor hours were worked at a total cost of $940,000.

Part 1 : 10/05/14 22:20:20

(c) HOCK international, page 140

Page 141: Hock_Performance Mgmt

8/16/2019 Hock_Performance Mgmt

http://slidepdf.com/reader/full/hockperformance-mgmt 141/152

The standard direct labor rate is $9 per hour.The standard direct labor time per unit is 4 hours.Variable overhead costs were $740,000.Fixed overhead costs were $540,000.

The direct labor efficiency variance for November was

 A. $108,000 favorable.B. $60,000 favorable.C. $54,000 unfavorable.D. $120,000 favorable.

 A. This is the difference between the capacity hours (100,000) and the standard hours (88,000), multiplied by thestandard rate. However, in the direct labor efficiency variance, we need to use actual hours (94,000) instead of capacityhours (100,000). Furthermore, the actual hours used was greater than the standard hours for the actual output, and sothe variance is unfavorable. See the correct answer for a complete explanation.

B.

The labor efficiency variance is calculated follows: (Actual Hours − Standard Hours for Actual Output) × Standard Rate.There were two mistakes made to get this result.

First the actual labor rate ($10) is used instead of the standard rate of $9. Second, the capacity hours (100,000) isused instead of standard hours for the actual output (88,000). See the correct answer for a complete explanation.

C. The labor efficiency variance is calculated follows: (Actual Hours − Standard Hours for Actual Output) ×Standard Rate. The actual hours worked are 94,000, and the standard hours for the actual output are 88,000 (4hours per unit multiplied by 22,000 units produced). The standard labor hourly rate is $9. The labor efficiencyvariance is (94,000 − 88,000) × $9 = 54,000 unfavorable. The variance is unfavorable (positive) because theactual hours were greater than the standard hours for the actual output, and this is a cost variance.

D.

The labor efficiency variance is calculated follows: (Actual Hours − Standard Hours for Actual Output) × Standard Rate.There are three mistakes made to get this answer.

First, the actual labor rate ($10) is used instead of the standard rate of $9. Second, the standard hours (88,000) is usedinstead of actual hours (94,000). Third, the capacity hours (100,000) is used instead of standard hours (88,000). Seethe correct answer for a complete explanation.

Question 204 - CIA 594 III.74 - Manufacturing Input Variances -- Materials and Labor 

 A company manufactures one product and has a standard cost system. In April the company had the followingexperience:

  Direct Materials Direct Labor Actual $/unit of input (lbs. & hrs.) $28 $18

Standard price/unit of input $24 $20

Standard inputs allowed per unit of output 10 4

 Actual units of input 190,000 78,000

 Actual units of output 20,000 20,000

The direct labor rate variance for April is:

Part 1 : 10/05/14 22:20:20

(c) HOCK international, page 141

Page 142: Hock_Performance Mgmt

8/16/2019 Hock_Performance Mgmt

http://slidepdf.com/reader/full/hockperformance-mgmt 142/152

 A. $40,000 unfavorable.B. $156,000 favorable.C. $156,000 unfavorable.D. $240,000 favorable.

 A.

The labor rate variance is calculated as follows: (Actual Rate − Standard Rate) × Actual Hours (i.e., actual units ofinput). An answer of $40,000 results from using the formula (Standard Rate − Actual Rate) × Actual Units of Output.

B. The labor rate variance is calculated as follows: (Actual Rate − Standard Rate) × Actual Hours (i.e., actualunits of input). All the components of the formula are in the data given. The actual rate is $18, and thestandard rate is $20. The actual hours are 78,000. Thus, the variance is ($18 − $20) × 78,000 = $(156,000). Thedirect labor rate variance for April is favorable because the actual labor rate was less than the standard laborrate.

C. The actual labor rate was less than the standard labor rate, which creates a favorable variance. See the correctanswer for a complete explanation.

D. This is the direct materials efficiency variance. However, the question asks for the direct labor rate variance. See thecorrect answer for a complete explanation.

Question 205 - CMA 695 3.27 - Sales and Market Variances

Clear Plus, Inc. manufactures and sells boxes of pocket protectors. The static master budget and the actual results forMay appear below.

 Actual

StaticBudget

Unit sales 12,000 10,000 Sales $132,000$100,000

Variable costs of sales 70,800 60,000 Contribution margin 61,200 40,000Fixed costs 32,000 30,000 Operating income $ 29,200$ 10,000

Which one of the following statements concerning Clear Plus, Inc.'s actual results for May is correct?

 A. The flexible budget variance is $8,000 favorable.B. The sales volume variance is $8,000 favorable.C. The flexible budget variable cost variance is $10,800 unfavorable.D. The sales price variance is $32,000 favorable.

 A. To answer such a question we have to solve for each possible suggested statement. It is better to start with theeasiest ones. When it is not otherwise specified, the flexible budget variance equals the difference between actualoperating income and the flexible budget operating income. The flexible budget contribution margin equals actual unitstimes standard contribution margin or $48,000 ($4.00 standard contribution margin × 12,000 actual quantity). Fixedcosts in the flexible budget are the same as fixed costs in the static budget. Therefore, the flexible budget operatingincome equals flexible budget contribution margin less the master budget (static budget) fixed costs: $18,000 ($48,000− $30,000). Now we can determine the flexible budget variance as $11,200 favorable: ($29,200 − $18,000). So thisstatement is not correct.

B. To answer such a question we have to solve for each possible suggested statement. It is better to start with

Part 1 : 10/05/14 22:20:20

(c) HOCK international, page 142

Page 143: Hock_Performance Mgmt

8/16/2019 Hock_Performance Mgmt

http://slidepdf.com/reader/full/hockperformance-mgmt 143/152

the easiest ones. The sales volume variance measures the impact of difference between actual sales volumeand budgeted sales volume. When it is not specified what item's sales volume variance is referred to, we canassume the contribution margin. The sales volume variance is calculated as follows: (Actual Sales Volume −Budgeted Sales Volume) × Standard Contribution per Unit. The actual quantity was 12,000 and the standardquantity was 10,000. The standard contribution per unit is $4.00 ($40,000 ÷ 10,000). The sales volume varianceis therefore $8,000 favorable: (12,000 − 10,000) × $4.00. The variance is favorable as the actual quantity of unitssold is greater than budgeted. So this statement is correct.

C. To answer such a question we have to solve for each possible suggested statement. It is better to start with theeasiest ones. The flexible budget variable cost variance is the difference between actual variable costs and the flexiblebudget variable costs. The actual variable cost is given as $70,800. The flexible budget variable cost is calculated asthe standard unit variable cost multiplied by the actual level of sales volume, which was 12,000. We can calculate thestandard unit variable cost using the static (master) budget figures given: $60,000 ÷ 10,000 = $6.00. So the flexiblebudget variable cost is $6.00 × 12,000 = $72,000. The flexible budget variable cost variance thus is $1,200 favorable($70,800 − $72,000). So this is not a correct statement.

D. To answer such a question we have to solve for each possible suggested statement. It is better to start with theeasiest ones. The sales price variance measures the impact of the difference caused by a variance in price. When it isnot specified what item's sales price variance is referred to, we can assume the contribution margin. The sales pricevariance for the contribution margin is calculated as follows: (Actual Contribution per unit − Standard Contribution perunit) × Actual Quantity. Actual contribution per unit is $5.10 ($61,200 ÷ 12,000), and the standard contribution margin is$4.00 ($40,000 ÷ 10,000). The actual quantity is 12,000. The sales price variance is therefore $13,200 favorable:($5.10 − $4.00) × 12,000. So this statement is not correct.

Question 206 - CMA 697 3.22 - Manufacturing Input Variances -- Materials and Labor 

The controller for Durham Skates is reviewing the production cost report for July. An analysis of direct materials costsreflects an unfavorable flexible budget variance of $25. The plant manager believes this is excellent performance on aflexible budget for 5,000 units of direct materials. However, the production supervisor is not pleased with this resultbecause he claims to have saved $1,200 in materials cost on actual production using 4,900 units of direct materials.The standard materials cost is $12 per unit. Actual materials used for the month amounted to $60,025.

The actual average cost per unit for materials was

 A. $12.01B. $12.25C. $12.00D. $12.24

 A.

This is not the correct answer. Please see the correct answer for an explanation.

We have been unable to determine how to calculate this incorrect answer choice. If you have calculated it, please letus know how you did it so we can create a full explanation of why this answer choice is incorrect. Please send us an

email at [email protected]. Include the full Question ID number and the actual incorrect answer choice --not its letter, because that can change with every study session created. The Question ID number appears in the upperright corner of the ExamSuccess screen. Thank you in advance for helping us to make your HOCK study materialsbetter.

B. Actual materials used for the month amounted to $60,025, and 4,900 units of direct materials were used.The actual average cost per unit of material was $12.25 ($60,025 ÷ 4,900).

C. This is standard cost per unit of material not the actual average cost per unit of material. See the correct answer fora complete explanation.

Part 1 : 10/05/14 22:20:20

(c) HOCK international, page 143

Page 144: Hock_Performance Mgmt

8/16/2019 Hock_Performance Mgmt

http://slidepdf.com/reader/full/hockperformance-mgmt 144/152

D.

This is not the correct answer. Please see the correct answer for an explanation.

We have been unable to determine how to calculate this incorrect answer choice. If you have calculated it, please letus know how you did it so we can create a full explanation of why this answer choice is incorrect. Please send us anemail at [email protected]. Include the full Question ID number and the actual incorrect answer choice --

not its letter, because that can change with every study session created. The Question ID number appears in the upperright corner of the ExamSuccess screen. Thank you in advance for helping us to make your HOCK study materialsbetter.

Question 207 - CMA 695 3.25 - Manufacturing Input Variances -- Materials and Labor 

Price variances and efficiency variances can be key to the performance measurement within a company. In evaluatingthe performance within a company, a materials efficiency variance can be caused by all of the following except the:

 A. Design of the product.B. Sales volume of the product.C. Actions of the purchasing department.D. Performance of the workers using the material.

 A. The materials efficiency variance could be caused by a number of reasons, one of which is poor design of theproduct. Poor design could make it impossible for the standards to be met if, for instance, two parts did not fit togetherthe way they were supposed to, and in working with them to make them fit, some were broken and became unusable.That would cause excess usage of the materials and unplanned costs.

B. The materials efficiency variance is the difference between the actual material usage and the standardmaterial usage for the actual level of output, multiplied by the standard price. There is no connection betweenthe sales volume of the product and the materials efficiency variance.

C. The materials efficiency variance could be caused by a number of reasons, one of which is poor quality of the

materials, which would be caused by actions of the purchasing department. For example, the purchasing departmentmight purchase inferior materials in an effort to save money. Instead of saving money, the result might be that toomany items purchased would be defective and unusable. This could result in additional costs to make them usable. Or,if the defective items could not be made usable, they would have to be thrown away, causing excess materials usage.

D. The materials efficiency variance could be caused by a number of reasons, one of which is poor workerperformance. If workers assembling the products are not performing their work properly, raw materials could be wastedor ruined.

Question 208 - CMA 691 3.25 - Performance Measures

Residual income is a better measure for performance evaluation of an investment center manager than return oninvestment because

 A. Only the gross book value of assets needs to be calculated.B. Desirable investment decisions will not be neglected by high return divisions.C. The problems associated with measuring the asset base are eliminated.D. Returns do not increase as assets are depreciated.

 A. The asset base or amount of invested capital are calculated in the same manner for both methods.

Part 1 : 10/05/14 22:20:20

(c) HOCK international, page 144

Page 145: Hock_Performance Mgmt

8/16/2019 Hock_Performance Mgmt

http://slidepdf.com/reader/full/hockperformance-mgmt 145/152

B. Residual income (RI) is calculated as the amount of return (net income before taxes) that is in excess of atargeted amount of return on the investments that are employed by that division. RI is focused on the dollaramount of income that is in excess of a targeted amount. It is not focused on a percentage of return as ROI is.When using RI to evaluate investment opportunities, any project that has a positive RI will be accepted even ifit will reduce the overall company or division ROI. Thus, desirable investment decisions will not be neglectedby high return divisions.

C. The asset base or amount of invested capital are calculated in the same manner for both methods, so residualincome does not eliminate this problem.

D. As assets are depreciated, both methods will be affected in the same manner.

Question 209 - CMA 694 3.29 - Performance Measures

One approach to measuring divisional performance is return on investment. Return on investment is expressed asoperating income:

 A. Divided by fixed assets.B. Minus imputed interest charged for invested capital.C. Divided by the current year's capital expenditures plus cost of capital.D. Divided by total assets.

 A. The formula for an individual division's return on investment is not its operating income divided by its fixed assets."Investment" includes more than just fixed assets.

B. The formula for an individual division's return on investment is not its operating income minus imputed interestcharged for invested capital. Operating income minus imputed interest charged for invested capital is residual income.

C. The formula for an individual division's return on investment is not its operating income divided by the current year'scapital expenditures plus cost of capital. "Investment" needs to include more than just the current year's capitalexpenditures, and the cost of capital is not used in the calculation of return on investment.

D. The formula for an individual division's return on investment is its operating income divided by its totalassets (or average total assets, if that is available or can be calculated).

Question 210 - CMA 1290 3.10 - Manufacturing Input Variances -- Overhead

Franklin Glass Works' production budget for the year ended November 30 was based on 200,000 units. Each unitrequires two standard hours of labor for completion. Total overhead was budgeted at $900,000 for the year, and thefixed overhead rate was estimated to be $3.00 per unit. Both fixed and variable overhead are assigned to the producton the basis of direct labor hours. The actual data for the year ended November 30 are presented as follows. Actual production in units 198,000

 Actual direct labor hours 440,000 Actual variable overhead $352,000 Actual fixed overhead $575,000

Franklin's fixed overhead volume variance for the year is

 A. $6,000 unfavorable.B. $19,000 favorable.C. $55,000 unfavorable.D. $25,000 favorable.

Part 1 : 10/05/14 22:20:20

(c) HOCK international, page 145

Page 146: Hock_Performance Mgmt

8/16/2019 Hock_Performance Mgmt

http://slidepdf.com/reader/full/hockperformance-mgmt 146/152

A.

The fixed overhead volume variance (also called the fixed overhead production-volume variance) is thebudgeted amount of fixed overhead minus the amount of fixed overhead applied (standard rate × standardquantity of application base allowed for the actual level of output).

The fixed overhead rate was estimated to be $3.00 per unit and 200,000 units were scheduled for production.The total budgeted fixed overhead was therefore $600,000 ($3.00 × 200,000). Two hours of labor are thestandard required to produce one unit. Thus, the application rate is $1.50 per direct labor hour ($3.00 ÷ 2).

The standard direct labor hours allowed for the actual output were 396,000 (2 × 198,000 actual production inunits). The applied overhead equals $594,000 ($1.50 × 396,000). We also could calculate the amount of appliedoverhead using actual units produced and the unit fixed overhead application rate ($3.00 × 198,000) whichgives the same answer: $594,000.

Thus, the fixed overhead volume variance is $6,000 unfavorable ($600,000 − $594,000). The applied amountwas less than the budgeted amount which means that some amount of overhead wasn't applied to the finalproduct i.e. it was underapplied. The variance is caused by the fact that production was planned to be 200,000units but only 198,000 units were actually produced. The facilities were not used at the level that was planned.

B. The fixed overhead volume variance (also called the fixed overhead production-volume variance) is the budgetedamount of fixed overhead minus the amount of fixed overhead applied (standard rate × standard quantity of applicationbase allowed for the actual level of output). See the correct answer for a complete explanation.

C. The fixed overhead volume variance (also called the fixed overhead production-volume variance) is the budgetedamount of fixed overhead minus the amount of fixed overhead applied (standard rate × standard quantity of applicationbase allowed for the actual level of output). See the correct answer for a complete explanation.

D. The fixed overhead volume variance (also called the fixed overhead production-volume variance) is the budgetedamount of fixed overhead minus the amount of fixed overhead applied (standard rate × standard quantity of applicationbase allowed for the actual level of output). See the correct answer for a complete explanation.

Question 211 - ICMA 10.P1.096 - Manufacturing Input Variances -- Overhead

 A company applies variable ovehead based upon direct labor hours and has a variable overhead efficiency variancethat is $25,000 favorable. A possible cause of this variance is that

 A. electricity rates were lower than expected.B. higher skilled labor was used.C. less supplies were used than anticipated.D. less units of finished goods were produced.

 A.

The variable overhead efficiency variance is calculated as follows:  Budgeted variable overhead based on inputs actually used− Standard variable overhead applied to production= Variable overhead efficiency variance

 Actual electricity rates would not affect this variance, because it is a comparison of a budgeted amount of variableoverhead calculated using direct labor hours actually used with the standard amount of variable overhead calculatedusing direct labor hours allowed for the actual production.

B.

Part 1 : 10/05/14 22:20:20

(c) HOCK international, page 146

Page 147: Hock_Performance Mgmt

8/16/2019 Hock_Performance Mgmt

http://slidepdf.com/reader/full/hockperformance-mgmt 147/152

The variable overhead efficiency variance is calculated as follows:  Budgeted variable overhead based on inputs actually used

− Standard variable overhead applied to production

= Variable overhead efficiency variance

The variable overhead efficiency variance is a comparison of a budgeted amount of variable overhead

calculated using direct labor hours actually used with the standard amount of variable overhead calculatedusing direct labor hours allowed for the actual production. So it is essentially a calculation of the effect of adifference between the number of direct labor hours used with the number of direct labor hours allowed.

A favorable variance means fewer labor hours were used than were allowed for the actual output. This couldbe caused by the use of labor that was more skilled than anticipated and was thus able to work more quickly.

C.

The variable overhead efficiency variance is calculated as follows:  Budgeted variable overhead based on inputs actually used− Standard variable overhead applied to production= Variable overhead efficiency variance

The actual usage of supplies and their associated cost would not affect this variance, because it is a comparison of abudgeted amount of variable overhead calculated using direct labor hours actually used with the standard amount ofvariable overhead calculated using direct labor hours allowed for the actual production.

D.

The variable overhead efficiency variance is calculated as follows:  Budgeted variable overhead based on inputs actually used− Standard variable overhead applied to production= Variable overhead efficiency variance

This variance is a comparison of a budgeted amount of variable overhead, calculated using direct labor hours actuallyused, with the standard amount of variable overhead, calculated using direct labor hours allowed for the actual

production.

The budgeted amount of variable overhead calculated using direct labor hours actually used is the standard applicationrate per direct labor hour multiplied by the number of direct labor hours actually used.

The standard amount of variable overhead calculated using direct labor hours allowed for the actual production is thestandard application rate per direct labor hour multiplied by the number of direct labor hours allowed for the actualproduction.

Fewer units of finished goods being produced than were planned would not affect the variable overhead efficiencyvariance, since a comparison of actual production with planned production is not a part of the calculation of thevariance.

Question 212 - CMA 1291 3.18 - Sales and Market Variances

Folsom Fashions sells a line of women's dresses. Folsom's performance report for November follows.Actual Budget

Dresses sold 5,000 6,000Sales $ 235,000 $ 300,000Variable costs (145,000) (180,000)

Part 1 : 10/05/14 22:20:20

(c) HOCK international, page 147

Page 148: Hock_Performance Mgmt

8/16/2019 Hock_Performance Mgmt

http://slidepdf.com/reader/full/hockperformance-mgmt 148/152

Contribution margin $ 90,000 $ 120,000Fixed costs (84,000) (80,000)Operating income $ 6,000 $ 40,000

The company uses a flexible budget to analyze its performance and to measure the effect on operating income of thevarious factors affecting the difference between budgeted and actual operating income.

What additional information is needed for Folsom to calculate the dollar impact of a change in market share onoperating income for November?

 A. Folsom's budgeted market share and the budgeted total market size.B. Folsom's budgeted market share and the actual total market size.C. Folsom's budgeted market share, the budgeted total market size, and average market selling price.D. Folsom's actual market share and the actual total market size.

 A. The budgeted market share and actual total market size in units have to be known in order to calculate the impact ofa change in market share. See the correct answer for a complete explanation.

B. The market share variance is calculated as follows: [(Actual Market Share − Expected Market Share) ×Actual Market Size in Units] × Standard Weighted Average Contribution Margin per Unit. To determine thedollar impact of a change in market share on operating income, we need to know the actual market size inunits, actual market share, budgeted market shares and standard weighted average contribution margin perunit. In the question, we are given the actual units sold, thus, we can calculate the actual market share if weobtain the additional information of actual market size in units. We also can calculate the standard weightedaverage contribution margin per unit from the given data. To complete the formula we also need to obtaininformation of budgeted (expected) market share.

C. The budgeted market share and actual total market size in units have to be known. See the correct answer for acomplete explanation.

D. The budgeted market share and actual total market size in units have to be known. See the correct answer for acomplete explanation.

Question 213 - CMA 1296 3.2 - Responsibility Centers and Reporting Segments

The segment margin of an investment center after deducting the imputed interest on the assets used by the investmentcenter is known as

 A. Return on investment.B. Residual income.C. Operating income.D. Return on assets.

 A. Return on investment is calculated as income divided by the invested amount. There is no deduction of an imputedinterest rate in the calculation of return on investment.

B. Residual income is the amount of return after a certain required return on the assets in use by the division.The calculation of the required return is based on an imputed interest rate for the cost of funds represented bythe assets in use. This describes the situation in the question.

C. Operating income does not take into account the imputed interest rate on the assets of the investment center. Seethe correct answer for a complete explanation.

D. Return on assets does not take into account the imputed interest rate on the assets of the investment center. Seethe correct answer for a complete explanation.

Part 1 : 10/05/14 22:20:20

(c) HOCK international, page 148

Page 149: Hock_Performance Mgmt

8/16/2019 Hock_Performance Mgmt

http://slidepdf.com/reader/full/hockperformance-mgmt 149/152

Question 214 - CMA 1291 3.2 - Manufacturing Input Variances -- Materials and Labor 

 Arrow Industries employs a standard cost system in which direct materials inventory is carried at standard cost. Arrowhas established the following standards for the prime costs of one unit of product.

 StandardQuantity

StandardPrice

StandardCost

Direct materials 8 pounds$1.80 per pound $14.40Direct labor .25 hours $8.00 per hour 2.00  $16.40

During November, Arrow purchased 160,000 pounds of direct materials at a total cost of $304,000. The total factorywages for November were $42,000, 90% of which were for direct labor. Arrow manufactured 19,000 units of productduring November using 142,500 pounds of direct materials and 5,000 direct labor hours.

The direct materials usage (quantity) variance for November is

 A. $14,400 unfavorable.

B. $17,100 unfavorable.C. $17,100 favorable.D. $1,100 favorable.

 A. This answer results from using the quantity of direct materials purchased instead of the quantity of direct materialsused in production as the actual quantity in the variance formula.

B. The material quantity variance is favorable because the amount of direct material used in production was less thanthe standard amount allowed for the actual production.

C. The material quantity variance (also called the efficiency or usage variance) is the difference between theactual material used and the standard material allowed for this level of output, multiplied by the standardprice; or (AQ − SQ) × SP. The actual amount used was 142,500 pounds. The standard quantity is 8 pounds per

unit, so the standard quantity allowed to produce 19,000 units is 19,000 × 8, or 152,000 pounds. The standardprice is $1.80. So the material quantity variance is (142,500 − 152,000) × $1.80 = $(17,100) favorable. Thevariance is negative, which is favorable for a cost variance, because the actual quantity used was less thanthe standard quantity allowed for the actual output.

D.

This is not the correct answer. See the correct answer for a complete explanation.

We have been unable to determine how to calculate this incorrect answer choice. If you have calculated it, please letus know how you did it so we can create a full explanation of why this answer choice is incorrect. Please send us anemail at [email protected]. Include the full Question ID number and the actual incorrect answer choice --not its letter, because that can change with every study session created. The Question ID number appears in the upperright corner of the ExamSuccess screen. Thank you in advance for helping us to make your HOCK study materials

better.

Question 215 - CMA 1291 3.3 - Manufacturing Input Variances -- Materials and Labor 

 Arrow Industries employs a standard cost system in which direct materials inventory is carried at standard cost. Arrowhas established the following standards for the prime costs of one unit of product.

Part 1 : 10/05/14 22:20:20

(c) HOCK international, page 149

Page 150: Hock_Performance Mgmt

8/16/2019 Hock_Performance Mgmt

http://slidepdf.com/reader/full/hockperformance-mgmt 150/152

 StandardQuantity

StandardPrice

StandardCost

Direct materials 8 pounds$1.80 per pound $14.40Direct labor 0.25 hours $8.00 per hour 2.00  $16.40

During November, Arrow purchased 160,000 pounds of direct materials at a total cost of $304,000. The total factorywages for November were $42,000, 90% of which were for direct labor. Arrow manufactured 19,000 units of productduring November using 142,500 pounds of direct materials and 5,000 direct labor hours.

The direct labor price (rate) variance for November is

 A. $2,200 favorable.B. $2,000 unfavorable.C. $2,090 favorable.D. $1,900 unfavorable.

A. The labor rate/price variance is: (Actual Rate − Standard Rate) × Actual Hours. The actual direct labor costwas $37,800 ($42,000 × 90%) for the month of November. The actual rate was $7.56 ($37,800 ÷ 5,000). Theactual number of hours was 5,000. The labor rate/price variance was ($7.56 − $8.00) × 5,000 = $(2,200)

favorable. A negative variance for a cost is a favorable variance.

B. This answer results from using the total factory wages of $42,000 to calculate the direct labor actual rate. However,only 90% of the total factory wages were direct labor. See the correct answer for a complete explanation.

C. The labor rate/price variance is calculated as: (Actual Rate − Actual Rate) × Actual Hours. This answer results fromusing the hours allowed for the actual output (19,000 × 0.25) instead of the actual hours. See the correct answer for acomplete explanation.

D. The labor rate/price variance is favorable as the actual direct labor rate is lower than the standard. See the correctanswer for a complete explanation.

Question 216 - CMA 693 3.16 - Variance Analysis Concepts

In analyzing company operations, the controller of the Jason Corporation found a $250,000 favorable flexible-budgetrevenue variance. The variance was calculated by comparing the actual results with the flexible budget. This variancecan be wholly explained by

 A. The total sales volume variance.B. Changes in unit selling prices.C. The total static budget variance.D. The total flexible budget variance.

 A.

The sales volume variance measures the impact of the difference between actual and budgeted sales volume. Aflexible budget is a budget that is prepared using the standard costs and the actual level of activity (sales orproduction). It is essentially what the budget would have been if the company had known when it developed the budgetwhat the actual level of activity would be. The flexible budget variance (or total flexible budget variance) is the actualresults minus the flexible budget amount.

Since the actual results are being compared with the flexible budget and the flexible budget amount is adjusted to theactual sales volume, there will be no difference in revenue due to a difference between actual sales volume andbudgeted sales volume. The total sales volume variance will always be zero.

Part 1 : 10/05/14 22:20:20

(c) HOCK international, page 150

Page 151: Hock_Performance Mgmt

8/16/2019 Hock_Performance Mgmt

http://slidepdf.com/reader/full/hockperformance-mgmt 151/152

B.

The simple formula to calculate revenue is Revenue = Quantity × Price. Variances in revenue come from eithervariances in quantity of product sold or variances in selling price.

A flexible budget is a budget that is prepared using the standard costs and the actual level of activity (sales orproduction). It is essentially what the budget would have been if the company had known when it developed

the budget what the actual level of activity would be. The flexible budget variance (or total flexible budgetvariance) is the actual results minus the flexible budget amount.

Since the actual results are being compared with the flexible budget and the flexible budget amount isadjusted to the actual sales volume, there will be no difference in revenue due to a difference between actualsales volume and budgeted sales volume. The difference between the actual revenue and the flexible budgetrevenue can be caused only by a difference between the actual and budgeted selling prices.

C.

The total static budget variance is the difference between the actual results and the amount in the static (master)budget. The total static budget variances simply compare the actual results against the static budget amounts.

 A flexible budget is a budget that is prepared using the standard costs and the actual level of activity (sales orproduction). It is essentially what the budget would have been if the company had known when it developed the budgetwhat the actual level of activity would be. The flexible budget variance (or total flexible budget variance) is the actualresults minus the flexible budget amount.

The total static budget variance for revenue cannot explain the flexible budget variance for revenue.

D. The total flexible budget variance is the same thing as the flexible budget variance. It cannot explain itself.

Question 217 - CMA 1294 3.21 - Responsibility Centers and Reporting Segments

Sherman Company uses a performance reporting system that reflects the company's decentralization of decisionmaking. The departmental performance report shows one line of data for each subordinate who reports to the groupvice president. The data presented show the actual costs incurred during the period, the budgeted costs, and allvariances from budget for that subordinate's department. Sherman is using a type of system called

 A. Cost-benefit accounting.B. Responsibility accounting.C. Contribution accounting.D. Flexible budgeting.

 A. This is not an accounting method.

B. Responsibility accounting is a system in which costs are allocated to managers and/or departments basedon who is responsible for the incurrence of the costs. This is the method described in the question.

C. Contribution margin analysis focuses on the variable revenues and costs and their behavior as activity levelschange. The selling price minus variable costs is the contribution margin. Fixed costs are then subtracted from thecontribution to determine the profit.

D. Flexible budgeting is the adaptation of the static budget to the level of activity actually achieved during the period.This is not what is described in the question.

Part 1 : 10/05/14 22:20:20

(c) HOCK international, page 151

Page 152: Hock_Performance Mgmt

8/16/2019 Hock_Performance Mgmt

http://slidepdf.com/reader/full/hockperformance-mgmt 152/152

Question 218 - ICMA 10.P1.125 - Responsibility Centers and Reporting Segments

Happy Time Industries uses segment reporting for all of its decentralized divisions. It has several products that aretransferred from one division to other divisions. Happy Time wants to motivate the manager of the selling division toproduce efficiently. Assuming the following methods are available, the optimal transfer pricing method should be a

 A. cost-based transfer price that uses budgeted amounts.B. cost-based transfer price that uses actual amounts.C. variable cost-based transfer price that uses actual amounts.D. market-based transfer price.

 A. A cost-based transfer price is not the optimal transfer price to use if a market price is available.

B. A cost-based transfer price is not the optimal transfer price to use if a market price is available.

C. A cost-based transfer price is not the optimal transfer price to use if a market price is available.

D. When there is an external market for the product, that is almost always the best transfer price to use forprofitability and performance measurement, because it is objective.

Question 219 - CMA 1295 3.11 - Manufacturing Input Variances -- Materials and Labor 

In a standard cost system, the investigation of an unfavorable material usage variance should begin with the

 A. Plant controller only.B. Production manager only.C. Production manager or the purchasing manager.D. Purchasing manager only.

Part 1 : 10/05/14 22:20:20